You are on page 1of 171

Cytopathology

Review
Cytopathology
Review

Fang Fan MD, PhD


Associate Professor of Pathology
The University of Kansas School of Medicine
Kansas City, Kansas, USA

Ivan Damjanov MD, PhD


Professor of Pathology
Department of Pathology and Laboratory Medicine
The University of Kansas School of Medicine
Kansas City, Kansas, USA

JAYPEE BROTHERS MEDICAL PUBLISHERS (P) LTD


New Delhi • Panama City • London
®

Jaypee Brothers Medical Publishers (P) Ltd.

Headquarter
Jaypee Brothers Medical Publishers (P) Ltd
4838/24, Ansari Road, Daryaganj
New Delhi 110 002, India
Phone: +91-11-43574357
Fax: +91-11-43574314
Email: jaypee@jaypeebrothers.com

Overseas Offices
J.P. Medical Ltd., Jaypee-Highlights Medical Publishers Inc.
83 Victoria Street London City of Knowledge, Bld. 237, Clayton
SW1H 0HW (UK) Panama City, Panama
Phone: +44-2031708910 Phone: 507-317-0160
Fax: +02-03-0086180 Fax: +50-73-010499
Email: info@jpmedpub.com Email: cservice@jphmedical.com

Website: www.jaypeebrothers.com
Website: www.jaypeedigital.com

© 2012 Jaypee Brothers Medical Publishers

All rights reserved. No part of this book may be reproduced in any form or by any means without the prior permission of
the publisher.

Inquiries for bulk sales may be solicited at: jaypee@jaypeebrothers.com

This book has been published in good faith that the contents provided by the contributors contained herein are original,
and is intended for educational purposes only. While every effort is made to ensure accuracy of information, the publisher
and the contributors specifically disclaim any damage, liability, or loss incurred, directly or indirectly, from the use or
application of any of the contents of this work. If not specifically stated, all figures and tables are courtesy of the author(s).
Where appropriate, the readers should consult with a specialist or contact the manufacturer of the drug or device.

Cytopathology Review

First Edition : 2012

ISBN-13: 978-93-5025-559-9

Printed in Ajanta Offset


Contributors
Fang Fan MD, PhD
Associate Professor of Pathology
The University of Kansas School of Medicine
Kansas City, Kansas, USA

Rashna Madan MD
Assistant Professor of Pathology
The University of Kansas School of Medicine
Kansas City, Kansas, USA

Maura O’Neil MD
Assistant Professor of Pathology
The University of Kansas School of Medicine
Kansas City, Kansas, USA

Ivan Damjanov MD, PhD


Professor of Pathology
Department of Pathology and Laboratory Medicine
The University of Kansas School of Medicine
Kansas City, Kansas, USA
Preface
Diagnostic cytopathology was once considered an esoteric diagnostic discipline and was practiced by only a few
chosen enthusiasts working in highly specialized cancer centers. Over the last 40 years, cytopathology has become
a major subspecialty of pathology widely used in all medical institutions worldwide. Originally limited to
examinations of “Pap-smears” obtained by exfoliative sampling of the cervix and vagina, modern cytopathology
has expanded its scope by widespread use of fine-needle aspiration techniques, or during other diagnostic procedures
such as entero-retrograde-cholangio-pancreatoscopy, ultrasound or computer tomography guided biopsies. It is also
used as an aid to intraoperative pathologic diagnosis most notably in form of “squash-cytology preparations” of
intracerebral lesions sampled during neurosurgery, or lymph node biopsy for lymphoma, just to mention a few.
Modern cytopathology examinations are often combined with other diagnostic ancillary methods such as
immunocytochemistry, flow cytometry, or molecular biology. These technical advances have dramatically changed
the practice of diagnostic pathology and have had a profound effect on our training of pathology residents.
Cytopathology rotations are today required parts of all residency training programs in Anatomic Pathology, and the
trainees are expected to master cytopathology for their Specialty Boards Examination in Anatomic Pathology.
Furthermore, a subspecialty fellowship in Cytopathology has been officially established qualifying the fellows after
one year of training for a Subspecialty Board Examination in Cytopathology.
In parallel with the changes in the practice of diagnostic pathology and the training of pathology residents, the
publishing industry has launched several subspecialty journals devoted to cytopathology. Almost all major medical
publishers have also produced cytopathology textbooks and monographs, which are prominently displayed on their
publication lists. Practicing pathologists and their trainees looking for excellent textbooks or atlases of cytopathology
could find them without any problem in bookstores or online. Many of these outstanding textbooks have been
already reprinted in several editions. On the other hand and in contrast to this plethora of textbooks, it is worth a
notice that currently there are only a few Cytopathology Subspecialty Board Preparatory texts based on questions
and answers. Cognizant of this fact and aware of the need for Cytopathology question and answers books, we
undertook this task and are herewith pleased to present it to potential users.
Our book is divided into 12 chapters covering all major aspects of diagnostic cytopathology. Each chapter
contains theoretical questions as well as questions pertaining to the microphotographs obtained from smears collected
in our daily cytopathology practice. The questions are presented in the standardized multiple choice format and
resemble in form and content those that the trainees could expect to encounter on the examination by the American
Board of Pathology, including the specialty examination in Anatomic Pathology, as well as the subspecialty
examination in cytopathology or examinations for the Maintenance of Certification (MOC).
Most of our questions were tested out on our pathology residents and fellows in Cytopathology. We are pleased
to report that such practice exams administered over many years have helped many of our trainees prepare for the
American Board of Pathology Examinations, as well as the Residents’ In-service Examinations provided by the
American Society for Clinical Pathology. We hope that the present collections of test questions, culled from our
teaching files, or based on our daily teaching and practice of cytopathology will likewise help the readers of this
book in their endeavors to obtain specialty and subspecialty certifications. If the trainees find it useful maybe it
viii Cytopathology Review

will reach their teachers as well—we are most eager to hear the comments and remarks of our peers. We hope also
that the book will be read by our other more seasoned colleagues and we think that it might be most useful to
especially Board Certified Pathologists preparing for recertification by the American Board of Pathology or the
MOC.
Our special thanks go to Mr Dennis Friesen for his tremendous work in adjusting and organizing all the
microscopic images to ensure high quality; and Dr Marilee Means for her valuable collections of study sets throughout
her 20 years of service as the Program Director of the Cytotechnology School at the University of Kansas Medical
Center.
We hope that our readers will not be disappointed and that they will not find too many mistakes, which tend to
creep into any text, by omission or commission. All your comments, be they positive or negative are nevertheless
welcome and could be sent by e-mail to ffan@kumc.edu. Until then, we hope you find our book useful and enjoyable.

Fang Fan
Ivan Damjanov
Contents
1. Cervical and Vaginal Cytology ....................................................................................................... 1
Fang Fan

2. Respiratory Tract .......................................................................................................................... 26


Fang Fan

3. Urine and Bladder Washings ....................................................................................................... 45


Fang Fan

4. Effusions and Peritoneal Washings ............................................................................................. 56


Fang Fan

5. Cerebrospinal Fluid ....................................................................................................................... 71


Fang Fan

6. Breast .............................................................................................................................................. 79
Fang Fan

7. Thyroid ........................................................................................................................................... 94
Rashna Madan

8. Salivary Gland ............................................................................................................................. 110


Maura O’Neil

9. Lymph Nodes ............................................................................................................................... 124


Fang Fan

10. Liver .............................................................................................................................................. 136


Maura O’Neil

11. Gastrointestinal Tract, Pancreas and Biliary Tree .................................................................. 146


Fang Fan

12. Laboratory Management, Quality Control and Quality Assurance ....................................... 157
Ivan Damjanov
1 Cervical and
Vaginal Cytology
Fang Fan

QUESTIONS
1. Which of the following guidelines should be followed in order to obtain an ideal Pap smear specimen?
A. A lubricated speculum is favored over a non-lubricated speculum
B. The sample should be obtained after the application of acetic acid
C. Excess mucus or other discharge should be retained for examination
D. An optimal specimen includes cells exclusively from the ectocervix
E. Two-weeks after the last menstrual period (LMP) is the preferred examination time

2. According to the specimen adequacy criteria in the 2001 Bethesda System, which of the following Pap
smear specimens is considered unsatisfactory for evaluation?
A. Thin-Prep smear containing approximately 6000 squamous cells
B. Conventional smear containing approximately 6000 squamous cells
C. Smear without endocervical cells
D. Smear with 50% of epithelial cells obscured by inflammation
E. Smear with 50% of epithelial cells obscured by drying artifact

3. How many cells must be present on a liquid-based Pap slide so that the specimen to be considered
satisfactory for diagnosis?
A. 1,000
B. 2,500
C. 5,000
D. 8,000
E. 10,000

4. The Pap smear from a 38-year-old woman showed cohesive flat sheets of cells with large nuclei, pale
chromatin, abundant cytoplasm and prominent nucleoli. Occasional mitoses were seen. These features
are characteristic of which of the following?
A. Atrophy
B. Repair
C. Atypical endocervical cells, not otherwise specified
D. Endocervical adenocarcinoma
E. Squamous cell carcinoma

5. Which of the following cytologic features is characteristic of radiation changes seen in a Pap smear?
A. High nuclear-to-cytoplasmic ratio
B. Abundant isolated cells
C. Cytoplasmic vacuolization and polychromasia
D. Frequent mitoses
E. Coarsely granular and hyperchromatic chromatin
2 Cytopathology Review

6. Women born to mothers who took diethylstilbestrol (DES) during pregnancy would most likely show
what type of changes in their VAGINAL smear specimens?
A. Shift in vaginal flora
B. Low-grade squamous intraepithelial lesion (LSIL)
C. High-grade squamous intraepithelial lesion (HSIL)
D. Atrophic vaginitis
E. Presence of glandular cells

7. HPV infection is established in which layer of the squamous epithelium?


A. Anucleated surface keratin layer
B. Parakeratotic surface squamous cells
C. Superficial squamous cells
D. Intermediate squamous cells
E. Basal squamous cells

8. Which of the following statements is true regarding cervical oncogenesis and the genome of human
papillomavirus (HPV)?
A. The E1 and E2 are most responsible for the cervical oncogenesis
B. The E3 and E4 are the principal components of the HPV vaccines
C. L1 binds to the retinoblastoma tumor suppression protein pRB and abolishes cell-cycle arrest
D. E6 binds to p53 and results in blocking of apoptosis
E. E7 binds to p16 and results in proliferation of cells

9. Which of the following statements is true regarding HPV?


A. HPV is a small RNA virus
B. HPV 6, 11 is the most common subtypes detected in cervical cancer
C. Only a minority of HPV infections persist and lead to cancer
D. Koilocytes are an artifact of fixation and not related to HPV infection
E. Serology is an accurate way to detect HPV infection

10. Which of the following is the most important cytomorphologic feature for high-grade squamous
intraepithelial lesion (HSIL)?
A. Prominent nucleoli
B. Syncytial cell growth
C. Prominent cytoplasmic vacuoles
D. Large cell size
E. High nuclear-to-cytoplasmic ratio

11. Which of the following statements is true about the follow-up management after a Pap smear
interpretation of low-grade squamous intraepithelial lesion (LSIL)?
A. Adolescents with LSIL are managed less aggressively
B. Postmenopausal women with LSIL are managed more aggressively
C. Pregnant women with LSIL are managed more aggressively
D. HPV testing is recommended for premenopausal women with LSIL
E. Diagnostic excisional procedures are routinely used in women with LSIL
Cervical and Vaginal Cytology 3

12. A 30-year-old woman had a routine Pap smear. The smear showed cells of squamous metaplasia with
some degree of nuclear atypia, concerning but not definitive for the diagnosis of high grade dysplasia.
How these cells are best classified?
A. Atypical squamous cells of undetermined significance (ASCUS)
B. Atypical squamous cells cannot exclude HSIL (ASC-H)
C. Low-grade squamous intraepithelial lesion (LSIL)
D. High-grade squamous intraepithelial lesion (HSIL)
E. Squamous cell carcinoma

13. A 25-year-old pregnant woman had a Pap smear which was interpreted as high-grade squamous
intraepithelial lesion (HSIL). Which of the following statements is correct for pregnant women with
HSIL?
A. Colposcopy during pregnancy is not recommended
B. Diagnostic excisional procedure should only be performed postpartum
C. Colposcopy and repeat Pap testing should be performed immediately postpartum
D. Biopsy of lesions suspicious for CIN 2 or CIN 3, if feasible
E. Endocervical curettage (ECC) should be always performed

14. Which of the following is a specific cytomorphologic feature for squamous intraepithelial lesion (SIL)?
A. Perinuclear halos
B. Nuclear enlargement
C. Prominent nucleoli
D. Hyperchromasia and irregular nuclear membrane
E. Finely granular chromatin

15. Which HPV subtype accounts for the majority of cervical squamous cell carcinoma worldwide?
A. HPV 6
B. HPV 11
C. HPV 16
D. HPV 18
E. HPV 31

16. Which of the following cytologic features may help distinguish squamous cell carcinoma (SCC) from
high-grade squamous intraepithelial lesion (HSIL)?
A. Tumor diathesis
B. High nuclear-to-cytoplasmic ratio
C. Abundant keratinizing cytoplasm
D. Hyperchromasia
E. Irregular cell shapes

17. Prominent macronucleoli are cytologic features of cells classified under which one of the following
diagnoses?
A. Low-grade squamous intraepithelial lesion (LSIL) cells
B. High-grade squamous intraepithelial lesion (HSIL) cells
C. Parabasal squamous cells in atrophy
D. Squamous cells in atypical repair
E. Benign endometrial cells
4 Cytopathology Review

18. “Feathering” is a term used to describe Pap smear findings in which of the following entities ?
A. Tubal metaplasia
B. Endocervical adenocarcinoma in situ (AIS)
C. Exfoliated endometrial cells
D. High-grade squamous intraepithelial lesion (HSIL)
E. Invasive adenocarcinoma

19. Which of the following features is more typical of neoplastic endocervical cells (AGUS or AIS) than
reactive/reparative endocervical cells?
A. Flat sheet arrangement
B. Prominent nucleoli
C. Marked variation in nuclear size
D. Elongated and hyperchromatic nuclei
E. Presence of mitosis

20. When differentiating endocervical adenocarcinoma from endometrial adenocarcinoma, which of the
following features favor endometrial rather than endocervical adenocarcinoma?
A. Prominent nucleoli
B. Mitoses
C. Tumor diathesis
D. Intracytoplasmic neutrophils
E. Abundant cytoplasm

21. Which of the following represents the most common metastatic tumor to cervix?
A. Metastatic breast carcinoma
B. Metastatic melanoma
C. Metastatic rectal carcinoma
D. Metastatic ovarian carcinoma
E. Metastatic urothelial carcinoma

22. A 30-year-old woman had a Pap test showing high-grade intraepithelial lesion at the time of her annual
gynecologic examination. She underwent a colposcopic examination. A biopsy of the cervix showed
moderate squamous dysplasia (CIN 2). What is the most appropriate management recommendation
for this woman?
A. Repeat the Pap test in 6 months
B. Perform the loop electrosurgical excision procedure (LEEP)
C. High-risk HPV test
D. Hysterectomy
E. Endocervical curettage

23. What is the most common anatomic location of cervical squamous intraepithelial neoplastic (CIN)
lesions?
A. Lateral side of ectocervix
B. The transformation zone/squamocolumnar junction
C. Lower end of the endocervical canal
D. Upper end of the endocervical canal
E. Vaginal fornix
Cervical and Vaginal Cytology 5

24. Which of the following statements is true about the currently used HPV vaccine?
A. The vaccine covers all high-risk HPV subtypes
B. Women are not protected if they have been infected by the same HPV types prior to vaccination
C. The vaccine is approved for use in sexually active women of all ages
D. The vaccine contains inactivated live viruses subtypes 16 and 18
E. The vaccine is given orally in three doses over a 6-month period

25. Dark blue or purple calcific spheres with concentric laminations are rarely encountered in Pap smears.
What is the clinical significance of the finding?
A. It represents a rare contaminant of no clinical significance
B. It represents a rare microorganism
C. It may be associated with endometriosis
D. It may be associated with an endocrine disorder
E. It may be associated with an ovarian neoplasm

26. What is the recommended initial workup for women with a Pap smear diagnosis of atypical endometrial
cells?
A. Reflex high-risk HPV testing
B. Immediate repeat Pap smear
C. Colposcopic examination
D. Endometrial and endocervical biopsy
E. Simple hysterectomy

27. Hyperchromatic crowded groups (HCGs) seen in a Pap smear under low power are helpful diagnostic
features for which of the following conditions or diseases?
A. Atypical squamous cells of undetermined significance (ASCUS)
B. Low-grade squamous intraepithelial lesion (LSIL)
C. High-grade squamous intraepithelial lesion (HSIL)
D. Repair
E. Radiation changes

28. When differentiating endocervical adenocarcinoma from endometrial adenocarcinoma, which positive
immunohistochemical stain favors the diagnosis of endocervical adenocarcinoma?
A. Estrogen receptor (ER)
B. Vimentin
C. p16
D. Pancytokeratin
E. Beta-catenin
6 Cytopathology Review

29. These cells (arrows) in a ThinPrep smear represent


which of the following?
A. Metaplastic squamous cells
B. Superficial squamous cells
C. Intermediate squamous cells
D. Endocervical cells
E. Endometrial cells

30. The group of epithelium shown here represents which


of the following?
A. Atypical squamous cells of undetermined signi-
ficance (ASC-US)
B. High-grade squamous intraepithelial lesion (HSIL)
C. Endocervical adenocarcinoma in situ (AIS)
D. Parabasal squamous cells in atrophy
E. Repair

31. The arrow points to a group of squamous cells


showing sign of which process?
A. Hyperkeratosis
B. Parakeratosis
C. Atrophy
D. Repair
E. Metaplasia
Cervical and Vaginal Cytology 7

32. This group of cells (arrow) represents which of the


following?
A. Benign endocervical cells with tubal metaplasia
B. Atypical endocervical cells, not otherwise specified
(AGUS, NOS)
C. Benign endometrial cells
D. Atypical endometrial cells
E. Adenocarcinoma

33. This Pap smear was obtained from a 32-year-old


woman, on day 6 of menstrual cycle. What are the
cells indicated by the arrow?
A. High-grade squamous intraepithelial lesion (HSIL)
B. Small cell carcinoma
C. Squamous cell carcinoma
D. Benign endometrial cells
E. Benign endocervical cells

34. This Pap smear obtained from a 64-year-old woman


is best interpreted as which of the following?
A. Benign endometrial cells
B. Lymphoid aggregates indicative of follicular
cervicitis
C. High-grade squamous intraepithelial lesion (HSIL)
D. Endometrial adenocarcinoma
E. Lymphoma
8 Cytopathology Review

35. The cells seen here are covered by coccobacilli due


to a shift in vaginal flora. Which term is best used to
denote these cells?
A. Cornflake
B. Koilocytes
C. Navicular cells
D. Clue cells
E. Blue blobs

36. This cervical vaginal smear was obtained from a 35-


year-old woman with a frothy vaginal discharge.
These findings are most consistent with an infection
with which one of the following pathogens?
A. Herpes simplex virus
B. Candida albicans
C. Human papilloma virus
D. Gardnerella vaginalis
E. Trichomonas vaginalis

37. This vaginal smear was obtained from a 55-year-old


diabetic woman complaining of vaginal pruritus and
white discharge. These findings are most consistent
with an infection with which one of the following
pathogens?
A. Herpes simplex virus
B. Candida albicans
C. Human papilloma virus
D. Gardnerella vaginalis
E. Trichomonas vaginalis
Cervical and Vaginal Cytology 9

38. This vaginal smear was obtained from a 23-year-old


woman complaining of occasional genital discomfort
and dyspareunia. These cytologic findings are most
consistent with an infection with which one of the
following pathogens?
A. Herpes simplex virus
B. Candida albicans
C. Human papilloma virus
D. Gardnerella vaginalis
E. Trichomonas vaginalis

39. The cells in the center of this image represent which


of the following?
A. Normal intermediate squamous cells
B. Atypical squamous cells of undetermined signi-
ficance (ASCUS)
C. Low-grade squamous intraepithelial lesion (LSIL)
D. Normal endocervical cells
E. Normal endometrial cells

40. A 28-year-old woman presented for routine Pap


smear. The cytology findings are shown here. What
is the most appropriate next step in the management
of this patient?
A. Routine Pap smear in one year
B. HPV testing on the Pap smear specimen
C. Colposcopic examination
D. Cervical loop electrosurgical excision procedure
(LEEP)
E. Endometrial sampling
10 Cytopathology Review

41. This slide shows a Pap smear from a 23-year-old


woman. What is the proper follow up management
for women with such findings?
A. Send the specimen for HPV DNA testing
B. Repeat Pap smear in one year
C. Repeat Pap smear in six months
D. Colposcopic examination should be performed
E. Offer HPV vaccine immunization

42. What is the correct interpretation of this Pap smear


from a 40-year-old woman?
A. Low-grade squamous intraepithelial lesion (LSIL)
B. High-grade squamous intraepithelial lesion (HSIL)
C. Atrophy
D. Repair
E. Endocervical adenocarcinoma in situ (AIS)

43. What is the correct interpretation of this Pap smear


from a 34-year-old woman?
A. Atypical squamous cell of uncertain significance
(ASCUS)
B. Low-grade squamous intraepithelial lesion (LSIL)
C. High-grade squamous intraepithelial lesion (HSIL)
D. Atrophy
E. Repair
Cervical and Vaginal Cytology 11

44. This cervical smear was obtained from a 23-year-old


sexually active nulliparous woman. What is the
correct diagnosis?
A. Atypical glandular cells of undetermined significance
(AGUS)
B. Low-grade squamous intraepithelial lesion (LSIL)
C. High-grade squamous intraepithelial lesion (HSIL)
D. Atypical squamous cells of undetermined signi-
ficance (ASCUS)
E. Benign reactive squamous cells

45. 42-year-old woman, with no significant history, had


a routine Pap smear which is shown here. What is
the interpretation of these findings?
A. Low-grade squamous intraepithelial lesion (LSIL)
B. Endocervical adenocarcinoma
C. Squamous cell carcinoma
D. High-grade squamous intraepithelial lesion (HSIL)
E. Atypical endocervical cells, favor neoplastic

46. 55-year-old woman had a routine Pap smear. There


were no prior Pap smears on file. What is the correct
interpretation of the cells in this Pap smear?
A. Reactive endocervical cells
B. Atypical endocervical cells
C. Atypical endometrial cells
D. Atypical squamous cells of undetermined
significance (ASCUS)
E. High-grade squamous intraepithelial lesion (HSIL)
12 Cytopathology Review

47. What is the correct diagnosis of this Pap smear


obtained from a 62-year-old woman?
A. Low-grade squamous intraepithelial lesion (LSIL)
B. Atrophy
C. Squamous cell carcinoma
D. High-grade squamous intraepithelial lesion (HSIL)
E. Trichomonas infection

48. The cell shown here (arrow) is most commonly


associated with which of the following?
A. Intrauterine device (IUD)
B. Herpes virus infection
C. Human papilloma virus infection
D. Oral contraceptive use
E. Postpartum state

49. What is the correct interpretation of this Pap smear


from a 22-year-old pregnant woman?
A. Normal
B. Atypical squamous cells of undetermined signi-
ficance (ASCUS)
C. Atypical squamous cells cannot exclude HSIL (ASC-
H)
D. Low-grade squamous intraepithelial lesion (LSIL)
E. High-grade squamous intraepithelial lesion (HSIL)
Cervical and Vaginal Cytology 13

50. This Pap smear was obtained from a 45-year-old


woman. What is the correct interpretation?
A. Repair
B. Tubal metaplasia
C. Atypical squamous cells can not exclude HSIL
(ASC-H)
D. Endocervical adenocarcinoma in situ (AIS)
E. Exfoliated endometrial cells

51. A 50-year-old woman presented with abnormal


bleeding. A Pap smear was obtained and is shown
here. What is the correct interpretation?
A. High-grade squamous intraepithelial lesion (HSIL)
B. Endocervical adenocarcinoma in situ (AIS)
C. Reactive endocervical cells
D. Benign endometrial cells
E. Endometrial adenocarcinoma

52. The abnormal finding shown in this Pap smear


(arrow) is most frequently associated with which of
the following?
A. HPV infection
B. Trichomonas vaginalis infection
C. Intrauterine device (IUD) use
D. Pregnancy
E. Diabetes
14 Cytopathology Review

53. This Pap smear was obtained as a routine Pap smear


from a 24-year-old woman. What is your inter-
pretation?
A. Negative for intraepithelial lesion or malignancy
B. Atypical squamous cells can not exclude high-grade
squamous intraepithelial lesion (ASC-H)
C. Low-grade squamous intraepithelial lesion (LSIL)
D. High-grade squamous intraepithelial lesion (HSIL)
E. Repair

54. What is the best interpretation for this Pap smear


obtained from a 55-year-old woman?
A. Atrophy
B. Atypical squamous cells of undetermined signifi-
cance (ASCUS)
C. Atypical squamous cells cannot exclude HSIL (ASC-
H)
D. High-grade squamous intraepithelial lesion (HSIL)
E. Atypical endometrial cells

55. What is the correct diagnosis of these cells from a Pap


smear obtained from a 42-year-old woman?
A. Benign endometrial cells
B. Lymphoid aggregate indicative of follicular cervicitis
C. Reparative squamous cells
D. Reactive endocervical cells
E. Parabasal squamous cells
Cervical and Vaginal Cytology 15

56. A 72-year-old woman who had a clinical history of


rectal adenocarcinoma presented with vaginal
bleeding. The vaginal smear is shown here. What is
the most likely diagnosis?
A. Endocervical adenocarcinoma
B. Endometrial adenocarcinoma
C. Squamous cell carcinoma
D. Metastatic adenocarcinoma
E. Lymphoma

57. What is the correct diagnosis of this Pap smear?


A. High-grade squamous intraepithelial lesion (HSIL)
B. Squamous cell carcinoma
C. Endometrial adenocarcinoma
D. Metastatic carcinoma
E. Melanoma

58. What is the proper name for the most prominent cells
in this Pap smear obtained from a 35-year-old
woman?
A. Koilocytes
B. Navicular cells
C. Clue cells
D. Cornflake cells
E. Tadpole cells
16 Cytopathology Review

59. What is the correct interpretation of this Pap smear


obtained from a 54-year-old woman?
A. Radiation changes
B. Cytomegalovirus-associated changes
C. Intrauterine device (IUD) effect
D. Adenocarcinoma
E. Squamous cell carcinoma

60. What are these cells in a Pap smear obtained from a


47-year-old woman?
A. Endometrial cell
B. Parabasal squamous cells
C. Lymphocytes
D. High-grade squamous intraepithelial lesion (HSIL)
cells
E. Low-grade squamous intraepithelial lesion (LSIL)
cells

61. What is the correct diagnosis for this Pap smear


obtained from a 54-year-old woman?
A. Parakeratosis
B. Atrophic vaginitis
C. Atypical squamous cells of undetermined signi-
ficance (ASCUS)
D. Atypical squamous cells cannot exclude HSIL (ASC-
H)
E. Squamous cell carcinoma (SCC)
Cervical and Vaginal Cytology 17

62. What is the best designation for the cells indicated


by the arrow?
A. Normal
B. Atypical squamous cells of undetermined
significance (ASCUS)
C. Atypical squamous cells cannot exclude HSIL (ASC-
H)
D. Low-grade squamous intraepithelial lesion (LSIL)
E. Repair

63. This Pap smear was obtained from a 46-year-old


nulliparous woman. This hyperchromatic crowded
group (HCG) is most consistent with which diagnosis?
A. Endometrial adenocarcinoma
B. Low-grade squamous intraepithelial lesion (LSIL)
C. High-grade squamous intraepithelial lesion (HSIL)
D. Parabasal squamous cells in atrophy
E. Benign endometrial cells

64. This Pap smear was obtained from a 62-year-old


woman. What is the most appropriate interpretation
of this epithelial group?
A. Benign endometrial cells
B. Parabasal squamous cells in atrophy
C. Endometrial adenocarcinoma
D. High-grade squamous intraepithelial lesion (HSIL)
E. Endocervical adenocarcinoma
18 Cytopathology Review

65. This Pap smear was obtained from a 36-year-old


woman during her routine annual examination. What
is the correct diagnosis?
A. Benign endometrial cells
B. Parabasal squamous cells in atrophy
C. Endometrial adenocarcinoma
D. High-grade squamous intraepithelial lesion (HSIL)
E. Endocervical adenocarcinoma

66. What is the best designation for this cell group in a


Pap smear from a 50-year-old woman?
A. High-grade squamous intraepithelial lesion (HSIL)
B. Endometrial adenocarcinoma
C. Parabasal squamous cells in atrophy
D. Reactive endocervical cells
E. Endocervical adenocarcinoma in situ (AIS)

67. A 43-year-old woman presents for routine screening.


What is the diagnosis for her Pap smear?
A. Reactive endocervical cells
B. Endocervical adenocarcinoma
C. Repair
D. Radiation changes
E. Intrauterine device (IUD) effect
Cervical and Vaginal Cytology 19

68. What is the correct interpretation of this Pap smear?


A. Atypical squamous cells of undetermined
significance (ASCUS)
B. Atypical endocervical cells
C. Atypical endometrial cells
D. Repair
E. Atrophy

69. A 55-year-old woman had hysterectomy for


endometrial adenocarcinoma a year ago. This is her
follow up vaginal smear. What is the correct
diagnosis?
A. Recurrent endometrial adenocarcinoma
B. Atypical squamous cells of undetermined
significance (ASCUS)
C. Atypical squamous cells cannot exclude HSIL
(ASC-H)
D. Radiation changes
E. Benign glandular cells status post hysterectomy

70. This Pap smear was obtained from a 47-year-old


woman. What is the correct diagnosis?
A. Metastatic adenocarcinoma
B. Lymphoma
C. Small cell carcinoma
D. Leiomyosarcoma
E. Melanoma
20 Cytopathology Review

ANSWERS

1. E 5. C
Note: As established by the Clinical and Laboratory Note: Characteristic cytomorphologic features of
Standards Institute, the following guidelines should be radiation changes include normal N/C ratio, finely
followed to obtain an ideal Pap smear: granular chromatin, cytoplasmic vacuolization and
 The examination should be scheduled 2 weeks after polychromatia, and multinucleation.
the first day of the last menstrual period. It is
preferable to avoid examination during menses. 6. E
 Specimens should be obtained after insertion of a non- Note: About one-third of the DES-daughters develop
lubricated speculum. Excess mucus or other discharge vaginal adenosis, characterized by the formation
should be removed gently. of glands in the vaginal mucosa. Most of them are
 The sample should be obtained before the application benign. Clear cell carcinoma of the vagina is the least
of acetic acid or Lugol iodine. common but the worst complication of in utero DES
 An optimal sample should include cells from the exposure.
ectocervix and endocervix.
7. E
2. B Note: HPV infection is established in the basal layers
Note: In the 2001 Bethesda System, the “satisfactory for of the squamous epithelium. As the epithelium matures
evaluation category” implies that a satisfactory toward the surface, gene amplification and viral
squamous component is present (the minimum number assembly occur, leading to eventual viral release.
of squamous cells for adequacy for liquid-based smears
is 5000 and for conventional smears is 8000 to 12,000). 8. D
The specimen is considered unsatisfactory for evaluation Note: The E6 and E7 genes are most responsible for the
if obscuring elements cover more than 75% of epithelial cervical oncogenesis. E6 binds to p53 and E7 binds to
cells. A smear without endocervical cells is not pRB thus leading to host cell transformation. L1 is the
considered unsatisfactory; instead the absence of an major viral capsid protein and is the principal component
endocervical or transformation zone is mentioned as a of the HPV vaccines.
“quality indicator” in the report.
9. C
3. C Note: HPV is a circular double-stranded DNA virus.
Note: In the 2001 Bethesda System, the “satisfactory for HPV 16 is the most common subtype detected in cervical
evaluation category” implies that a satisfactory cancer. HPV 6,11 are low-risk HPV viruses. Koilocytes
squamous component is present (the minimum number are associated with HPV virus infection. Only 50-60%
of squamous cells for adequacy for liquid-based of infected women have circulating HPV antibodies.
smears is 5000 and for conventional smears is 8000 to
12,000). 10. E
Note: Characteristic cytomorphologic features of HSIL
4. B include high N/C ratio, cells of parabasal-size, marked
Note: The features described are characteristic of repair. irregular nuclear membrane, coarse chromatin and
Parabasal squamous cells in atrophy have moderate hyperchromasia.
amount of cytoplasm without prominent nucleoli.
Atypical endocervical cells have elongated hyper- 11. A
chromatic nuclei and scant cytoplasm. Endocervical Note: Except for adolescents and postmenopausal
adenocarcinoma contains crowded groups of cells or women, colposcopy is recommended for women with
cells forming microacini or rosettes. Squamous cell LSIL. HPV testing and routine diagnostic excisional
carcinoma has necrotic debris, abundant singly dispersed procedures are not recommended for women with
malignant cells with coarsely granular chromatin. LSIL.
Cervical and Vaginal Cytology 21

12. B reparative processes. Mitoses may occur in both


Note: ASC-H is used when metaplastic squamous cells neoplastic and reactive endocervical cells.
or atrophic squamous epithelium show certain degree
of atypia, impossible to be distinguished from HSIL. 20. D
Note: Tumor diathesis can be seen in all invasive
13. D carcinomas. Prominent nucleolus, mitosis and abundant
Note: Colposcopy should be performed by an cytoplasm are features of both endocervical and endo-
experienced physician on all women found to have metrial adenocarcinomas. However, intracytoplasmic
HSIL in a Pap smear. Any lesion suspicious for CIN 2 , neutrophils are more commonly seen in endometrial
CIN 3 or cancer should be biopsied if feasible. ECC is adenocarcinoma cells.
generally unacceptable during pregnancy. If invasive
cancer is suspected, a diagnostic excisional procedure 21. D
is acceptable. If CIN 2,3 is not found histologically,
reevaluation with colposcopy and a Pap smear is 22. B
recommended no sooner than 6 weeks postpartum. Note: Loop electrosurgical excision procedure (LEEP)
is currently the treatment of choice for CIN 2 and CIN 3.
14. D
Note: There are nonspecific perinuclear halos including 23. B
inflammatory halos and glycogen halos. Irregular nuclear Note: CIN lesions typically begin at the squamo-
membrane and hyperchromasia are the most important columnar junction because the HPV virus preferentially
diagnostic features for LSIL even in the absence of nuclear infects the cells of this transformation zone.
enlargement. Prominent nucleoli are a feature of reactive
squamous cells or invasive squamous cell carcinoma. 24. B
Note: The currently used HPV vaccine (Gardasil) is
15. C produced by the recombinant DNA procedure and it does
Note: HPV 16 accounts for about 50-60% of all cervical not contain live virus particles. It is given in form of
squamous cell carcinomas worldwide. HPV 18 accounts three injections over a six-month period. Women are not
for an additional 10-15% of all cases. protected if they have been infected with that HPV
type(s) prior to vaccination. The vaccine is approved for
16. A use in females 9-26 years of age. It contains epitopes of
Note: Prominent macronucleoli and tumor diathesis are most common cancerogenic HPV subtypes 6,11,16,18.
the main cytologic features that help in distinguish SCC
from HSIL. Tumor diathesis refers to the granular debris 25. E
in the background from lysed blood and cell fragments. Note: Psammoma bodies are very rarely found in Pap
It is a feature of invasive carcinoma in Pap smears. smears. Their presence should prompt a search for
ovarian/tubal neoplasms.
17. D
Note: Prominent macronucleolus is a cytologic feature 26. D
of repair, invasive squamous cell carcinoma and Note: The recommended initial workup for women with
adenocarcinoma. a Pap smear diagnosis of atypical endometrial cells is
endometrial and endocervical sampling. If no endo-
18. B metrial pathology is found, colposcopy is recommended.
Note: In AIS, hyperchromatic columnar cells arrange in
rosettes or strips, or splaying out around the edges of 27. C
the cell groups and sheets, so-called “feathering”. Note: HCGs include the following differential
diagnoses: HSIL (carcinoma in situ, CIS), AIS, atrophy
19. D and benign endometrial cells. Examination of the cell
Note: Elongated and dark nuclei are typical of AGUS group arrangement, cells around the edge of the group,
or AIS. Flat sheet arrangement, prominent nucleoli and nuclear features, and background cells will help making
marked variation in size are features of reactive/ a correct diagnosis.
22 Cytopathology Review

28. C useful clues for the diagnosis of chronic follicular


Note: Pancytokeratin is positive in both adenocarci- cervicitis.
nomas. Endometrial adenocarcinomas are usually ER+,
vimentin+, and beta-catenin+. 35. D
Note: The term clue cell is used for squamous cells
29. A covered by adherent small coccobacilli, most frequently
Note: The cells are arranged in flat sheets, they have Gardnerella vaginalis. Lactobacilli, which form the
moderate amounts of dense cytoplasm, and show mild normal vaginal flora are absent. The term cornflake
variation in nuclear size with slightly irregular nuclear refers to the refractile brown artifact seen on superficial
contours. They differ from superficial and intermediate squamous cells due to air-bubbles trapped under the
squamous cells; they have less cytoplasm than these cells cover slip. Koilocytes are cells showing cytopathic
and it is less dense. Endocervical cells are usually changes due to human papilloma virus (HPV)- cells have
arranged in a honeycomb or picket-fence pattern and a large cytoplasmic cavity with a condensed sharply
have mucinous cytoplasm. Endometrial cells usually defined inner edge. Nuclei are often enlarged showing
form small three-dimensional groups. hyperchromasia and irregular nuclear contour.
Binucleation is common. Navicular cells are squamous
30. D cells with abundant pale-yellow colored cytoplasm
Note: This is a typical atrophic smear showing a large representing glycogen. They are more abundant during
group of squamous cells arranged in cohesive flowing pregnancy. Blue blobs are dark, round, amorphous
sheets. Nuclei are bland with smooth nuclear membrane. masses seen in atrophic smears. They represent either
Prominent nucleoli (as seen in repair) are not present. condensed mucus or degenerated bare nuclei.
36. E
31. B
Note: This smear contains Trichomonas vaginalis, an
Note: Parakeratosis is characterized by keratinized cells
ovoid, pear-shaped protozoon (arrow) measuring 1-2 mm
with elongated pyknotic nuclei and dense orangeophilic
in longest diameter. It has a nucleus, scant cytoplasm,
cytoplasm. This benign reactive change is usually and also flagellae which are, however, not seen readily
associated with chronic irritation. There is no nuclear in Pap smears. The squamous cells show reactive
atypia. Hyperkeratosis refers to anucleated mature, changes such as slight nuclear enlargement and an
keratinized squamous cells which may appear isolated indistinct perinuclear halo.
or are arranged into plaques.
37. B
32. A Note: This smear shows pseudohyphae and yeast forms
Note: These tightly packed cells are columnar with of Candida albicans. Tangles of pseudohyphae admixed
abundant cytoplasm and cilia on their apical surface. The with yeast forms are colloquially described as “spaghetti
appearance of cilia is a feature of tubal metaplasia, a and meatballs”. Sometimes the squamous cells appear
benign change of the endocervical glandular cells. in linear arrays as if skewered by the pseudohyphae
(popularly described as “shish-kebabs”).
33. D
Note: Spontaneously exfoliated endometrial cells may 38. A
be seen in Pap smears taken during the first 12 days of Note: Multinucleated cells with ground-glass chromatin,
menstrual cycle. They form small three-dimensional like the ones shown here (arrow), are diagnostic of
groups composed of cells with scant cytoplasm and herpes simplex virus infection.
nuclei that are the same size as that of intermediate squa-
mous cells. Their nuclear contours are often irregular. 39. D
Note: The cells form a flat sheet and at the periphery
34. B are columnar in shape with abundant mucinous
Note: This group of cells is composed of aggregates of cytoplasm, which is characteristic of endocervical cells.
lymphocytes. Small mature lymphocytes predominate, Squamous cells (normal, ASCUS and LSIL) do not form
but there are also a few large lymphocytes and tangible- such glandular structures. Endometrial cells are smaller
body macrophages (arrow). The latter cells are very and form three-dimensional groups.
Cervical and Vaginal Cytology 23

40. B which is a characteristic feature of atypical endometrial


Note: The cells shown in this Pap smear have mildly cells.
enlarged nuclei, irregular nuclear membrane and mild
hyperchromasia, which are all features of ASCUS. 47. C
According to the current recommended management Note: The cell in the center is densely keratinized with
guidelines developed by American Society for Colpo- orangeophilic cytoplasm, which is a clue to the diagnosis
scopy and Cervical Pathology (ASCCP) this woman of squamous cell carcinoma. The cells are large with
should undergo HPV testing. If HPV test is positive, the irregular hyperchromatic nuclei, one with mitosis.
patient is referred to colposcopy; if HPV test is negative, The detached and fragmented orangeophilic cytoplasm
the patient goes back to routine yearly screening. (arrow) and granular debris (arrowhead) in this
image are also features of invasive squamous cell
41. D carcinoma.
Note: The cells shown here are koilocytes and represent
LSIL. Colposcopic examination is the proper follow-up 48. A
and should be included in the management of all patients Note: Cytomorphologic changes associated with IUD
with LSIL. effect include vacuolated glandular cells (as shown here)
and small dark cell with scant cytoplasm. The vacuolated
42. B glandular cells of IUD effect may be indistinguishable
Note: The cells have hyperchromatic nuclei with from cells of adenocarcinoma especially those in
irregular nuclear membrane and high nuclear to endometrial adenocarcinoma. Diagnosing endometrial
cytoplasmic ratio (N/C ratio). High N/C ratio is one of adenocarcinoma should be prudent in a woman with IUD
the key features of HSIL. use; sometimes a repeat Pap after removal of the IUD
may be recommended.
43. E
Note: The cells form a flat sheet with streaming (so 49. D
called “school of fish”). The nuclei contain prominent Note: The image shows a group of koilocytes with
nucleoli, a feature of reactive squamous cells. enlarged hyperchromatic nuclei, peri-nuclear halos with
condensed rim and irregular nuclear membrane. These
44. B. cells are in contrast to the adjacent normal intermediate
Note: This smear shows koilocytes, which are squamous cells which have smaller nuclei and yellow
characteristic of low-grade intraepithelial lesions (LSIL). glycogen in the cytoplasm.
The cells have moderately enlarged nuclei and
characteristic peri-nuclear halo with a condensed rim. 50. D
Note: Cells are arranged in hyperchromatic and crowded
45. E groups. Cells are columnar in shape and contain dark
Note: The cells are columnar in shape, have elongated elongated nuclei with inconspicuous nucleoli. There is
and hyperchromatic nuclei, characteristic of atypical “feathering” at the periphery of the group. These are
endocervical cells. These features are not characteristic characteristic features for AIS.
of a squamous intraepithelial lesion (LSIL or HSIL).
Endocervical adenocarcinoma cells have abnormal 51. E
glandular arrangement (microacini, rosettes) and Note: Tumor cells form a three-dimensional group and
prominent nucleoli. Non-keratinizing squamous cell have enlarged nuclei (compare to the adjacent nuclei of
carcinoma cells have scant and dense cytoplasm, coarse normal intermediate squamous cells). Tumor cells have
chromatin and prominent nucleoli. abundant vacuolated cytoplasm and intracytoplasmic
neutrophils—all of which are features of endometrial
46. C adenocarcinoma.
Note: The cells form a small cluster and they have
irregular nuclei and vacuolated cytoplasm. The nuclei 52. C
are larger than that of adjacent intermediate squamous Note: The arrow points to elongated rods of Actinomyces
cells. The cytoplasmic vacuoles contain neutrophils sp. which are commonly associated with IUD use.
24 Cytopathology Review

53. A 61. E
Note: The perinuclear halos shown here are small and Note: In this figure the arrow shows keratinizing
represent inflammatory halos associated with infection elongated squamous cells (tadpole cells) associated with
such as Candida albicans or Trichomonas vaginalis a background of granular debris (tumor diathesis),
infection. characteristic of keratinizing SCC.

54. C 62. B
Note: The smear contains metaplastic squamous cells Note: The cells show mildly enlarged nuclei, mild
in the center. While some cells have round nuclei with hyperchromasia, slightly irregular nuclear membrane
smooth nuclear membrane representing normal and abundant cytoplasm (normal nuclear/cytoplasmic
metaplastic squamous cells, others have mildly enlarged ratio), features qualitatively insufficient for a diagnosis
nuclei with nuclear grooves and scant cytoplasm, of a SIL lesion; hence the diagnosis of ASCUS. The
features concerning for HSIL. However, the changes are diagnosis ASC-H is made when some of the cells have
not sufficient for a diagnosis of HSIL and fit better in a high nuclear/cytoplasmic ratio concerning for HSIL.
the category of atypical squamous cells cannot exclude
HSIL (ASC-H) 63. C
Note: The cells in this hyperchromatic crowded group
55. A
(HCG) have high nuclear/cytoplasmic ratio and
Note: The smear shows a three-dimensional group with
hyperchromatic nuclei. This hyperchromatic syncytial
a typical “double contour” appearance of benign
group is characteristic of squamous cell carcinoma in situ
exfoliated endometrial cells – an outer layer of glandular
(CIS, HSIL). LSIL cells have moderate amount of
cells surrounding an inner layer of condensed stromal
cytoplasm. Parabasal squamous cells have a more flat
cells.
sheet arrangement with less nuclei crowding and less
56. D nuclei hyperchromasia. Benign endometrial cells usually
Note: The malignant tumor cells are columnar in shape form smaller three-dimensional groups with cells smaller
and form a glandular-like structure. In light of the and less nuclear hyperchromasia. Endometrial
clinical history, the findings are most consistent with adenocarcinoma cells have vacuolated cytoplasm and
metastatic colonic adenocarcinoma. prominent nucleoli.

57. E 64. B
Note: Cytoplasmic melanin pigment is an important Note: Compare to the hyperchromatic crowded group
diagnostic clue in this case. (HCG) shown in the previous case, this group has a more
flat sheet arrangement with less crowding of nuclei.
58. B Cells have moderate amount of cytoplasm and less
Note: These cells with glycogenated perinuclear halo are nuclei hyperchromasia. These features support the
called navicular cells. Koilocytes are HPV infected cells; interpretation of parabasal squamous cells in this smear.
clue cells are squamous cells covered by coccobacilli;
cornflake cells represent an air-drying artifact; tadpole 65. A
cells are elongated keratinizing cells commonly seen in Note: This is another example of hyperchromatic
squamous cell carcinoma. crowded group (HCG). Compare this image to the
previous two images! This hyperchromatic crowded
59. A group is a three-dimensional group composed of cells
Note: The cells show the characteristic changes induced with small nuclei (same size as the adjacent intermediate
by radiation, including cytomegaly (large nucleus and squamous cells). Some nuclei are bean-shaped, which
abundant cytoplasm), nuclear and cytoplasmic vacuoles is characteristic of endometrial cells. Marked hyper-
and polychromasia, prominent nucleoli. chromasia as in HSIL is not present.
60. D 66. B
Note: The cells show typical features of HSIL: high Note: The cells in this three-dimensional group have
N/C ratio, hyperchromasia and coarse granular chromatin. enlarged nuclei with vesicular chromatin. The cytoplasm
Cervical and Vaginal Cytology 25

is vacuolated and contains intracytoplasmic neutrophils. 69. D


All findings are characteristic of endometrial adenocar- Note: These cells have a flat sheet arrangement and show
cinoma. features characteristic of radiation changes: cytomegaly
(large cells with big nuclei and abundant cytoplasm) and
67. B cytoplasmic vacuoles.
Note: This figure shows a group of adenocarcinoma
cells. Compare to the endometrial adenocarcinoma in 70. C
the previous image, cells in this group have prominent Note: This smear shows clusters of small blue cells with
macronucleoli and granular cytoplasm—features more hyperchromatic nuclei, scant cytoplasm, and nuclear
characteristic of endocervical adenocarcinoma. molding, characteristic of small cell carcinoma. The
tumor morphology resembles that of the lung small cell
68. B carcinoma. Small cell carcinoma of the cervix is
Note: These cells have an organized arrangement with commonly associated with HPV type 18, and it is a
elongated and hyperchromatic nuclei without nucleoli, highly aggressive tumor with early development of
features characteristic of atypical endocervical cells. distant metastases.
26 Cytopathology Review

2
Respiratory Tract
Fang Fan

QUESTIONS
1. The adequacy of a sputum sample is established by finding which of the following cells in the specimen?
A. Ciliated columnar cells
B. Squamous cells
C. Pulmonary macrophages
D. Goblet cells
E. Neutrophils

2. Bronchoalveolar lavage (BAL) is most often used for the diagnosis of which condition?
A. Interstitial pneumonia
B. Sarcoidosis
C. Carcinoid
D. Opportunistic infections
E. Lymphoma

3. Which of the following cytologic features is important in separating carcinoid from atypical carcinoid
tumors in a cytology specimen?
A. Loosely cohesive groups of tumor cells
B. Uniform nuclei with finely dispersed “salt and pepper” chromatin
C. Prominent nucleoli
D. Absence of necrosis
E. Absence of nuclear molding

4. A CT-guided fine needle aspiration of a large lung mass shows groups of cells with scant cytoplasm
and frequent mitoses. Differential diagnoses include poorly differentiated squamous cell carcinoma
and small cell carcinoma. Which of the following immunohistochemical stains favors a diagnosis of
squamous cell carcinoma?
A. CK 7+
B. CK20-
C. TTF-1 +
D. p63+
E. CD56+
Respiratory Tract 27

5. A 42-year-old woman presented with dyspnea, non-productive cough, and expectoration of gelatinous
material. Chest CT showed diffuse lung infiltrates. Bronchoalveolar lavage demonstrated numerous
acellular blobs of amorphous material that stained red with PAS (periodic acid Schiff reaction).
What is the correct diagnosis?
A. Pulmonary amyloidosis
B. Pulmonary alveolar mucinosis
C. Pulmonary edema
D. Pulmonary alveolar proteinosis
E. Pneumocystis jiroveci pneumonia

6. A 32-year-old man presents with shortness of breath and multiple lung masses. Clinical history reveals
that he has had left orchiectomy for a tumor a few years ago. CT-guided fine needle aspiration of the
lung mass is performed. Which of the following immunohistochemical stains is most likely to confirm
the diagnosis of a metastatic germ cell tumor?
A. Pancytokeratin
B. OCT-4
C. TTF-1
D. CD45
E. p63

7. A 39-year-old man presented with fever, cough and hematuria. Chest X-ray showed a right lobe lung
mass. Fine-needle aspiration yielded granular necrotic debris, giant cells, granulomas and neutrophils.
All of the following are appropriate follow-up tests, EXCEPT:
A. Send specimen for fungal culture
B. Send specimen for acid-fast bacilli culture
C. Perform Congo red stain on the smear
D. Test serum c-ANCA
E. Test serum p-ANCA

8. A diagnosis of lung adenocarcinoma was established by FNA of a lung mass in a 57-year-old


woman. Therapy with tyrosine kinase inhibitors is contemplated. The specimen should be submitted
for molecular biologic analysis of mutations of which gene?
A. c-KIT
B. p53
C. EGFR
D. PAX-8
E. PDGFR

9. Which of the following gene mutations is a predictor of failure to EGFR tyrosine kinase inhibitor (TKI)
therapy?
A. PDGFR-alpha
B. c-KIT
C. KRAS
D. p53
E. p16
28 Cytopathology Review

10. Re-arrangements of which of the following genes may occur in some non-small cell carcinomas of the
lung?
A. c-KIT
B. BRAF
C. ALK
D. PAX-8
E. PDGFR

11. Clear cell tumor (“sugar tumor”) of the lung is extremely rare. Differential diagnoses in cytology include
various tumors with clear cytoplasm. Which of the following positive stains supports the diagnosis of
clear cell tumor of the lung?
A. p63
B. TTF-1
C. CD10
D. HMB-45
E. CK7

12. The findings of creola bodies, Charcot-Leyden crystals, eosinophils, and occasional Curschmann spirals
in a bronchial brushing specimen are suggestive of which condition?
A. Asthma
B. Tuberculosis
C. Wegener’s granulomatosis
D. Asbestos exposure
E. Sarcoidosis

13. Ciliocytophthoria (CCP) represents detachment of the terminal bar and cilia from a bronchial cell or
decapitation of the ciliated columnar cells. CCP is most commonly associated with which of the following
conditions?
A. Asbestos exposure
B. Adenovirus infection
C. Bacterial pneumonia
D. Histoplasmosis
E. Radiation effect

14. A 55-year-old man presented for follow-up CT of his left upper lobe lung nodule which was identified
1 year ago on chest imaging. The nodule was a solitary discrete mass and appeared to have increased in
size now. A CT-guided biopsy of the nodule was performed and showed benign glandular cells,
adipocytes, cartilage, bland spindle cells and fibromyxoid matrix. What is the most likely diagnosis?
A. Non-diagnostic specimen
B. Sarcoidosis
C. Pulmonary hamartoma
D. Inflammatory myofibroblastic tumor
E. Chondrosarcoma
Respiratory Tract 29

15. A cytology smear from a lung mass displays large polygonal tumor cells with vesicular chromatin,
irregular nuclear membrane and prominent nucleoli. Immunohistochemically the tumor cells were
positive for pancytokeratin and CD34, and were negative for TTF-1. What is the most appropriate
diagnosis?
A. Large cell carcinoma
B. Anaplastic large cell lymphoma
C. Epithelioid angiosarcoma
D. Melanoma
E. Carcinosarcoma

16. A 42-year-old man presented with fever, cough, dyspnea and chest pain. Imaging study revealed
an opaque area in the left upper lobe. Fine needle aspiration showed large atypical lymphocytes admixed
with small mature lymphocytes. Ancillary tests demonstrated that the large atypical lymphocytes were
B lymphocytes and positive for Epstein-Barr virus (EBV). The background small mature lymphocytes
are T lymphocytes. What is the most likely diagnosis?
A. Lymphocytic interstitial pneumonia
B. Lymphomatoid granulomatosis
C. Hodgkin lymphoma
D. Extranodal marginal zone B-cell lymphoma (MALT lymphoma)
E. Diffuse large B cell lymphoma

17. Which of the following cell types must be present for a bronchioalveolar lavage specimen to be considered
adequate?
A. Squamous cells
B. Ciliated columnar cells
C. Goblet cells
D. Macrophages
E. Pneumocytes type II

18. All of the following are cytomorphologic features of small cell carcinoma of the lung, EXCEPT:
A. Scant cytoplasm
B. Prominent nucleoli
C. Nuclear molding
D. Mitoses
E. Necrosis
30 Cytopathology Review

19. This group of cells is obtained from a bronchial


washing specimen. What is the correct
diagnosis?
A. Adenocarcinoma
B. Carcinoid tumor
C. Small cell carcinoma
D. Reactive bronchial cells
E. Pulmonary macrophages

20. A bronchial washing specimen shows a group of


tightly packed small cells with scant cytoplasm as
shown here. What is the correct interpretation?
A. Carcinoid tumor
B. Small cell carcinoma
C. Lymphoma
D. Reserve cell hyperplasia
E. Type II pneumocyte hyperplasia

21. What is the structure (arrows) identified in this


sputum specimen?
A. Fungus
B. Parasite
C. Inspissated mucus
D. Foreign material
E. Ferruginous body
Respiratory Tract 31

22. These structures in this bronchial washing specimen


(arrows) are commonly described as which of the
following?
A. Ferruginous body
B. Corpora amylacea
C. Psammoma body
D. Charcot-Leyden crystal
E. Vegetable matter

23. A 45-year-old man presents with fever, dyspnea and


cough. Chest image shows diffuse reticular interstitial
infiltrates. A smear from the bronchioloalveolar
lavage specimen is shown here. What is the correct
interpretation?
A. Herpes virus infection
B. Cytomegalovirus (CMV) infection
C. Measles virus infection
D. Adenovirus
E. Respiratory syncytial virus (RSV)

24. A 39-year-old man who is HIV-positive presents with


dry cough, fever and dyspnea. Chest image shows
bilateral diffuse infiltrates. A stat bronchioloalveolar
lavage is performed and shown in this Diff-Quik
stained smear. What is the correct diagnosis?
A. Cryptococcus
B. Histoplasmosis
C. Candidiasis
D. Pneumocystis pneumonia
E. Alveolar proteinosis
32 Cytopathology Review

25. A 42-year-old African American woman presents


with multiple lung nodules. Fine needle aspiration of
the nodules is shown here. What is the correct
interpretation?
A. Pulmonary hamartoma
B. Inflammatory pseudotumor
C. Granulomatous process
D. Granular cell tumor
E. Organizing pneumonia

26. A discrete solitary mass was discovered in the


periphery of the right upper lobe of the lung in this
65-year-old woman. What is the most appropriate
diagnosis to be made from this cytologic smear
prepared from a FNA specimen?
A. Pulmonary hamartoma
B. Carcinoid tumor
C. Adenocarcinoma
D. Pulmonary granulomas
E. Pulmonary amyloidosis

27. This fine needle aspiration was obtained from a hilar


mass of the lung. What is the correct diagnosis?
A. Squamous cell carcinoma
B. Adenocarcinoma
C. Small cell carcinoma
D. Large cell carcinoma
E. Lymphoma
Respiratory Tract 33

28. A 62-year-old woman has an endobronchial mass.


Bronchial brushing is performed and the smear is
shown here. What is the correct diagnosis?
A. Benign reactive bronchial cells
B. Benign reactive lymphocytes
C. Carcinoid tumor
D. Atypical carcinoid tumor
E. Adenocarcinoma

29. A 56-year-old woman presents with a right upper lobe


lung infiltrate. A CT-guided fine needle aspiration is
performed and shown here. What does this cell group
represent?
A. Benign reactive type II pneumocytes
B. Hyperplastic goblet cells
C. Pulmonary macrophages
D. Mucinous adenocarcinoma
E. Squamous cell carcinoma

30. A 57-year-old man was discovered to have a large


hilar mass on CT. An endobronchial ultrasound
(EBUS)-guided fine needle aspiration was performed
and shown here. What is the correct diagnosis?
A. Squamous cell carcinoma
B. Small cell carcinoma
C. Adenocarcinoma
D. Carcinoid tumor
E. Large cell neuroendocrine carcinoma
34 Cytopathology Review

31. A 56-year-old man was found to have a lung mass in


the follow up imaging study after right nephrectomy.
Fine needle aspiration was performed and the smear
prepared from this procedure is shown here. What
is the most likely diagnosis?
A. Granuloma
B. Hamartoma
C. Squamous cell carcinoma
D. Metastatic renal cell carcinoma
E. Angiosarcoma

32. These cells (arrow) are seen on a bronchoalveolar


lavage specimen from a 45-year-old woman with
marked acute respiratory distress and diffuse lung
infiltrates. What is the most likely interpretation?
A. Reactive type II pneumocytes
B. Bronchial reserve cells
C. Pulmonary macrophages
D. Lung adenocarcinoma
E. Metastatic breast adenocarcinoma

33. This is a Grocott methenamine silver (GMS) stained


smear of a pulmonary bronchoalveolar lavage
specimen. What is the fungal pathogen indicated by
the arrow?
A. Candida albicans
B. Histoplasma capsulatum
C. Blastomyces dermatitidis
D. Aspergillus fumigatus
E. Pneumocystis jiroveci
Respiratory Tract 35

34. What is this structure seen in a smear obtained by


FNA of lungs?
A. Foreign material
B. Aspergillus
C. Mucor
D. Candida
E. Rhizopus

35. A transbronchial fine needle aspiration of a large


bronchial mass was performed. The smear is shown
here. Immunohistochemical stains on the cellblock
sections demonstrated positivity for chromogranin
and CD56. What is the correct diagnosis?
A. Carcinoid tumor
B. Atypical carcinoid tumor
C. Small cell carcinoma
D. Large cell neuroendocrine carcinoma
E. Lymphoma

36. A smear prepared from a bronchial brushing


specimen is shown here. Tumor cells were positive
for keratin CK5/6. What is the correct diagnosis?
A. Adenocarcinoma
B. Squamous cell carcinoma
C. Small cell carcinoma
D. Atypical carcinoid
E. Pulmonary blastoma
36 Cytopathology Review

37. This image was taken from a lung fine needle


aspiration smear of a lung nodule in a 34-year-old
female. She had a history of thyroidectomy three
years ago. What is the most likely diagnosis?
A. Reactive bronchial cells
B. Reactive mesothelial cells
C. Hamartoma
D. Adenocarcinoma of the lung
E. Metastatic papillary thyroid carcinoma

38. This smear was prepared from a fine needle


aspiration biopsy of a lung lesion in a 60-year-old
man. What is the correct diagnosis?
A. Non-small cell carcinoma
B. Small cell carcinoma
C. Lymphoma
D. Melanoma
E. Mesothelioma

39. This positive Oil-Red O stain (arrow) is highly


suggestive of which of the following diagnosis?
A. Fungal infection
B. Viral infection
C. Lipid pneumonia
D. Mucinous adenocarcinoma
E. Melanoma
Respiratory Tract 37

40. What is the correct diagnosis of this bronchial


washing specimen?
A. Squamous cell carcinoma
B. Adenocarcinoma
C. Large cell carcinoma
D. Lymphoma
E. Melanoma

41. This lung fine needle aspiration specimen was


obtained from a patient who had a history of a
salivary gland tumor. What is the most likely
diagnosis?
A. Lung amyloidosis
B. Lung adenocarcinoma
C. Metastatic adenoid cystic carcinoma
D. Metastatic mucoepidermoid carcinoma
E. Metastatic medullary carcinoma

42. This Grocott methenamine silver stain is positive for


which of the following organisms?
A. Candida albicans
B. Histoplasma capsulatum
C. Cryptococcus neoformans
D. Aspergillus fumigatus
E. Pneumocystis jiroveci
38 Cytopathology Review

43. What is this structure (arrow) in a bronchioalveolar


lavage specimen?
A. Vegetable matter
B. Ferruginous body
C. Aspergillus
D. Strongyloides
E. Alternaria

44. These cells are seen in a bronchial brushing specimen.


What is the correct interpretation?
A. Normal bronchial epithelial cells
B. Reserve cell hyperplasia
C. Lymphoid aggregate
D. Carcinoid tumor
E. Small cell carcinoma

45. This smear represents a fine needle aspiration


specimen from a lung mass. What is the diagnosis?
A. Reserve cell hyperplasia
B. Carcinoid tumor
C. Atypical carcinoid
D. Small cell carcinoma
E. Adenocarcinoma
Respiratory Tract 39

46. A 64-year-old man has multiple lung masses most


consistent with metastases from the gastrointestinal
tract. Fine needle aspiration is performed and shown
here. Which of the following positive immuno-
histochemical stains will confirm the diagnosis?
A. S100
B. CD10
C. CDX2
D. TTF-1
E. Chromogranin

47. This smear represents sampling from a peripheral


lung lesion. What is the correct interpretation?
A. Non-diagnostic specimen
B. Necrotizing granuloma
C. Sarcoma
D. Small cell carcinoma
E. Squamous cell carcinoma

48. What is the diagnosis of this cell group from a


bronchial washing specimen?
A. Reactive bronchial cells
B. Reactive type II pneumocytes
C. Granuloma
D. Small cell carcinoma
E. Adenocarcinoma
40 Cytopathology Review

49. This Grocott stain is positive for which of the


following organisms?
A. Cryptococcus neoformans
B. Histoplasma capsulatum
C. Candida albicans
D. Pneumocystis jiroveci
E. Blastomyces dermatitidis

50. An 81-year-old woman presented with dyspnea and


chest discomfort. She had a history of breast cancer
status post radiation and lumpectomy. CT scan shows
bilateral pulmonary infiltrates. This picture shows the
bronchioalveolar lavage specimen. What is the
correct interpretation?
A. Reactive pneumocytes
B. CMV infection
C. Herpes infection
D. Hodgkin lymphoma
E. Lung adenocarcinoma
Respiratory Tract 41

ANSWERS

1. C the malignant stem cells of nonseminomatous germ cell


Note: The adequacy of a sputum sample is established tumors (NSGCT).
by finding numerous pulmonary macrophages typical of
deep cough specimens from of the lower respiratory 7. C
tract. Ciliated columnar cells and goblet cells may be Note: The cytomorphologic findings are that of a
from sinonasal passages or upper respiratory tract necrotizing granulomatous process. Differential
linings; squamous cells and neutrophils may be nothing diagnoses include infectious granulomas caused by fungi
more than oral contaminants. or M. tuberculosis, and Wegener granulomatosis. All the
tests are appropriate except the Congo red stain which
2. D is used to demonstrate amyloid deposition.
Note: BAL is particularly useful for the diagnosis of
8. C
opportunistic infections in immunocompromised
Note: Somatic mutations in the exons 18 to 21 of the
patients, especially Pneumocystis jiroveci in HIV
tyrosine kinase domain of EGFR occur in approximately
patients. In detecting malignancy, BAL is more sensitive
10% of Western and up to 50% of Asian patients. These
in detecting diffuse tumors like adenocarcinoma in situ
mutations are associated with sensitivity to treatment
(previously called bronchioloalveolar carcinoma).
with EGFR tyrosine kinase inhibitors (TKIs). Lung
tumors associated with EGFR-mutated lung cancer
3. D
typically occur in women who did not smoke and are
Note: Carcinoid tumor has many cytologic and architec-
histologically classified as adenocarcinomas. These
tural features in common with atypical carcinoids some
tumors are more common in women of East Asian
of which are listed here as possible answers. However,
ethnicity than in those belonging to other ethnic groups.
atypical carcinoids contain foci of necrosis which are
absent in carcinoid tumors. Atypical carcinoids also 9. C
contain more mitotic figure than carcinoids. Note: The mutations of KRAS and other genes (BRAF,
HER2, etc.) encoding proteins involved in the EGFR
4. D signaling cascade exist as mutually exclusive mutations.
Note: Pulmonary small cell carcinomas are usually TTF- KRAS mutations occur more often in adenocarcinomas
1+ and CD56+. Lung adenocarcinomas are CK7+, of smokers and are adverse prognostic factor. They are
CK20-, TTF-1+. Squamous cell carcinomas are p63+ predictors of resistance to EGFR TKI therapy. Mucinous
and CK5/6+. differentiation of adenocarcinomas appears to strongly
correlate with KRAS mutations.
5. D
Note: The finding of PAS-positive extracellular 10. C
amorphous aggregates in a BAL specimen is charac- Note: ALK (anaplastic lymphoma kinase) rearrange-
teristic of pulmonary alveolar proteinosis. This disease ments (not mutation) resulting in EML4-ALK fusion
of unknown etiology is characterized by accumulation gene have been identified in some patients with non-
of lipid-rich, surfactant-like material within alveoli, from small cell carcinoma of the lung. The presence of EML4-
which it cannot be removed due to the impaired function ALK seems to be mutually exclusive to that of EGFR
of alveolar macrophages. and KRAS mutations. Various methods can be used for
detection of EML4-ALK rearrangements including
6. B FISH, RT-PCR and immunohistochemistry.
Note: The history and the cytology image are highly
suggestive of a metastatic germ cell tumor in this young 11. D
man. Therefore a positive stain for OCT-4 would Note: Clear cell tumor of the lung is believed to arise
confirm the diagnosis of germ cell tumor. This immuno- from perivascular epithelioid cells and therefore stains
histochemical stain is positive in nuclei of seminoma positive for HMB-45, a feature it shares with angiomyo-
cells as well as embryonal carcinoma cells, which are lipoma and PEComa.
42 Cytopathology Review

12. A Compare to small cell carcinoma, there is no mitosis or


Note: Creola bodies (reactive bronchial cells), Charcot- necrosis in reserve cell hyperplasia, and the nuclei
Leyden crystals (by-product of eosinophil degranula- appear bland without hyperchromasia.
tion), eosinophils, and occasional Curschmann’s spirals
(inspissated mucus) may be found in patients with 21. C
asthma. Note: These coils are inspissated mucus—so called
“Curschmann spiral”. It is a non-specific finding in the
13. B lung cytology specimen.
Note: The curious morphologic decapitation of the
22. D
ciliated columnar cells, called ciliocytophthoria, can be
Note: These rhomboid-shaped crystals are formed from
prominent in adenovirus infection.
the degranulated products of eosinophils and are
commonly seen in patients with asthma.
14. C
Note: The described cytomorphologic features are 23. B
characteristic of pulmonary hamartoma. Note: The cells have large intranuclear inclusions (arrow),
characteristic of CMV infection. Herpes virus is charac-
15. C terized by multinucleation with nuclear molding and
Note: Epithelioid angiosarcoma (EAS) may occur as a ground-glass appearing intranuclear inclusions. Measles
primary lung tumor resembling large cell carcinoma of virus and RSV are demonstrated by giant multinucleated
the lung. The diagnosis of EAS is established by cells with intranuclear and intracytoplasmic inclusions.
positivity for vascular markers such as CD34. About Adenovirus presents as large intranuclear inclusion that
30% of EAS are positive for keratins. fills the entire nucleus and has a smudged appearance.
16. B 24. D
Note: Lymphomatoid granulomatosis, an EBV-associated Note: The image (arrow) shows foamy proteinaceous
T cell rich B cell lymphoproliferative disorder, has the sphere composed of cup-shaped cysts and intracystic
features described in the stem of this question. bodies. These features of Pneumocystis Jiroveci are
usually described as “crushed ping-pong balls” or “soap
17. D and bubbles”.
Note: An adequate bronchioalveolar lavage specimen (like
the sputum specimen- see question #1!) should contain a 25. C
large number of macrophages to ensure adequate sampling Note: The smear shows aggregates of epithelioid
of distal alveolar spaces and bronchioles. histiocytes and giant cells, which are features of
granulomas. Pulmonary hamartoma is characterized by
18. B a mixture of fibromyxoid matrix, mature cartilage, bland
Note: Nucleoli are absent or inconspicuous in small cell spindle cells and adipocytes. Inflammatory pseudotumor
carcinoma of the lung. (inflammatory myofibroblastic tumor) is composed of
bland spindle cell proliferation admixed with poly-
19. D morphous inflammatory cells. Granular cell tumor
Note: This group of cells in a three-dimensional sphere presents as small clusters of cells with small uniform
represents a cluster of reactive bronchial, as indicated nuclei and abundant granular cytoplasm. The cells
by cilia at the periphery of the cells. Such a group is resemble macrophages. Organizing pneumonia may
also called Creola body. It is more commonly present as suspicious lesions radiologically. The smear
encountered in bronchial washing specimens in chronic shows loosely arranged fibroblasts admixed with
lung disease, such as asthma. macrophages and chronic inflammatory cells.

20. D 26. A
Note: The cells in this group have high nuclear-to- Note: The smear shows chondromyxoid matrix,
cytoplasm ratio, fine chromatin and small or connective tissue and adipose tissue consistent with a
inconspicuous nucleoli representing reserve cells. pulmonary hamartoma.
Respiratory Tract 43

27. A broad-based budding. Aspergillus is characterized by


Note: The smear shows malignant keratinizing septate hyphae with acute angle branching. Pneumo-
squamous cells with orangeophilic cytoplasm and a cystis is cup-shaped with a central dark zone and no
necrotic background, characteristic of keratinizing budding.
squamous cell carcinoma.
34. B
28. C Note: These organisms demonstrate septate hyphae with
Note: These cells form a small cluster with uniform and acute angle branching, characteristic of Aspergillus.
bland morphology. Necrosis, mitoses or nuclear crush Zygomycosis fungi, such as Mucor or Rhizopus have
artifact are absent. These are characteristic features of non-septate hyphae and show wide-angle branching.
carcinoid tumor. Atypical carcinoid tumors have shown Candida usually forms pseudohyphae and non-
more pleomorphism with increased mitotic activity and branching hyphae.
focal necrosis.
35. D
29. D Note: The cells are large (more than the size of two
Note: This smear shows the typical features of a lymphocytes) with finely granular chromatin, prominent
mucinous adenocarcinoma of the lung. The tumor cells,
nucleoli and marked atypia. The diagnosis of large cell
arranged in a honeycomb-like sheet, have eccentrically
neuroendocrine carcinoma is confirmed by the positive
placed round nuclei and translucent foamy cytoplasm
immunohistochemical stains for neuroendocrine markers
with mucin vacuoles.
(chromogranin and CD56).
30. B
36. B
Note: This image demonstrates characteristic features
Note: The smear shows a non-small cell carcinoma with
of small cell carcinoma, including high nuclear-to-
cytoplasmic ratio, nuclear molding and significant crush poorly differentiated features. Positive CK5/6 stain
artifact. supports the diagnosis of squamous cell carcinoma.

31. D 37. E
Note: The smear shows a group of tumor cells with Note: The tumor cells have optically clear nuclei, nuclear
abundant clear cytoplasm with fine cytoplasmic vacuoles grooves and intranuclear pseudo-inclusions, which are
and surrounded by magenta-colored strandlike basement all typical features of papillary thyroid carcinoma.
membrane material. These features are characteristic of Metastatic papillary thyroid carcinoma may have
renal cell carcinoma. In light of the clinical history of identical cytologic features as primary lung adenocar-
right nephrectomy, the most likely diagnosis is cinomas that were the previously called bronchiolo-
metastatic renal cell carcinoma. alveolar carcinomas. Clinical history and positive
immunohistochemical staining for thyroglobulin help to
32. A reach the correct diagnosis.
Note: This is a loosely cohesive cluster with cell having
large nuclei, irregular nuclear membrane, prominent 38. D
nucleoli and abundant vacuolated cytoplasm, features Note: The tumor cells shown here contain melanin
resemble adenocarcinoma. However, in light of the pigment in their cytoplasm, which make it possible to
clinical history of acute respiratory distress and diffuse diagnose the lung tumor as malignant melanoma.
lung infiltrates without a discrete mass, a diagnosis of Furthermore the cells have a plasmacytoid appearance,
definitive malignancy should be avoided. and their nuclei have prominent nucleoli, which are all
features of melanoma cells.
33. B
Note: The stain shows small intracellular budding yeasts, 39. C
characteristic of Histoplasma. Candida has budding Note: The Oil-Red O stain is positive for lipid material
yeast forming pseudohyphae (“sausage links”). in the cytoplasm of the macrophage, suggestive of lipid
Blastomyces are bigger yeast with thick cell wall and pneumonia.
44 Cytopathology Review

40. B adenocarcinoma. Positive immunohistochemical stain


Note: The tumor cells show marked pleomorphism with for CDX2 in the tumor cells will confirm metastatic
prominent nucleoli, translucent cytoplasm and focal colonic adenocarcinoma.
cytoplasmic vacuoles, typical of an adenocarcinoma.
47. B
41. C Note: The smear shows necrotic debris and a granuloma
Note: This image shows characteristic features of (arrow) consistent with a necrotizing granulomatous
adenoid cystic carcinoma, including magenta-colored process. The specimen should be sent to microbiology
hyaline spherical globules with adherent tumor cells. lab for culture after this on-site assessment.

42. C 48. E
Note: The yeasts have typical narrow based budding of Note: The cells arranged in a cohesive group. Tumor
Cryptococcus neoformans. cells have prominent nucleoli and abundant foamy and
vacuolated cytoplasm. These features are characteristic
43. B of adenocarcinoma.
Note: They are asbestos fibers with a golden-yellow
coating of protein with iron, therefore the name 49. D
ferruginous. Note: The Grocott stain shows abundant organisms with
a “crushed ping-pong balls” appearance. A few
44. A intracystic bodies can be identified. These are recognized
Note: The cells are columnar in shape and have terminal features of Pneumocystis jiroveci.
bars with cilia – an important helpful feature to always
look for in lung cytology. 50. A
Note: The indicated cells (arrow) have enlarged nuclei
45. D and abundant cytoplasm but do not show an increased
Note: The cells show characteristic nuclear molding, N/C ratio. These cells have prominent nucleoli and
coarsely granular chromatin and background necrosis. abundant vacuolated cytoplasm, which are features of
These are features of a small cell carcinoma of the lung. reactive pneumocytes associated with radiation therapy.
Another group of hyperplastic pneumocytes are also
46. C present in this image. Clinical history of radiation
Note: The image shows a malignant epithelial group therapy and lack of a discrete lung mass are very
associated with a large area of “dirty necrosis”. This important clinical data, essential for formulating this
morphology is highly suggestive of metastatic colonic interpretation.
Urine and Bladder Washings 45

3 Urine and Bladder


Washings
Fang Fan

QUESTIONS
1. What is the most common indication for urinary cytology?
A. Screening test for bladder cancer
B. Hematuria
C. Urinary tract infection
D. Surveillance for recurrent bladder cancer
E. Monitoring for renal transplant

2. Which of the following is an advantage of voided urine samples compare to other bladder samples such
as catheterized urine and bladder washing?
A. High cellularity
B. Better cell preservation
C. Selective sampling
D. No instrumentation artifact
E. Less chance of vaginal contamination

3. When a large number of degenerated intestinal epithelial cells are seen in a urine sample, the specimen
is collected from which of the following method/location?
A. Voided urine
B. Catheterized urine
C. Bladder washings
D. Renal pelvis brushings
E. Ileal conduit

4. When evaluating papillary clusters in a urine specimen, what is the imperative information?
A. Knowledge of specimen color/texture
B. Knowledge of specimen collection method
C. Knowledge of a history of urinary tract infection
D. Knowledge of a history of urothelial carcinoma
E. Knowledge of a history of prostatic carcinoma
5. Which of the following statements is true regarding the role of urinary cytology in detecting urothelial
carcinomas?
A. Urine cytology is reliable in distinguishing reactive urothelial cells from low-grade urothelial carcinoma
B. Urine cytology is reliable in distinguishing papillary lesions from flat lesions
C. Urine cytology is reliable in distinguishing in situ from invasive urothelial carcinomas
D. Urine cytology is reliable in diagnosing high-grade urothelial carcinomas
E. Urine cytology is reliable in diagnosing renal cell carcinoma
46 Cytopathology Review

6. All of the following are cytologic features of high-grade urothelial carcinoma, EXCEPT:
A. High nuclear-to-cytoplasmic ratio
B. Coarse cytoplasmic vacuolization
C. Marked nuclear hyperchromasia
D. Irregular nuclear membrane
E. Coarsely granular chromatin

7. Which of the following immunohistochemical staining patterns, obtained with antibodies to cytokeratin
(CK) proteins, is typical of urothelial carcinoma?
A. CK7+ CK20-
B. CK7- CK20+
C. CK7+ CK20+
D. CK7- CK20-
8. In making the distinction between reactive urothelial cell and high-grade urothelial carcinoma, which
of the following cytologic features favors high-grade urothelial carcinoma?
A. Prominent nucleoli
B. Vacuolated cytoplasm
C. Multinucleation
D. Normal nuclear-to-cytoplasmic ratio
E. Nuclear hyperchromasia

9. The UroVysion test is used as an adjunct to cytology in the detection of urothelial carcinoma. Which of
the following statements is true regarding the UroVysion test?
A. It detects specific gene translocation
B. It detects specific gene mutation
C. It detects specific gene amplification
D. It detects microsatellite instability
E. It detects polysomy of certain chromosomes and/or loss of certain chromosome band

10. What is this cell type indicated by an arrow in a


voided urine specimen?
A. Superficial urothelial cells
B. Intermediate urothelial cells
C. Squamous cells
D. Prostatic acinar cells
E. Renal tubular cells
Urine and Bladder Washings 47

11. This voided urine sample was collected from a patient


with renal transplant. What does the cell indicated
by an arrow represent?
A. Cytomegalovirus infection
B. Polyomavirus infection
C. Herpes virus infection
D. Degenerated benign urothelial cells
E. High-grade urothelial carcinoma

12. These cells were seen in a smear from a catheterized


urine from a patient with a clinical history of
urothelial carcinoma. What is the correct diagnosis?
A. Reactive urothelial cells
B. Seminal vesicle cells
C. Polyomavirus infection
D. Recurrent urothelial carcinoma
E. Prostatic adenocarcinoma

13. This bladder washing specimen is collected from a


56-year-old man with hematuria. What is the correct
interpretation?
A. Benign/reactive changes
B. Low-grade urothelial carcinoma
C. High-grade urothelial carcinoma
D. Prostatic adenocarcinoma
E. Metastatic carcinoma
48 Cytopathology Review

14. A 48-year-old man presents for follow-up surveillance


for urothelial carcinoma. Voided urine sample was
obtained and shown here. What do the cells indicated
by arrows represent?
A. Benign basal urothelial cells
B. Degenerated urothelial cells
C. Polyomavirus infection
D. Low-grade urothelial carcinoma
E. High-grade urothelial carcinoma

15. This urine specimen is collected from a patient with


a clinical history of urothelial carcinoma. What is the
most likely interpretation?
A. Many degenerated cells, likely recurrent urothelial
carcinoma
B. Many degenerated cells, likely stone atypia
C. Many degenerated cells, likely fungal infection
D. Many degenerated cells, likely ileal conduit specimen
E. Many degenerated cells, metastatic carcinoma

16. The structure pointed by an arrow is seen in a


degenerated urothelial cell in a voided urine sample.
It represents which of the following?
A. Cytomegalovirus inclusion
B. Michaelis-Gutmann bodies
C. Melamed-Wolinska bodies
D. Trichomonas vaginalis
E. Uric acid crystals
Urine and Bladder Washings 49

17. This cell cluster from a urinary bladder washing


specimen of a 56-year-old woman is diagnostic of
which of the following conditions?
A. Benign urothelial cells
B. Papilloma
C. Papillary urothelial neoplasm of low malignant
potential (PUNLMP)
D. Low-grade papillary urothelial carcinoma
E. High-grade papillary urothelial carcinoma

18. This image was taken from a catheterized urine


specimen from an 82-year-old man. What is the
correct diagnosis?
A. Reactive urothelial cells
B. Renal tubular cells
C. Seminal vesicle cells
D. Low-grade urothelial carcinoma
E. High-grade urothelial carcinoma

19. This bladder washing specimen was collected from a


young man who had a recent travel history to Africa
along the Nile River. What is the most accurate
diagnosis?
A. Urinary calculi
B. Urine crystals
C. Trichomonas vaginalis infection
D. Schistosoma hematobium infection
E. Entamoeba histolytica infection
50 Cytopathology Review

20. What is this structure (arrow) seen in a voided urine


specimen?
A. Triple phosphate crystals
B. Uric acid crystals
C. Calcium oxalate crystals
D. Cholesterol crystals
E. Tyrosine crystals

21. A 67-year-old man who had a history of lung cancer


presented with hematuria. A cystoscopy and bladder
washing was performed and shown here. What is the
correct diagnosis?
A. Lymphoma
B. Metastatic squamous cell carcinoma
C. Metastatic adenocarcinoma
D. Metastatic small cell carcinoma
E. High-grade urothelial carcinoma

22. What does this cell (arrow) in a voided urine sample


represent?
A. High-grade urothelial carcinoma
B. Low-grade urothelial carcinoma
C. Squamous cell carcinoma
D. Cytomegalovirus inclusion
E. Polyomavirus inclusion
Urine and Bladder Washings 51

23. This sample is collected as catheterized urine. What


do these cells indicate?
A. Normal urothelial cells
B. Polyomavirus infection
C. Herpes virus infection
D. Low-grade urothelial carcinoma
E. High-grade urothelial carcinoma

24. This image demonstrates a urinary bladder washing


specimen from a middle-aged man. Which of the
following statements is correct regarding this lesion?
A. It is associated with urinary lithiasis
B. It is associated with malakoplakia
C. It is associated with HPV infection
D. It is associated with Schistosoma hematobium
infection
E. It is associated with trichomonas infection

25. A 78-year-old woman presented for repair of her


urethral diverticulum. A voided urine sample was
submitted for cytology. An image is shown here. What
is the most likely diagnosis?
A. Reactive urothelial cells
B. Urothelial carcinoma
C. Adenocarcinoma
D. Renal cell carcinoma
E. Squamous cell carcinoma
52 Cytopathology Review

26. This image is taken from a ureteral washing


specimen. What is the correct interpretation?
A. Benign urothelial cells
B. Low-grade urothelial carcinoma
C. High-grade urothelial carcinoma
D. Adenocarcinoma
E. Renal cell carcinoma

27. A follow-up bladder washing specimen is collected 6


months after treatment for urothelial carcinoma. A
characteristic image is shown here. What is the
correct interpretation?
A. Herpes virus infection
B. Fungal infection
C. Therapy-related reactive changes
D. Recurrent urothelial carcinoma
E. Sarcoma

28. This voided urine sample was collected from a renal


transplant patient. What is the correct diagnosis?
A. CMV inclusions
B. Herpes virus inclusions
C. Polyomavirus inclusions
D. HPV inclusions
E. Melamed-Wolinska bodies
Urine and Bladder Washings 53

ANSWERS

1. B 7. C
Note: Clinical indications for urinary cytology include
hematuria, follow-up surveillance for recurrent bladder 8. E
cancer and screening test for individuals having high- Note: Nuclear hyperchromasia and irregular nuclear
risk factors for bladder cancer (e.g., occupational membrane are cytologic features of high-grade urothelial
exposure to aniline dyes). Among them, hematuria is the carcinoma. The other features listed (A to D) are
most common indication for urinary cytology. characteristic of reactive urothelial changes.
2. D 9. E
Note: Common cytology specimen types of urinary Note: UroVysion is a FISH test that using probes to
bladder include voided urine, catheterized urine, and detect gains of two or more chromosomes (polysomy)
bladder washings. Compare to the other two types of of chromosome 3, 7, or 17, or complete loss of both 9p21
specimens, voided urine is obtained through a non- signals. UroVysion is approved as a screening test in
invasive procedure and has no instrumentation artifact. patients with hematuria or as a surveillance tool in
However, the disadvantage of voided urine sample is patients with urothelial carcinoma.
low cellularity, poor cell preservation and vaginal
contamination in females. 10. B
3. E Note: Intermediate urothelial cells represent the majority
Note: Ileal conduit is made by anastomosing a segment of urothelial cells in voided urine specimens. These cells
of ileum to the ureters to provide a conduit for urine have round or oval nuclei and moderate amount of
after cystectomy for bladder cancer. Therefore, urine cytoplasm. Superficial urothelial cells, called umbrella
samples for ileal conduits contain a large number of cells (arrowhead), often have multiple nuclei and
degenerated intestinal epithelial cells. abundant cytoplasm. Basal urothelial cells are rare in
voided urine and often form a cohesive flat sheet.
4. B
Note: Papillary or three-dimensional clusters of 11. B
urothelial cells are commonly seen in urine samples Note: The indicated cell has an enlarged nucleus
collected with instrumentation, therefore knowing the completely replaced by a glassy homogeneous viral
method of specimen collection is important. inclusion characteristic of polyomavirus infection.
Cytomegalovirus inclusion is an intranuclear eosino-
5. D
philic inclusion; herpes virus is usually a multinucleated
Note: Urine cytology is neither sensitive nor specific in
intranuclear inclusion with nuclear moldings. Polyoma-
diagnosing low-grade urothelial carcinomas due to lack
virus infected cells may mimic malignant cells due to
of reliable cytologic and architectural criteria. However,
hyperchromasia and high N/C ratio, therefore
urine cytology is highly sensitive (79%) and specific
named “decoy cells”. However, compare to malignant
(>95%) in diagnosing high-grade urothelial carcinomas.
urothelial cells which have irregular nuclear membrane
Unfortunately, urine cytology cannot reliably separate
and coarse chromatin, polyomavirus infected cells have
in situ from invasive urothelial carcinomas, or papillary
smooth nuclear membrane and homogenous chromatin.
from flat lesions. Urine cytology has low sensitivity and
Malignant urothelial cells may form clusters whereas
specificity for diagnosing renal cell carcinoma.
polyomavirus infected cells are always present in
6. B isolated forms.
Note: Coarse cytoplasmic vacuolization is not a feature
for high-grade urothelial carcinoma. Its presence is a 12. A
feature of reactive changes. All others (A, C, D and E) Note: These cells have maintained N/C ratio, prominent
are characteristic morphologic features for high-grade nucleoli and vacuolated cytoplasm, features charac-
urothelial carcinoma in cytology. teristic of reactive urothelial cells.
54 Cytopathology Review

13. A feature of this infestation is its frequent association with


Note: This cytology specimen shows typical benign/ squamous cell carcinoma of the bladder.
reactive bladder cytology: abundant superficial urothe-
lial cells and scattered intermediate urothelial cells. 20. B
Note: Triple phosphate crystals: prisms-like resemble
14. E coffin lids; uric acid crystals: vary in size and shape;
Note: These cells (arrow) have high N/C ratio, marked calcium oxalate crystals: oval dumbbell shaped;
nuclear hyperchromasia (“chunk of charcoal”) and cholesterol crystals: cleft- or rectangle-shaped; tyrosine
irregular nuclear membrane – features characteristic for crystals: slender needle-shaped.
high-grade urothelial carcinoma.
21. D
15. D Note: The malignant cells here are small with high
Note: This image shows a characteristic urine sample nuclear-to-cytoplasmic ratio and nuclear molding
from an ileal conduit: many degenerated intestinal cells characteristic of small cell carcinoma. The diagnosis is
and granular debris. Features of malignancy are absent. confirmed by the patient’s history of small cell car-
cinoma of the lung and positive immunohistochemical
16. C stains for neuroendocrine markers including
Note: This round eosinophilic cytoplasmic inclusion is chromogranin and synaptophysin.
called Melamed-Wolinska body. It is commonly seen
in degenerated urothelial cells, especially in voided urine 22. E
specimens. The pathogenesis of this structure is unclear Note: This cell shows the characteristic ground-glass
and it has no diagnostic significance when seen in appearance of polyomavirus inclusion in the nucleus
bladder specimens. When seen in cells of extra urinary with smooth nuclear membrane. In contrast, high-grade
bladder cytology specimens, their presence may suggest urothelial carcinoma cells have coarse and hyper-
a urothelial origin of the cells. Michaelis-Gutmann chromatic chromatin with irregular nuclear membranes.
bodies are basophilic round lamellated intracellular or Note there is an adjacent degenerated urothelial cell with
extracellular bodies identified in an uncommon hyperchromatic nucleus mimicking high-grade urothelial
condition called malakoplakia which is a histiocytic carcinoma, therefore sometimes referred as “decoy
inflammatory lesion due to bacterial infection. cells”.

17. A 23. E
Note: Cell clusters are very common and non-specific Note: These cells have markedly hyperchromatic nuclei
findings in urine cytology specimens especially those with irregular nuclear membrane (arrow), and associated
obtained with instrumentation. Unless seen with a true with necrotic debris, these features are characteristic of
fibrovascular core, these cell clusters do not have a high-grade urothelial carcinoma.
specific diagnostic significance. Urine cytology also
does not have reliable criteria to diagnose low-grade 24. D
papillary lesion, including papilloma, PUNLMP and Note: This image shows a pure population of malignant
low-grade urothelial carcinoma. squamous cells consistent with squamous cell carcinoma
of the bladder. Pure squamous cell carcinoma of the
18. E bladder is rare and has a strong association with
Note: These cells have high nuclear-to-cytoplasmic ratio, Schistosoma hematobium infection.
irregular nuclear membrane and nuclear hyperchromasia,
features diagnostic of high-grade urothelial carcinoma. 25. C
Note: These cells form a crowded group, have thin and
19. D clear cytoplasm, irregular nuclear membrane and
Note: The structure (arrow) is an oval shaped ovum of prominent nucleoli, most likely represent primary
Schistosoma hematobium with a characteristic terminal adenocarcinoma arising in the urethral diverticulum
spine. The ova are usually deposited in the submucosa (which is the most common malignancy associated with
of the urinary bladder and distal ureter. An important urethral diverticulum).
Urine and Bladder Washings 55

26. A 27. C
Note: Normal urothelial cells from upper tract washings Note: The image shows multinucleated giant cells
typically show more atypia including increased nuclear- admixed with scattered inflammatory cells, characteristic
to-cytoplasmic ratio and mild hyperchromasia as of changes associated with intravesical Bacillus
compare to cells from urinary bladder. Caution should Calmette-Guérin (BCG)-treatment.
be taken when interpreting such specimens. Comparison
between bilateral upper tract specimens is very helpful. 28. A
Diagnostic threshold should be raised; marked morpho- Note: These cells have large eosinophilic intranuclear
logic changes such as irregular nuclear membrane and inclusions (arrow) typical of CMV inclusions. Herpes
coarse granular and hyperchromatic chromatin should and polyomavirus impart infected nuclei a ground-glass
be evident before a positive diagnosis is made. appearance. HPV affects squamous cells and leads to
koilocytic appearance. Melamed-Wolinska bodies are
non-specific eosinophilic cytoplasmic inclusions.
56 Cytopathology Review

4 Effusions and Peritoneal


Washings
Fang Fan

QUESTIONS
1. Which of the following features favors an exudate over a transudate in an effusion specimen?
A. Low fluid protein level
B. Low cell counts
C. Low fluid specific gravity
D. High fluid glucose
E. High lactate dehydrogenase (LDH)

2. Which of the following is not a typical feature for reactive mesothelial cells?
A. Binucleation and multinucleation
B. Irregular nuclear membrane
C. Mitosis
D. Cytoplasmic vacuoles
E. Large clusters

3. All of the following cytologic features favor a diagnosis of malignant effusion, EXCEPT:
A. “Second population” of cells
B. Large cell clusters
C. Smooth community borders of cell clusters
D. “Lacunae” around cell groups in cell block sections
E. Mitosis

4. Which of the following statements is true regarding malignant effusions?


A. Most cases have a known malignancy
B. In most cases this diagnosis does not change the prognosis
C. Most cases respond to systemic chemotherapy
D. Most cases respond to local radiation therapy
E. Most cases do not recur after thoracenteses/paracenteses

5. Which of the following represents the most common cause of so called eosinophilic pleural effusions?
A. Malignancy
B. Tuberculosis
C. Churg-Strauss syndrome
D. Parasitic infections
E. Pneumothorax or hemothorax
Effusions and Peritoneal Washings 57

6. The finding of lymphocytosis in a pleural effusion is most characteristic of which of the following
conditions?
A. Effusion in renal failure
B. Effusion in congestive heart failure
C. Rheumatoid pleuritis
D. Lupus pleuritis
E. Tuberculosis
7. A pleural fluid reveals multinucleated giant cells and large elongated epithelioid histiocytes in a granular,
“sandy” necrotic background. These cytologic findings are characteristic of which of the following
conditions?
A. Acute serositis
B. Renal failure
C. Systemic lupus erythematosus
D. Tuberculosis
E. Rheumatoid pleuritis

8. In differentiating metastatic adenocarcinoma from malignant mesothelioma in a pleural fluid, which


of the following stains favors the diagnosis of malignant mesothelioma?
A. Pancytokeratin +
B. Vimentin
C. Moc-31 +
D. B72.3 +
E. WT-1 +

9. What is the most common tumor that causes malignant peritoneal effusion in men?
A. Lung carcinoma
B. Gastric carcinoma
C. Pancreatic carcinoma
D. Prostatic carcinoma
E. Lymphoma/leukemia

10. A patient with human immunodeficiency virus (HIV) infection presents with a large pleural effusion. There
is no evidence of lymphadenopathy or organomegaly. The effusion cytology demonstrates dispersed large
cells with irregular nucleus and prominent nucleoli. Positive immunohistochemical stain to which of the
following antigens is essential for establishing the diagnosis of primary effusion lymphoma in this patient?
A. CD20
B. CD19
C. CD 3
D. Human herpes virus 8 (HHV-8)
E. Parvovirus

11. Hematoxylin body is a large glassy homogenous intracytoplasmic body in neutrophils or macrophages.
The identification of hematoxylin body in a pleural effusion is indicative of which of the following
conditions?
A. Viral infection
B. Tuberculosis infection
C. Lupus pleuritis
D. Rheumatoid pleuritis
E. Renal failure
58 Cytopathology Review

12. Which of the following organs is the most common site of tumors causing malignant pleural effusions
in women?
A. Breast
B. Lung
C. Ovary
D. Gastrointestinal tract
E. Lymphoma or leukemia

13. Cytogenetic analysis is considered highly sensitive and specific in the distinction between reactive
mesothelial cells and mesothelioma. Which of the following chromosomal aberrations occur most
commonly in mesothelioma?
A. Point mutation
B. Amplification
C. Deletion
D. Translocation
E. Microsatellite instability

14. What is the clinical significance of identifying positive tumor cells in the peritoneal washing specimen
of a patient with an ovarian serous borderline tumor?
A. It converts the borderline diagnosis to carcinoma
B. It upstages the borderline ovarian tumor
C. It predicts poor survival rate
D. It mandates postoperative chemotherapy
E. It requires intraperitoneal chemotherapy
15. All of the followings are indications for peritoneal washings, EXCEPT:
A. Staging ovarian cancer
B. Staging endometrial cancer
C. Staging pancreatic cancer
D. Excluding an occult cancer in patients with hysterectomy for leiomyomata
E. Treating primary peritoneal cancer

16. This cell group is seen in a smear from abdominal


fluid. Which of the following immunohistochemical
stains supports the diagnosis of metastatic
adenocarcinoma?
A. Pancytokeratin +
B. WT-1 +
C. Moc-31 +
D. Calretinin +
E. Podoplanin (D2-40) +
Effusions and Peritoneal Washings 59

17. This image represents a pleural effusion specimen


from a 43-year-old man with pneumonia. What are
these cells indicated by an arrow?
A. Histiocytes
B. Lymphocytes
C. Mesothelial cells
D. Squamous cells
E. Fibroblasts

18. This image was taken from a smear prepared from a


pleural effusion of a 43-year-old woman with a
clinical history of breast carcinoma. What is the
correct diagnosis?
A. Reactive mesothelial cells
B. Mesothelioma
C. Metastatic breast carcinoma
D. Granulomas
E. Lymphoma

19. A 78-year-old man with hepatitis C and liver cirrhosis


developed ascites. The effusion cytology is shown
here. What do these cells represent?
A. Reactive mesothelial cells
B. Metastatic lung adenocarcinoma
C. Metastatic renal cell carcinoma
D. Metastatic colonic adenocarcinoma
E. Metastatic squamous cell carcinoma
60 Cytopathology Review

20. A 68-year-old man presents with chest pain and


shortness of breath. Imaging study shows a large
right-sided effusion and pleural thickening. The
effusion smear is shown here. What is the most likely
diagnosis?
A. Reactive mesothelial cells
B. Mesothelioma
C. Metastatic adenocarcinoma
D. Melanoma
E. Lymphoma

21. This image was taken from a pleural fluid specimen


of a 57-year-old man who had a clinical history of
chronic lymphocytic leukemia. Which of the following
is the most appropriate test for this effusion
specimen?
A. Immunohistochemical stains on the cell block
sections
B. Flow cytometry analysis
C. Cytogenetic study
D. Gene rearrangement study
E. Microbiology culture

22. This structure (arrow) is commonly seen in peritoneal


washing specimens. What is the significance of this
cytologic finding?
A. Diagnostic of endometriosis
B. Diagnostic of endosalpingiosis
C. Diagnostic of ovarian borderline serous tumors
D. Indication of mesothelial hyperplasia
E. None (of no clinical significance)
Effusions and Peritoneal Washings 61

23. This image represents a large right-sided pleural


effusion specimen from a 58-year-old smoker who has
a history of working in a mining area 25 years ago.
What is the most likely diagnosis?
A. Reactive mesothelial cells
B. Granulomas
C. Mesothelioma
D. Metastatic lung adenocarcinoma
E. Metastatic gastric adenocarcinoma

24. A 62-year-old woman developed left-sided pleural


effusion and multiple lung nodules. This image
depicts the smear from the pleural effusion. What is
the correct diagnosis?
A. Lupus pleuritis
B. Rheumatoid pleuritis
C. Pleural tuberculosis
D. Metastatic carcinoma
E. Lymphoma

25. This image is taken from an H&E stained cellblock


section of a pleural fluid. Which of the features
depicted in this image is more commonly associated
with metastatic adenocarcinoma?
A. Cohesive groups
B. Flat sheet arrangement
C. Clear space around cell groups (lacunae)
D. Irregular contour of the cell groups
E. Background lymphocytes
62 Cytopathology Review

26. A 58-year-old man was admitted for pneumonia and


right side pleural effusion. Inquiry of the clinical
history revealed that he was a smoker and had a
history of head and neck squamous cell carcinoma.
The pleural effusion specimen was sent to cytology
and a representative image is shown here. The
cytologic features are highly suggestive of which of
the following diagnosis?
A. Empyema
B. Rheumatoid pleuritis
C. Tuberculous pleuritis
D. Metastatic squamous cell carcinoma
E. Mesothelioma

27. This cell group is seen in a pelvic washing specimen.


The bilateral ovaries were involved by serous
borderline tumors. What is the correct diagnosis?
A. Reactive mesothelial cells
B. Endosalpingiosis
C. Endometriosis
D. Positive for neoplastic cells
E. Positive for malignant cells

28. This peritoneal fluid specimen was collected from a


56-year-old woman. Her bilateral ovaries were also
involved by the same process. Which of the following
statements is correct?
A. It is most likely a primary ovarian carcinoma
B. It is most likely a primary peritoneal carcinoma
C. It is most likely a primary appendiceal carcinoma
D. It is most likely a primary rectal carcinoma
E. It is most likely a primary breast carcinoma
Effusions and Peritoneal Washings 63

29. Which of the following immunohistochemical stains


would support the diagnosis of appendiceal
adenocarcinoma in this peritoneal fluid specimen?
A. Calretinin+
B. Keratin CK7+
C. Epithelial membrane antigen (EMA)+
D. CDX2+
E. Estrogen receptor (ER)+

30. Based on morphology, what is the most likely


diagnosis of this ascites smear from a 72-year-old
man?
A. Metastatic colon cancer
B. Metastatic lung cancer
C. Metastatic prostate cancer
D. Metastatic renal cell carcinoma
E. Metastatic germ cell tumor

31. A 1-year-old boy was discovered to have a large


retroperitoneal mass and ascites. A representative
image of the abdominal fluid smear is shown here.
Based on the clinical setting and the cell morphology,
what is the most likely diagnosis?
A. Metastatic small cell carcinoma
B. Neuroblastoma
C. Desmoplastic small round cell tumor
D. Lymphoblastic lymphoma
E. Mesothelioma
64 Cytopathology Review

32. This pleural effusion is from a 26-year-old man. What


is the most likely diagnosis?
A. Reactive mesothelial cells
B. Reactive lymphocytosis
C. Mesothelioma
D. Diffuse large B-cell lymphoma
E. Metastatic carcinoma

33. A 45-year-old man developed recurrent ascites. He


did not have clinical history of liver disease or
malignancy. Previous fluid cytology showed reactive
mesothelial cells. This image shows a recent smear
from the ascitic fluid. Which of the following special
stains should be performed?
A. Fungal stain
B. Giemsa stain
C. Oil-Red-O stain
D. Mucin stain
E. Fontana-Masson stain

34. These cells are seen in a pleural effusion specimen.


All of the following are included in the differential
diagnoses based on morphology, EXCEPT:
A. Metastatic adenocarcinoma of the lung
B. Metastatic adenocarcinoma of stomach
C. Metastatic renal cell carcinoma
D. Metastatic melanoma
E. Metastatic small cell carcinoma
Effusions and Peritoneal Washings 65

35. This pelvic washing specimen is collected from a


women undergoing hysterectomy for endometrial
adenocarcinoma. What is the correct diagnosis?
A. Reactive mesothelial cells
B. Endometriosis
C. Endosalpingiosis
D. Metastatic endometrial adenocarcinoma
E. Mesothelioma

36. This peritoneal washing specimen is most likely


collected from a patient with which of the following
conditions?
A. Ovarian serous borderline tumor
B. Ovarian serous carcinoma
C. Ovarian granulosa cell tumor
D. Ovarian endometriosis
E. Ovarian torsion

37. This group of cells most likely represents which of


the following?
A. Benign mesothelial cells in peritoneal washing
B. Ovarian serous borderline tumor in peritoneal
washing
C. Metastatic pancreatic adenocarcinoma in ascites
D. Metastatic hepatocellular carcinoma in ascites
E. Malignant mesothelioma in ascites
66 Cytopathology Review

38. A 42-year-old man developed pleural effusion.


Inquiry of the clinical history revealed previous
thyroidectomy for “cancer” 7 years ago. This group
of cells identified in the effusion smear is highly
suggestive of which of the following diagnosis?
A. Reactive mesothelial hyperplasia
B. Mesothelioma
C. Metastatic papillary thyroid carcinoma
D. Metastatic lung adenocarcinoma
E. Metastatic lung squamous cell carcinoma

39. A 23-year-old patient with CDKN2A (cyclin-


dependent kinase 2A) mutation developed right
pleural effusion along with multiple metastatic lesions
in the liver. The effusion specimen has a high protein
level and high LDH level. The pleural effusion smear
is shown here. What is the correct diagnosis?
A. Mesothelioma
B. Metastatic lung adenocarcinoma
C. Metastatic gastric adenocarcinoma
D. Metastatic melanoma
E. Diffuse large B cell lymphoma

40. A 39-year-old woman developed fatigue, ascites and


right-sided pleural effusion. Clinical examination
revealed normal cardiac, liver and renal functions.
The pleural fluid’s cytology is shown here. Further
examination discovered a large ovarian mass. Based
on the effusion cytology and clinical history, what is
the most likely diagnosis of the ovarian mass?
A. Benign serous cystadenoma
B. Borderline serous tumor
C. Fibroma
D. Teratoma
E. Granulosa cell tumor
Effusions and Peritoneal Washings 67

ANSWERS

1. E leads to withdrawal of therapy, but sometimes they are


Note: Effusions are classified as transudates and an indication to intensify the therapy. Malignant
exudates based on the pathogenesis. Transudates result effusions are typically resistant to radiation therapy or
from an imbalance of hydrostatic and oncotic pressure chemotherapy. Chest tube placement or pleurodesis for
as occurring in congestive heart failure, cirrhosis and pleural effusions and repeated paracenteses for ascites
the nephritic syndrome. Exudates result from injury to are performed as palliative methods.
the mesothelium as occurring in inflammation and
malignancy. Typically, transudates have low specific 5. E
gravity, low fluid protein level and low LDH level and Note: A pleural effusion is called eosinophilic effusion
exudates have the opposite findings. when eosinophils comprise 10% or more of the
nucleated cells in the fluid. The most common causes
2. E are pneumothorax and hemothorax, caused by
Note: Reactive mesothelial cells can have significant the introduction of air or blood into the pleural space,
atypia which is one of the major diagnostic challenges respectively. Less common causes include drug
for cytopathologists. The atypia generally include a reactions, parasitic infections, and Churg-Strauss
spectrum of changes ranging from relatively normal syndrome. About one-third of eosinophilic effusions are
appearing mesothelial cells to mesothelial cell with idiopathic.
marked atypia, such as variation in nuclear size,
hyperchromatic nuclei with irregular nuclear membrane, 6. E
cytoplasmic vacuoles, mitosis, binucleation and Note: Conditions commonly associated with
multinucleation, and prominent nucleoli. Large cell lymphocytic effusions include malignancy, tuberculosis,
clusters (especially with 12 or more cells) are highly and status post coronary artery bypass surgery. Effusions
unusual for reactive mesothelial cells and should raise in renal failure and congestive heart failure commonly
a suspicion for malignancy. contain reactive mesothelial cells but no lymphocytes.
Rheumatoid pleuritis is characterized by abundant
3. E granular debris and macrophages in the fluid. Lupus
Note: Looking for a “second population” of cells besides pleuritis may have the characteristic lupus erythematosus
mesothelial cells in an effusion specimen is always a (LE) cells in the effusion. LE cell is a macrophage or
good way to start examining an effusion smear. Large neutrophil that contains a large basophilic glassy
cell clusters are usually indicative of malignancy. intracytoplasmic body, called hematoxylin body.
Metastatic adenocarcinoma, especially metastatic breast
carcinoma cells, often form “cannon balls” with smooth 7. E
borders in an effusion specimen. Lacunae around cell Note: The findings described in this case are
groups in cell block sections are artifacts and are characteristic of rheumatoid pleuritis. Acute serositis is
commonly seen in adenocarcinomas. Mitosis can be seen characterized by purulent effusions showing abundant
in benign reactive mesothelial cells as well as in neutrophils in cytology. Renal failure is marked by
malignant cells. reactive mesothelial cells in the effusion. Lupus contains
characteristic LE cells and reactive mesothelial cells.
4. A Tuberculosis is associated with lymphocytosis.
Note: Most patients with malignant effusions have a
previously known primary malignancy. In a minority of 8. E
cases malignant effusion may be the first presentation Note: Mesothelioma and adenocarcinoma are both
of an occult malignancy. Lung is the most common positive for pancytokeratin and negative for vimentin;
occult primary site in both men and women. In rare adenocarcinoma is positive for Moc-31 and B72.3 while
cases, the primary site is never discovered. Malignant mesothelioma is positive for WT-1 (Wilms tumor gene-
effusion is an ominous prognostic indicator; it often 1 encoded protein).
68 Cytopathology Review

9. E 15. E
Note: The most common tumor that causes malignant Note: Peritoneal washing is used as a staging procedure
peritoneal effusion in men is lymphoma or leukemia, for gynecologic malignancies (ovarian, fallopian tube
followed by gastrointestinal cancer and pancreatic cancer. and endometrial) and some non-gynecologic malig-
nancies (pancreatic and gastric). It is also used to rule
10. D out an occult cancer in patients undergoing surgery
Note: Demonstrating the presence of HHV-8 in the for benign conditions such as endometriosis or
tumor cells is an essential step in establishing the leiomyomata. It is not a treatment procedure for primary
diagnosis of primary effusion lymphoma (PEL). In most peritoneal cancer.
cases, the neoplastic cells are co-infected with Epstein-
Barr virus (EBV). PEL cells usually express CD45, but 16. C
are negative for CD19 and CD20. Tumor cells often Note: Metastatic adenocarcinomas react with antibody
express CD30, CD38 and CD138. to Moc-31, a transmembrane glycoprotein of unknown
function expressed in many adenocarcinomas.
11. C Mesothelial cells are positive for Wilms tumor-1 gene
Note: Hematoxylin bodies represent degenerated nuclei product (WT-1), calretinin and podoplanin (D2-40).
that are engulfed by neutrophils or macrophages. They Both adenocarcinoma cells and mesothelial cells are
often occupy the entire cytoplasm and push the nuclei positive for pancytokeratin.
to the side. The cells containing hematoxylin bodies are
also called lupus erythematosus (LE) cells as they 17. C
represent a characteristic feature of lupus pleuritis. Note: These cells demonstrate characteristic features of
mesothelial cells, including round cells with round
12. A nuclei, a perinuclear dense zone with a peripheral clear
Note: Breast cancer, followed by lung cancer and rim (“lacy skirt”), and windows between adjacent cells.
lymphoma or leukemia, are the top three causes of
malignant pleural effusions in women. Ovarian cancer 18. C
is the most common cause of malignant peritoneal Note: The image shows abundant spheres of malignant
effusion in women. cells with smooth borders, so called “cannonballs”,
which is a characteristic common pattern for metastatic
13. C breast carcinoma.
Note: Mesotheliomas are characterized genetically by
clonal chromosomal deletions, which commonly involve 19. A
1p, 3p, 6q, 9p, and 22q. Fluorescence in situ Note: These cells have round nuclei, perinuclear dense
hybridization (FISH) with appropriate probes can be zone and peripheral “skirt”, which are all features of
used to detect these deletions in cytologic specimens mesothelial cells. Some cells are enlarged with irregular
prepared from effusions. nuclear membrane and prominent nucleoli. These
reactive mesothelial cells can be seen in effusion
14. B specimens of the following medical conditions: cirrhosis,
Note: The identification of serous borderline tumor in a renal failure, pulmonary infarction, lupus and AIDS.
peritoneal washing specimen does not convert the Clinical correlation is very important in these conditions.
borderline ovarian tumor to carcinoma as the distinction
between a borderline and malignant ovarian tumor is 20. B
based on histologic identification of stromal invasion. Note: The specimen is cellular and composed of many
But the presence of such a group in peritoneal large clusters and dispersed large mesothelial cells. The
washing specimen upstages a borderline tumor from cells are typical of mesothelial cells with normal nuclear-
stage Ia/b to stage Ic. Postoperative chemotherapy offers to-cytoplasmic ratio, dense cytoplasm and peripheral
no survival advantage for women with early stage skirts. The typical clinical presentation (unilateral
borderline serous tumors. The overall survival for serous effusion and pleural thickening) combined with the
borderline tumor is good even without adjuvant effusion cytology findings (large clusters of mesothelial
chemotherapy cells) are diagnostic of mesothelioma.
Effusions and Peritoneal Washings 69

21. B cells have disorganized arrangement and demonstrate


Note: The image shows abundant small mature lympho- mild cytologic atypia, typical of serous borderline
cytes. Flow cytometry study is the most appropriate next tumors. Reactive mesothelial cells have a flat sheet
step test to confirm/exclude clonality of these lympho- arrangement and bland nuclei. Endosalpingiosis may be
cytes in order to differentiate between reactive lympho- difficult to distinguish from serous borderline tumors,
cytosis and chronic lymphocytic leukemia. however, the former usually have cilia at the periphery
of cell groups and do not have cytologic atypia. A
22. E malignant diagnosis should be avoided in this case
Note: This spherical mass is called collagen balls, formed because a positive washing specimen does not convert
by flattened mesothelial cells surrounding a ball of a borderline ovarian tumor to a malignant tumor. This
collagen. Collagen balls are a common finding in peri- patient was later found to have multiple implants on the
toneal washings and have no known clinical significance. peritoneum and omentum corresponding to the positive
finding in the pelvic washing specimen.
23. C
Note: The image shows large clusters of mesothelial 28. C
cells with knobby edges (“mulberry” appearance). This Note: This peritoneal fluid shows abundant mucin and
morphologic finding in light of the clinical history of groups of highly atypical epithelial cells, these are
asbestos exposure is diagnostic of mesothelioma. characteristic features of pseudomyxoma peritonei.
Bilateral ovarian involvement by mucinous tumors plus
24. B pseudomyxoma peritonei often represent metastasis,
Note: The smear shows clumps of granular debris, with appendix being the most likely primary site.
scattered histiocytes and a giant cell; these features are
characteristic and diagnostic of rheumatoid pleuritis. 29. D
They correspond to pleural rheumatoid nodules which Note: This image shows similar features as the previous
are necrotizing granulomas. The lung nodules most case with abundant mucin and a group of highly atypical
likely are also rheumatoid nodules. neoplastic cells. Appendiceal mucinous adenocarcino-
mas typically do not react immunohistochemically with
25. C the antibodies to the antigens listed here except for
Note: Lacunae around cell groups are seen in 75% of CDX2, which is typically expressed in colonic and
cellblocks of malignant effusions, most commonly appendiceal adenocarcinomas. This protein is encoded
adenocarcinomas. They are artifacts of unknown cause. by the CDX2 homeobox gene, which acts as a tumor
Although this feature is very helpful in identifying suppressor gene in the ileocecal region.
malignant effusions, it is not entirely specific as it can
also be seen in a small percentage of benign effusions. 30. A
The other features listed in the answers are more Note: The smear shows a malignant epithelial group with
commonly seen in benign effusions. columnar cell morphology and an adjacent clump of
necrotic debris (“dirty necrosis”). These morphologic
26. D findings are characteristic of metastatic colonic
Note: The smear shows necrotic debris, keratin debris adenocarcinoma.
(arrow) and malignant cells consistent with metastatic
squamous cell carcinoma. Keratin debris or anucleated 31. B
squamous are always abnormal findings in pleural Note: The tumor cells appear as “small blue cells” with
effusions. Squamous cell carcinoma metastatic to pleura, scant cytoplasm. The cell morphology is compatible with
pericardium or peritoneum is very rare; the primary small cell carcinoma, neuroblastoma or desmoplastic
carcinoma is almost always diagnosed before the small round cell tumor. However, small cell carcinoma
effusion develops. occurs in an old age group; desmoplastic small round cell
tumor affects adolescents and young adults; neuro-
27. D blastoma is the most common malignancy in the neonate
Note: The smear shows a papillary epithelial cell cluster and most tumors arise in the adrenal gland or anywhere
containing multiple psammoma bodies. The epithelial along the sympathetic chain. Lymphoblastic lymphoma
70 Cytopathology Review

has large, discohesive tumor cells. Mesothelioma has large recognized in effusions by the presence of clusters of
tumor cells in clusters and isolated forms. large pleomorphic tumor cells as shown in this figure.
Tumor cells often have abundant vacuolated cytoplasm.
32. D Ovarian torsion and ovarian endometriosis are associated
Note: The cells are singly dispersed, have large nuclei, with reactive mesothelial cells in a peritoneal washing
high nuclear to cytoplasmic ratio, and prominent nucleoli. specimen. Granulosa cell tumor rarely involves perito-
Mesothelial cells and carcinoma cells form cohesive neal space and the cells are not as pleomorphic. Ovarian
groups. Reactive lymphocytes have small mature nuclei serous borderline tumors do not have such overly
with clumped chromatins. Focal karyorrhexis (not shown malignant cytologic features.
in this image) is also seen in the smear; this is a feature
commonly associated with lymphomas, but uncommon 37. A
in benign effusions or other malignant effusions caused Note: This type of large flat sheet of cells is charac-
by non-lymphoid neoplasms. teristic of benign mesothelial cells seen in peritoneal
washing specimens.
33. D
Note: The cell in the center has a signet-ring appearance 38. C
and possible intracytoplasmic mucin. There are also Note: It is a papillary group with psammoma bodies.
scattered reactive mesothelial cells in the smear. The Although the chromatin detail is not clearly seen in this
malignant cells are difficult to identify because they image, the finding of a papillary epithelial group with
resemble histiocytes or mesothelial cells. A positive psammoma bodies in a pleural effusion of a patient with
mucin stain would confirm a diagnosis of adeno- a clinical history of thyroid cancer is highly suggestive
carcinoma. This patient was later discovered to have a of metastatic papillary thyroid carcinoma.
gastric signet ring cell adenocarcinoma.
39. D
34. E Note: The cell in the center is large, binucleated with
Note: The cells have abundant clear cytoplasm and prominent nucleoli and diagnostic cytoplasmic melanin
prominent nucleoli, features not compatible with small pigment. CDKN2A mutation is associated with familial
cell carcinoma. and sporadic melanomas. The clinical history and cyto-
logic findings are characteristic for metastatic melanoma.
35. D
Note: The cells forming the crowded groups are large 40. C
with hyperchromatic and clumped chromatin and Note: The cytology shows benign mesothelial cells and
prominent nucleoli. These are features of metastatic scattered small mature lymphocytes. The setting in this
adenocarcinoma. case is characteristic of Meigs syndrome, which includes
a triad of ascites, pleural effusion, and benign ovarian
36. B fibroma. These effusions typically resolve after resection
Note: Ovarian serous carcinoma can be easily of the ovarian tumor.
5
Cerebrospinal Fluid
Fang Fan

QUESTIONS
1. All of the following cells may be seen in a normal cerebrospinal fluid (CSF), EXCEPT:
A. Lymphocytes
B. Plasma cells
C. Monocytes
D. Red blood cells
E. Brain tissue

2. Abundant plasma cells seen in a CSF specimen may be related to which of the following conditions?
A. Acute bacterial meningitis
B. Epileptic seizure
C. Subarachnoid hemorrhage
D. Intraventricular hemorrhage
E. Multiple sclerosis

3. In a patient with acquired immune deficiency syndrome (AIDS), numerous neutrophils in the CSF are
highly suggestive of which of the following conditions?
A. Cytomegalovirus (CMV) radiculopathy
B. Pneumocystis jiroveci infection
C. Cryptococcus neoformans infection
D. Coccidioides immitis infection
E. Parasitic infection

4. Which of the following conditions is associated with eosinophils in CSF?


A. Acute bacterial meningitis
B. Viral meningitis
C. Cerebral infarction
D. Multiple sclerosis
E. Ventriculoperitoneal shunts
5. What type of cells are “Mollaret cells” seen in CSF specimens?
A. Lymphocytes
B. Monocytes
C. Plasma cells
D. Neutrophils
E. Macrophages
72 Cytopathology Review

6. A CSF specimen was obtained from an immunocompromised patient who developed meningoencepha-
litis. Examination of the CSF cytology revealed small crescent-shaped organisms with a tiny round
nucleus in the cytoplasm of macrophages. Which of the following organisms causing the infection?
A. Cysticercus cellulosae
B. Toxoplasma gondii
C. Cryptococcus neoformans
D. Histoplasma capsulatum
E. Mycobacterium tuberculosis

7. Malignant epithelial cells were found in the CSF. Which of the following organs is the most common
site of tumors that metastasize to the brain?
A. Oropharynx
B. Lungs
C. Colon
D. Kidneys
E. Urinary bladder

8. What is the most common type of primary central nervous system lymphoma?
A. Diffuse large B cell lymphoma
B. Small lymphocytic lymphoma
C. Follicular lymphoma
D. Peripheral T cell lymphoma
E. Anaplastic T cell lymphoma

9. Which of the following primary central nervous system tumors will least likely spread to CSF?
A. Ependymoma
B. Meningioma
C. Choroid plexus tumor
D. Astrocytoma
E. Medulloblastoma

10. This cell in a CSF specimen represents which of the


following?
A. Macrophage
B. Chondrocyte
C. Squamous cell
D. CMV inclusion
E. Parasite
Cerebrospinal Fluid 73

11. This positive mucin stain demonstrates that this CSF


specimen is positive for which of the following
conditions?
A. Metastatic lobular carcinoma of the breast
B. Toxoplasmosis
C. Foreign material
D. Cryptococcal meningitis
E. Cysticercosis

12. This image was taken from a CSF specimen. What


does this structure represent?
A. Necrotic debris
B. Fungal organisms
C. Parasites
D. Brain tissue
E. Bone marrow elements

13. This CSF specimen was obtained from a patient with


a clinical history of acute lymphoblastic leukemia.
What is the correct interpretation?
A. Aseptic meningitis
B. Viral meningitis
C. Multiple sclerosis
D. Melanoma
E. Leukemia
74 Cytopathology Review

14. What is the correct diagnosis of this CSF cytology


from a 56-year-old man?
A. Reactive lymphocytes
B. Lymphoma
C. Melanoma
D. Metastatic adenocarcinoma
E. Metastatic small cell carcinoma

15. What is the most likely diagnosis of this CSF


specimen (cell block) from a 43-year-old patient?
A. Ependymoma
B. Choroid plexus papilloma
C. Medulloblastoma
D. Glioblastoma multiforme
E. Lymphoma

16. A 46-year-old woman with a history of breast cancer


six years ago presented with headache. A CSF sample
was collected and shown here. The finding is most
consistent with which of the following diagnoses?
A. Reactive lymphocytes
B. Macrophages
C. Metastatic breast carcinoma
D. Plasmacytoma
E. Melanoma
Cerebrospinal Fluid 75

17. This image was taken from a CSF specimen. What is


the most likely interpretation?
A. Melanoma
B. Medulloblastoma
C. Leukemia
D. Small cell carcinoma
E. CMV infection

18. This CSF specimen is seen in a patient with fever and


headache. What would be the most appropriate
ancillary test for this specimen?
A. Immunohistochemistry
B. Mucin stain
C. Fungal stain
D. Gram stain
E. Flow cytometry

19. What is the correct diagnosis of this CSF sample?


A. Metastatic adenocarcinoma
B. Metastatic small cell carcinoma
C. Glioblastoma
D. Ependymoma
E. Leukemia
76 Cytopathology Review

20. What is the correct diagnosis for this smear prepared


from a CSF sample?
A. Metastatic small cell carcinoma
B. Leukemia
C. Metastatic squamous cell carcinoma
D. Metastatic adenocarcinoma
E. Astrocytoma

21. An 8-year-old boy has a large cerebellar mass and


this finding in the CSF. What is the correct diagnosis?
A. Lymphoma
B. Leukemia
C. Medulloblastoma
D. Glioblastoma multiforme
E. Small cell carcinoma

22. A 72-year-old patient who was a smoker and had a


history of lung cancer developed neurologic
symptoms. CSF examination revealed scant cells as
shown here in the image. What do these cells
represent?
A. Macrophages
B. Ependymal cells
C. Monocytes
D. Carcinoma
E. Leukemia
Cerebrospinal Fluid 77

ANSWERS

1. B 8. A
Note: Lymphocytes and monocytes are the usual Note: Diffuse large B cell lymphoma is the most
components of a normal CSF. Inflammatory cells, such common type of primary central nervous system
as macrophages, plasma cells and eosinophils are lymphoma. CSF cytology is positive in one-third of
abnormal findings in CSF. Red blood cells are the cases. Flow cytometry is very helpful in proving
commonly seen in a normal CSF due to procedure clonality.
related trauma. Brain tissue may be seen in CSF samples
obtained through a ventricular tap. Choroid plexus/ 9. B
ependymal cells are seen rarely, i.e. in less than 0.5% Note: It is extremely uncommon for meningioma to
of lumbar puncture specimens. spread through CSF.
2. E 10. B
Note: Plasma cells in CSF are seen in patients with Note: This large cell with a small nucleus is surrounded
multiple sclerosis, but may be also associated with viral by a halo and myxoid matrix, representing a chondro-
meningitis, tuberculosis, and syphilis. cyte. Such cells are rarely seen in smears prepared from
CSF and should not be mistaken for malignancy.
3. A
Note: In AIDS patients, numerous neutrophils in CSF 11. D
are highly suggestive of CMV infection. Note: Cryptococcus neoformans have a muco-
polysaccharide capsule that is characteristically mucin-
4. E positive. This smear shows abundant fungal forms and
Note: Eosinophils in CSF may be associated with characteristic tear-drop shaped budding (arrow).
ventriculoperitoneal shunts. Other causes of eosinophilia
in the CNS are parasitic infections and Rocky Mountain 12. D
spotted fever. Note: This tissue fragment has a fibrillary texture-
characteristic of brain tissue. A CSF specimen may
5. B contain brain tissue if it is collected through a ventricular
Note: Mollaret meningitis or idiopathic recurrent tap.
meningitis is a rare form of aseptic meningitis charac-
terized by recurring attacks of fever, headache, and neck 13. E
stiffness. CSF findings are non-specific, but often times Note: The cells have irregular nuclei with fine chromatin
marked by monocytosis. Mollaret cells are monocytes and prominent nucleoli, features characteristic of
with deep nuclear clefts; the nucleus has a footprint-like blasts. It is usually sufficient to identify blasts in
appearance. Mollaret cells can be seen in Mollaret CSF; a definitive diagnosis of AML or ALL requires
meningitis, sarcoidosis or Behçet disease. bone marrow biopsy and appropriate hematologic
studies.
6. B
Note: The above description delineates the typical 14. E
features of tachyzoites of Toxoplasma gondii. Note: The cells are small and form small clusters with
nuclear molding, features characteristic of small cell
7. B carcinoma. Small cell carcinoma in CSF specimens may
Note: Lung carcinoma is the most common malignancy display in a linear molded arrangement with a “vertebral
metastasizing to the brain. Other common primary body” appearance.
tumors include carcinoma of the breast, stomach,
kidneys, malignant melanoma and lymphoma. It is to 15. D
be noted that metastatic tumors are much more frequent Note: The smear shows necrotic debris and large tumor
than primary CNS tumors in CSF. cells which have hyperchromatic nuclei and some
78 Cytopathology Review

multinucleated, therefore the most likely diagnosis is atypical large lymphocytes. Flow cytometry will help
Glioblastoma multiforme. Ependymoma cells are more differentiate between a reactive process and a neoplastic
uniform with round eccentric nuclei. Choroid plexus process in this case. Flow cytometry is highly sensitive
papilloma is characterized by uniform cuboidal cells and can detect malignant cells that represent as few as
forming large clusters. Medulloblastoma is a small blue 2% of the total cell population in CSF samples.
cell tumor composed of small to medium sized tumor
cells with hyperchromatic nuclei and nuclear molding. 19. A
Lymphoma cells do not have this degree of pleomor- Note: These tumor cells have abundant cytoplasm,
phism. prominent nucleoli and form gland-like structures
consistent with metastatic adenocarcinoma.
16. C
Note: The cells are singly dispersed with irregular nuclei, 20. A
prominent nucleoli and finely vacuolated cytoplasm- Note: The tumor cells grow in a crowded group with
characteristic of metastatic carcinoma. In a patient with prominent nuclear molding. This cytomorphologic
clinical history of breast carcinoma, the finding is most feature is characteristic of metastatic small cell
consistent with metastatic breast carcinoma. carcinoma.

17. A 21. C
Note: The cells have prominent cherry-red nucleoli and Note: The age and tumor location are characteristic of
abundant cytoplasm, this morphology is less likely to medulloblastoma. The image here demonstrates a
be medulloblastoma and small cell carcinoma (so called crowded group of small blue tumor cells. The charac-
small blue cell tumors). Blasts in leukemia have scant teristic rosette formation is not evident in this image.
cytoplasm, finely granular chromatin and small nucleoli.
Intracytoplasmic melanin pigment, if present, is the 22. D
diagnostic clue. Otherwise, a definitive diagnosis relies Note: The image shows two cohesive cells with enlarged
on clinical history and immunohistochemistry. nuclei (compare to an adjacent monocyte) and thin
delicate cytoplasm, one cell has irregular nuclear
18. E membrane, the other has a prominent nucleolus. These
Note: This specimen shows lymphocytosis with some are cytomorphologic features of adenocarcinoma.
6
Breast
Fang Fan

QUESTIONS
1. Fine needle aspiration (FNA) of breast is most reliable in which of the following differential diagnoses?
A. To distinguish ductal proliferative lesions, including usual ductal hyperplasia, atypical ductal hyperplasia
and low-grade ductal carcinoma in situ.
B. To distinguish high-grade ductal carcinoma in situ from invasive carcinoma
C. To distinguish intraductal papilloma from florid ductal hyperplasia
D. To distinguish fibroadenoma from phyllodes tumor
E. To distinguish fibroadenoma from invasive carcinoma

2. Which of the following morphologic features is most characteristic of malignancy in smears prepared
from breast fine needle aspirations?
A. Hypercellularity
B. Cohesive crowded epithelial groups
C. Isolated cells with nuclear atypia
D. Cytoplasmic vacuoles
E. Prominent nucleoli

3. A 55-year-old woman presented with a breast mass. She had lumpectomy and radiation therapy for
breast cancer on the same breast eight years ago. She is at increased risk for developing which of the
following tumors?
A. Paget disease
B. Inflammatory carcinoma
C. Angiosarcoma
D. Medullary carcinoma
E. Lymphoma

4. A 32-year-old woman noticed a right breast mass 1 week ago. Her grandmother and mother were
diagnosed with breast cancer at very young ages. She was tested positive for a genetic BRCA1 mutation.
Fine needle aspiration of the breast mass was performed and showed invasive breast cancer. Which of
the following is the most likely characteristic feature of her breast cancer?
A. Invasive lobular carcinoma, luminal A
B. Invasive ductal carcinoma, luminal B
C. Invasive ductal carcinoma, HER2 positive
D. Tubular carcinoma, luminal A
E. Medullary carcinoma, triple negative
80 Cytopathology Review

5. A 67-year-old woman with a prior history of breast cancer presented with headache. Image study of
the brain revealed findings indicative of “carcinomatous meningitis”. Cerebrospinal fluid smear showed
small, poorly cohesive cells and signet ring cells. What is the most likely diagnosis for this patient?
A. Metastatic ductal carcinoma of the breast
B. Bacterial meningitis
C. Metastatic lobular carcinoma of the breast
D. Viral meningitis
E. Multiple sclerosis

6. The smear from a fine needle aspiration (FNA) of a


breast lesion in a 42-year-old woman is shown here.
What is the correct diagnosis?
A. Non-diagnostic specimen
B. Fat necrosis
C. Fibrocystic changes
D. Intraductal papilloma
E. Fibroadenoma

7. What is the correct interpretation of this smear


prepared from a breast fine needle aspiration?
A. Normal breast lobule
B. Fibrocystic changes
C. Fibroadenoma
D. Ductal hyperplasia without atypia
E. Ductal hyperplasia with atypia
Breast 81

8. This 42-year-old woman developed a right breast


mass after a recent car accident. Fine needle
aspiration of the mass was performed and the smear
is shown here. What is the diagnosis?
A. Non-diagnostic specimen
B. Fat necrosis
C. Fibrocystic changes
D. Subareolar abscess
E. Acute mastitis

9. A 52-year-old woman who was a smoker presented


with a firm subareolar mass. Fine needle aspiration
of the mass was performed and the smear is shown
here. What is the diagnosis?
A. Fat necrosis
B. Subareolar abscess
C. Fibrocystic changes
D. Intraductal papilloma
E. Metaplastic carcinoma

10. What is the correct interpretation of this breast fine


needle aspiration smear?
A. Fibrocystic changes
B. Ductal hyperplasia without atypia (usual ductal
hyperplasia)
C. Fibroadenoma
D. Ductal carcinoma in situ
E. Invasive carcinoma
82 Cytopathology Review

11. What is the best designation for these cells (arrow)


in a smear obtained by breast fine needle aspiration?
A. Apocrine cells
B. Macrophages
C. Granular cells
D. Myoepithelial cells
E. Carcinoma cells

12. A 32-year-old woman presented with a 2.0 cm


movable rubbery right breast mass. A fine needle
aspiration was performed and the smear is shown
here. What is the correct diagnosis?
A. Ductal hyperplasia without atypia
B. Ductal hyperplasia with atypia
C. Fibroadenoma
D. Intraductal papilloma
E. Ductal carcinoma

13. A 29-year-old pregnant woman was found to have a


discrete breast nodule. Fine needle aspiration was
performed and the smear is shown here. What is the
correct interpretation?
A. Fibrocystic changes
B. Pregnancy-related changes
C. Acute mastitis
D. Invasive ductal carcinoma
E. Invasive lobular carcinoma
Breast 83

14. What is the correct diagnosis for this smear prepared


from a fine needle aspiration of a breast mass of a
54-year-old woman?
A. Acute mastitis
B. Fat necrosis
C. Fibroadenoma
D. Ductal carcinoma
E. Lobular carcinoma

15. A 42-year-old woman presented with a “lumpy-


bumpy” breast. Fine needle aspiration is performed
and the smear is shown here. What is the correct
interpretation?
A. This lesion needs routine follow-up
B. This lesion needs excision
C. This lesion needs antibiotics treatment
D. This lesion needs to be re-biopsied
E. This lesion needs radiation therapy

16. What is the most likely diagnosis of this cytology


smear from a breast fine needle aspiration?
A. Invasive ductal carcinoma
B. Invasive lobular carcinoma
C. Tubular carcinoma
D. Colloid carcinoma
E. Medullary carcinoma
84 Cytopathology Review

17. This breast fine needle aspiration smear was obtained


from a 65-year-old woman. What is the correct
diagnosis?
A. Insufficient for diagnosis
B. Abscess
C. Fibroadenoma
D. Invasive lobular carcinoma
E. Mucinous carcinoma

18. What is the correct diagnosis for this smear obtained


by aspiration biopsy of a breast lesion in a 72-year-
old woman?
A. Lymphoma
B. Melanoma
C. Small cell carcinoma
D. Medullary carcinoma
E. Invasive lobular carcinoma

19. This smear was prepared by fine needle aspiration


of a breast mass in a 45-year-old man. What is the
correct diagnosis?
A. Fibrocystic changes
B. Intraductal papilloma
C. Gynecomastia
D. Invasive ductal carcinoma
E. Metastatic carcinoma
Breast 85

20. This smear represents a nipple discharge smear


obtained from a 40-year-old woman. What is the
correct interpretation?
A. Duct ectasia
B. Fibrocystic changes
C. Papillary neoplasm
D. Fibroadenoma
E. Invasive ductal carcinoma

21. A 46-year-old woman had lumpectomy and radiation


therapy for invasive breast carcinoma 6 month ago
developed a mass in the previous surgery area. A fine
needle aspiration was performed and the smear is
shown here. What is the correct diagnosis?
A. Recurrent breast carcinoma
B. Sarcoma
C. Granulomas
D. Abscess
E. Radiation changes

22. A 45-year-old man presented with a subareolar breast


mass. Fine needle aspiration is performed and shown
here. What is the diagnosis for this patient?
A. Gynecomastia
B. Subareolar abscess
C. Invasive ductal carcinoma
D. Phyllodes tumor
E. Fat necrosis
86 Cytopathology Review

23. A 15-year-old girl presented with a large right breast


mass. Fine needle aspiration of the mass was
performed and the smear is shown here. What is the
diagnosis?
A. Fibroadenoma
B. Fibrocystic changes
C. Florid ductal hyperplasia
D. Ductal carcinoma in-situ
E. Invasive ductal carcinoma

24. A 59-year-old woman presented with a well-


circumscribed left breast mass. The fine needle
aspiration was performed and the smear is shown
here. What is the most likely interpretation?
A. Lymphoma
B. Small cell carcinoma
C. Medullary carcinoma
D. Melanoma
E. Invasive lobular carcinoma

25. A 47-year-old woman had a right breast “cystic


lesion” detected by ultrasound examination. The
smear prepared from an aspirate of the cyst fluid is
shown here. What is the diagnosis?
A. Apocrine cyst
B. Invasive ductal carcinoma
C. Duct ectasia
D. Fat necrosis
E. Mucinous carcinoma
Breast 87

26. This fine needle aspiration was obtained from a right


breast lesion is a 65-year-old woman and the smear
is shown here. What is the correct interpretation?
A. Fibroadenoma
B. Intraductal papilloma
C. Invasive lobular carcinoma
D. Mucinous carcinoma
E. Metaplastic carcinoma

27. A 32-year-old lactating woman developed a painful


right breast nodule. Fine needle aspiration of the
nodule was performed and shown here. What is the
correct diagnosis?
A. Fat necrosis
B. Duct ectasia
C. Subareolar abscess
D. Acute mastitis
E. Fibrocystic changes

28. This smear represents a fine needle aspiration from


a large breast lesion. What is the structure indicated
by arrows?
A. Necrosis
B. Calcifications
C. Granulomas
D. Foreign material
E. Mucin
88 Cytopathology Review

29. What is the correct diagnosis of this breast fine needle


aspiration specimen?
A. Fibroadenoma
B. Phyllodes tumor
C. Fat necrosis
D. Granuloma
E. Invasive ductal carcinoma

30. What does this group of cells from a breast fine needle
aspiration represent?
A. Benign breast lobules
B. Ductal hyperplasia without atypia
C. Granuloma
D. Ductal carcinoma in situ
E. Invasive ductal carcinoma

31. A 52-year-old woman developed a firm nodule in the


previous lumpectomy site. A fine needle aspiration
was performed and shown here. What is the
diagnosis?
A. Recurrent ductal carcinoma
B. Metaplastic carcinoma
C. Angiosarcoma
D. Fibrocystic changes
E. Lumpectomy scar
Breast 89

32. This breast fine needle aspiration smear was collected


from a 40-year-old woman. What is the correct
interpretation?
A. Benign breast lobules
B. Fibroadenoma
C. Intraductal papilloma
D. Ductal carcinoma
E. Lobular carcinoma

33. A 32-year-old woman presented with abnormal


mammogram. A fine needle aspiration was
performed and shown here. What is the correct
diagnosis?
A. Fibroadenoma
B. Atypical ductal hyperplasia
C. Tubular carcinoma
D. Invasive ductal carcinoma
E. Medullary carcinoma

34. What is the diagnosis of the breast fine needle


aspiration smear?
A. Invasive ductal carcinoma
B. Invasive lobular carcinoma
C. Small cell carcinoma
D. Lymphoma
E. Plasmacytoma
90 Cytopathology Review

ANSWERS

1. E 6. C
Note: Fibroadenomas can be usually distinguished from Note: The image shows amorphous cyst contents and
invasive carcinomas in FNA smears: fibroadenoma has apocrine cells (arrow). These cytologic findings are
the typical three components; invasive carcinoma has characteristic of fibrocystic changes. Apocrine cells have
loosely cohesive and singly dispersed cells with highly abundant granular cytoplasm, centrally located nuclei,
atypical morphology. All other entities listed here (i.e. and prominent nucleoli. These cells are usually arranged
ductal proliferative lesions, including usual ductal in small sheets. The diagnosis of fat necrosis, intraductal
hyperplasia, atypical ductal hyperplasia and low-grade papilloma and fibroadenoma can be excluded because
ductal carcinoma in situ) are defined primarily by their these lesions have typical cytologic features, which will
architectural features in histology sections; therefore, it be described later. In breast fine needle aspiration
is difficult to distinguish these lesions one from another practice, a specimen is marked as “non-diagnostic
on fine needle aspirations. However, there are cytomor- (insufficient) specimen” if the smear shows scant
phologic features (described below) that can help us cellularity, i.e. fewer than 6 epithelial cell clusters of at
favor one over the other on cytology smears. Intraductal least 5 to 10 cells each.
papilloma is cytologically indistinguishable from florid
ductal hyperplasia; however, clinical presentation, such 7. A
as nipple discharge or a subareolar mass may favor the Note: This smear shows well-maintained lobular
diagnosis of intraductal papilloma. Separation of architecture with small and round lobules corresponding
fibroadenoma from phyllodes tumor may be difficult on to the terminal ductal-lobular units (TDLU) in histology.
breast FNAs, reflecting the fact that these two lesions Other suggested answers can be excluded because all
have many overlapping histologic features in common. these lesions have typical cytologic features not shown
here. Smear obtained from a breast with fibrocystic
2. C changes would show foam cells and apocrine cells.
Note: Singly dispersed or isolated atypical cells are Fibroadenoma is characterized by hypercellular smears
highly characteristic of breast carcinoma. Hypercellu- containing large epithelial groups, fibrillar stromal
larity and cohesive crowded epithelial groups can be fragments, and bare oval nuclei in the background.
seen in benign conditions such as proliferative breast Ductal hyperplasia with or without atypia are composed
disease and fibroadenomas. Cytoplasmic vacuoles and of large epithelial cells, arranged in groups without the
prominent nucleoli can be seen in reactive changes such maintained lobular architecture as shown here.
as radiation changes. 8. B
Note: The image shows foamy macrophages with oval
3. C nuclei and abundant vacuolated cytoplasm (arrow). In
Note: Previous radiation therapy or lymphedema light of the clinical history of trauma, the findings are
associated with axillary node dissection are risk factors consistent with fat necrosis. Other diagnoses listed here
for developing angiosarcoma in the breast. are not correct. Subareolar abscess is characterized by
numerous anucleated squamous admixed with neutro-
4. E phils due to squamous metaplasia of lactiferous ducts
Note: Breast cancers associated with BRCA mutation with subsequent keratin plugging and rupture. Acute
are more commonly medullary type and are typically mastitis usually occurs during lactation and the aspirate
triple negative (ER-/PR-/Her2-). of this inflammatory lesion contains numerous
neutrophils readily identified in the smears.
5. C
Note: Invasive lobular carcinoma of the breast has a 9. B
different metastatic pattern from invasive ductal Note: The image shows scattered anuclear squamous
carcinoma. Lobular carcinomas tend to metastasize to cells (arrow) admixed with neutrophils, characteristic
leptomeninges causing “carcinomatous meningitis”. of a subareolar abscess.
Breast 91

10. B 14. D
Note: This cohesive cell group is arranged in a flat sheet Note: The smear shows loosely cohesive and singly
without significant nuclear overlapping. The nuclei have dispersed cells. Cells have marked pleomorphism with
finely granular chromatin and inconspicuous nucleoli. enlarged nuclei and prominent nucleoli. These are
Scattered small dark nuclei of myoepithelial cells can features of ductal carcinoma.
be seen within the group (arrow). These cytologic
features are typical of ductal hyperplasia without atypia. 15. A
Note: This image shows apocrine cells and proteina-
11. A ceous cyst contents, i.e. features characteristic of
Note: Apocrine cells (as shown here) are usually fibrocystic changes. Fibrocystic changes are not asso-
arranged in sheets. Cells have centrally located, round ciated with increased risk of developing breast cancer;
nuclei, prominent nucleoli, and abundant granular therefore, the biopsied woman only needs routine follow
cytoplasm. They appear distinctly different from an up.
adjacent group of benign ductal epithelial cells (arrow-
16. B
head). Macrophages have irregularly shaped or
Note: The smear shows singly dispersed uniform
eccentrically located nuclei and vacuolated cytoplasm.
malignant cells. The predominant pattern is cell
They differ from granular cells which have small nuclei
dissociation with characteristic single cell filing (arrow).
and a coarsely granular cytoplasm. Myoepithelial cells
These cytologic findings are typical of invasive lobular
appear in smears as singly dispersed cells with elongated
carcinoma. Invasive ductal carcinoma may show more
nuclei and scant cytoplasm. Carcinoma cells are usually
pleomorphism and some loosely cohesive tumor cell
discohesive and show marked nuclear atypia.
groups. Tubular carcinoma is characterized by cohesive
epithelial groups with formation of rigid tubular archi-
12. C tecture (so called “hose pipe-like” structure). Colloid
Note: This smear is very cellular and shows charac- carcinoma is composed of abundant mucin admixed with
teristic features of a fibroadenoma which include three loosely cohesive tumor cell groups. Medullary carci-
components: numerous branching fragments of epithelial noma is characterized by large pleomorphic tumor cells
groups (arrow), fibromyxoid stromal fragments (arrow- in association with a background of lymphocytes.
head), and abundant myoepithelial cells in the back-
ground. The combination of these three features separate 17. E
fibroadenoma from ductal hyperplasia (with or without Note: The image shows clusters of tumor cells associated
atypia), the smears of which typically contain only with pools of dense mucin (arrow), characteristic of
epithelial groups. Intraductal papilloma or papillary mucinous carcinoma of the breast.
neoplasms do not have the fibrillary stromal fragments
and the background naked bipolar cells. Ductal 18. C
carcinoma has loosely cohesive and singly dispersed Note: Primary small cell carcinoma of the breast is
tumor cells with marked nuclear atypia. uncommon and metastasis needs to be excluded before
a diagnosis of primary small cell carcinoma of the breast
13. B is made. The cytologic features are similar to those seen
Note: Pregnancy may occasionally be associated with in the lung, including cell clusters with cells having high
the development of a discrete nodule in the breast called N/C ratio and nuclear molding.
lactating adenoma. FNA shows a cellular smear
composed of loose cell clusters with cells having enlar- 19. C
ged nuclei, prominent nucleoli and finely vacuolated Note: The findings of gynecomastia are similar to those
cytoplasm. The smear may have a proteinaceous and in fibroadenomas. This smear contains a large cohesive
frothy background with numerous round naked nuclei and branching epithelial group in a flat sheet
representing fragile epithelial cells’ nuclei (not shown arrangement and scattered bare ovoid nuclei in the
here). Clinical history is the key to establishing such a background, these are characteristic for the diagnosis of
diagnosis. gynecomastia.
92 Cytopathology Review

20. C 27. D
Note: The smear shows a crowded three-dimensional Note: The smear shows abundant neutrophils
papillary group, features of a papillary neoplasm. characteristic of acute mastitis. Acute mastitis almost
Intraductal papilloma is the most common cause of always occurs during lactation and is commonly caused
nipple discharge in women, although duct ectasia, by bacteria infection. Fat necrosis is composed of
fibrocystic changes and invasive ductal carcinoma may necrotic fat and macrophages. Duct ectasia may show
be associated with nipple discharge as well. Duct ectasia dense inspissated secretions and foamy macrophages.
would demonstrate thick secretion and foamy Subareolar abscess is characterized by anucleated
macrophages. Fibrocystic changes have apocrine cells squamous cells admixed with inflammatory cells.
and small benign ductal epithelial groups. Invasive Fibrocystic changes show apocrine cells, foam cells and
ductal carcinoma contains loosely cohesive and single cyst contents.
malignant cells.
28. B
21. E Note: The structure shown here represents calcifications
Note: The smear is hypocellular and contains a cohesive associated with proliferative breast epithelium. They are
epithelial group with cells having maintained low easily identified by modern mammography. Calcifica-
nuclear to cytoplasmic ratio. The nuclei are hypo- tions may be associated with fibrocystic changes, ductal
chromatic with prominent nucleoli; the cytoplasm is hyperplasia, ductal carcinoma in situ and invasive
abundant, dense with vacuoles. These are features of carcinomas.
radiation changes.

22. C 29. E
Note: The smear shows loosely cohesive epithelial Note: This smear is composed of loosely cohesive and
groups with highly atypical cells, some cells have a plasmacytoid cells showing nuclear hyperchromasia and
plasmacytoid appearance, a characteristic feature for an occasional mitotic figure (arrow). These are features
invasive ductal carcinoma of the breast. characteristic of invasive ductal carcinoma of the breast.

23. A 30. B
Note: The smear is cellular and shows typical features Note: This is a tightly cohesive epithelial group showing
of a fibroadenoma: large antler-shaped epithelial groups nuclear overlapping and focal nuclear streaming-features
(long arrow), stromal fragments (arrowhead), and bare of ductal hyperplasia without atypia. Ductal carcinoma
bipolar nuclei (short arrow) in the background. in situ or invasive ductal carcinoma is characterized by
loosely cohesive epithelial cells with marked cellular
24. C atypia.
Note: The smear shows a small loosely cohesive group
of malignant cells with large nuclei and prominent 31. E
nucleoli, admixed with lymphocytes. These features are Note: The smear shows multinucleated giant cells and
consistent with medullary carcinoma. macrophages consistent with previous surgery-related
healing changes. No malignant cells are seen to support
a diagnosis of carcinoma.
25. B
Note: The smear shows a necrotic cystic background and 32. A
obviously malignant cells. Note: The smear shows a nice lobular architecture
containing round and cohesive lobules, reminiscent of
26. C a terminal ductal-lobular unit (TDLU) in histology.
Note: The smear shows clusters and singly dispersed
tumor cells. Tumor cells have vacuolated cytoplasm. 33. B
Focal area of “single cell filing” of tumor cells is present Note: The smear shows a large cohesive epithelial group
which is characteristic of invasive lobular carcinoma. with significant nuclear overlapping. The cells are
Breast 93

loosely cohesive at the peripheral edges of the group, which Medullary carcinoma is characterized by highly
represents a feature seen in atypical ductal hyperplasia. The pleomorphic tumor cells admixed with lympho-
epithelial groups in a fibroadenoma arrange in a flat sheet and cytes.
usually have an antler configuration.
Fibroadenoma also has the stromal and myoepithelial 34. B
components. Tubular carcinoma’s cell groups have charac- Note: The cell at the arrow shows a single cyto-
teristic “garden hose” architecture. Invasive ductal carcinoma plasmic vacuole containing a targetoid mucin
has loosely cohesive and isolated malignant epithelial cells. inclusion— a feature of invasive lobular carcinoma.
94 Cytopathology Review

7
Thyroid
Rashna Madan

QUESTIONS
1. Fine needle aspiration (FNA) is indicated in the evaluation of which thyroid nodules?
A. All thyroid nodules
B. Nodules that are “hot” on radionuclide scan
C. Nodules detected on CT scans
D. Nodules detected by MRI scans
E. Nodules that show high activity on a PET scan

2. All the following statements regarding ultrasound guided FNA of the thyroid are true, EXCEPT:
A. It is the preferred approach for non-palpable nodules
B. It is the preferred approach for cystic nodules
C. It is the preferred approach to nodules that were non-diagnostic on previous FNA
D. It is a better technique than palpation guided FNA
E. It ensures that the nodule of interest is aspirated

3. An implied risk of malignancy of 5-15% would be expected of lesions in which of the following FNA
Bethesda diagnostic categories?
A. Suspicious for a follicular neoplasm
B. Suspicious for malignancy
C. Atypia of undetermined significance
D. Nondiagnostic
E. Benign
4. How soon should a fine needle aspiration be performed again following an FNA diagnosis of
“Unsatisfactory (nondiagnostic)”?
A. Immediately
B. 2 weeks
C. No sooner than 1 year
D. No sooner than 3 months
E. No sooner than 6 months

5. Multinucleated giant cells in thyroid aspirate smears may be seen in all of the following, EXCEPT:
A. Multinodular goiter
B. Follicular neoplasms
C. Papillary carcinoma
D. Anaplastic carcinoma
E. Subacute (De Quervain) thyroiditis
Thyroid 95

6. Which of the following statements applies to the thyroid FNA diagnosis of “Atypia of undetermined
significance (AUS)”, as listed in the Bethesda diagnostic categories?
A. This diagnosis may be used with any evidence of cellular atypia
B. This diagnosis may be used with any evidence of architectural atypia
C. This diagnosis should be limited to less than 10% of all thyroid aspirations
D. The implied risk of malignancy is 50%
E. The suggested follow-up management is lobectomy

7. Which of the following statements is true about subacute (de Quervain) thyroiditis?
A. It is a complication of pulmonary tuberculosis, typically affecting males
B. Fine needle aspiration is required for the diagnosis
C. It is characterized by granulomas on fine needle aspiration
D. It is characterized by lymphoid follicles on fine needle aspiration
E. It is characterized by abundant Hurthle cells on fine needle aspiration

8. All of the following features are usually seen in the FNAs of multinodular cystic goiter, EXCEPT:
A. Abundance of colloid
B. High cellularity
C. Hurthle cells
D. Follicular cells forming sheets
E. Hemosiderin-laden and foamy macrophages

9. Smears prepared from thyroid FNAs may show a predominant macrofollicular pattern in all the
following lesions, EXCEPT:
A. Colloid nodule
B. Adenomatoid nodule
C. Atypia of undetermined significance
D. Hurthle cell neoplasm
E. Papillary thyroid carcinoma

10. Which of the following statements is accurate regarding the diagnosis of “Suspicious for follicular
neoplasm, Hurthle cell type” as listed in the published Bethesda diagnostic categories?
A. The suggested management is repeat aspiration
B. Most lesions prove to be Hurthle cell adenomas
C. Abundant watery colloid is usually present
D. Abundant lymphocytes are usually present
E. Abundant Hurthle cells are usually arranged in flat sheets

11. Which of the following findings supports a diagnosis of follicular carcinoma rather than follicular
adenoma in fine needle aspiration material?
A. Detection of the translocation t (2;3)(q13;p25)
B. Increased proliferation rate detected immunohistochemically with MIB1
C. Positive galectin-3 immunostain
D. DNA polyploidy detected by flow cytometry
E. Abnormal morphometry and image analysis data
96 Cytopathology Review

12. Which of the following statements is true regarding smears prepared from thyroid FNA containing
abundant lymphoid cells?
A. Most primary thyroid lymphomas arise in the setting of Hashimoto thyroiditis
B. Primary thyroid lymphoma is more common than secondary involvement
C. The most frequent primary thyroid lymphoma is Hodgkin lymphoma
D. A clonal B-cell population on flow cytometry in the setting of Hashimoto’s thyroiditis is diagnostic of
lymphoma
E. A mixture of small and large lymphoid cells admixed with oncocytes on the aspirate smears favors
lymphoma

13. A patient presented with a thyroid mass and nodal metastases. A smear prepared from the FNA appeared
hypercellular, containing fairly uniform cells, mostly in crowded groups. Some cells were in form of
microfollicles and some were present as isolated cells. Mitoses were also noted. Few larger cells were
also present. The cells were thyroglobulin positive and a diagnosis of follicular neoplasm was made. A
different final diagnosis is rendered on resection where necrosis and mitoses are evident but
pleomorphism is absent. Which of the following is the correct final diagnosis?
A. Follicular carcinoma
B. Papillary carcinoma
C. Medullary carcinoma
D. Anaplastic carcinoma
E. Poorly differentiated carcinoma

14. A patient presented with a very firm, somewhat enlarged thyroid. An FNA was performed but the
aspirate contained no diagnostic cells or tissue. What is the most likely diagnosis?
A. Undifferentiated (anaplastic) carcinoma
B. Subacute thyroiditis
C. Primary thyroid lymphoma
D. Riedel thyroiditis
E. Graves disease

15. A 65 year old woman presents with hoarseness and a recently noticed firm, irregularly enlarged thyroid.
A diagnosis of undifferentiated (anaplastic) thyroid carcinoma is rendered on fine needle aspiration.
All of the following are cytologic features of undifferentiated (anaplastic) thyroid carcinoma, EXCEPT:
A. Abundant neutrophils and relatively few malignant cells
B. Paucicellular aspirate with rare overtly malignant cells
C. Spindled and squamoid cells
D. Absence of intranuclear pseudoinclusions
E. Pleomorphic and osteoclast-type multinucleated giant cells

16. A young adult patient with acne had a thyroid FNA and the smear prepared from the aspirated material
contained numerous dark brown granules within the cytoplasm of the follicular cells. This finding is
most likely a consequence of treatment with which of the following medications?
A. Tetracycline group antibiotics
B. Isoniazid
C. Iodine
D. Carbimazole
E. Sulfa drugs
Thyroid 97

17. A chronically ill patient presents with a sudden increase in size of the thyroid gland as well as difficulty
in breathing and swallowing. The fine needle aspiration shows colloid-like material with some mildly
contorted bland spindled nuclei. Which of the following stains or immunohistochemical reactions is
the best for confirming the presumptive diagnosis in this patient?
A. Pancytokeratin
B. Fontana
C. Calcitonin
D. Synaptophysin
E. Congo red

18. A 47-year-old female has an aspirate of a solitary asymptomatic “cold” thyroid nodule which displays
the expected nuclear features of a papillary thyroid carcinoma (including several grooves and
intranuclear inclusions) and psammoma bodies. Even though a papillary carcinoma was suspected, the
histologic examination of the resected thyroid proved that the lesion is a largely benign tumor. What is
the most likely diagnosis of this tumor?
A. Follicular adenoma
B. Hyalinizing trabecular tumor
C. Adenomatoid nodule
D. Hurthle cell adenoma
E. Cystic goiter

19. Which of the following thyroid region nodules could show intranuclear pseudoinclusions in FNA smears?
A. Parathyroid adenoma
B. Metastatic melanoma
C. Poorly differentiated thyroid carcinoma
D. Radiation associated atypia of the thyroid follicular cells
E. All of the above

20. A patient with a history of Graves’ disease had a thyroid FNA. The smear contained follicular cells
with obvious nuclear atypia comprising smudgy dark nuclei with anisonucleosis, occasional grooves
and nuclear pseudoinclusions and a relatively normal nuclear to cytoplasmic (N/C) ratio. The cells were
arranged in fairly cohesive orderly two-dimensional groups. What is the most likely diagnosis?
A. Radiation changes secondary to administration of I131
B. Follicular carcinoma
C. Papillary carcinoma
D. Undifferentiated (anaplastic) carcinoma
E. Metastatic carcinoma

21. Immunohistochemical evaluation of the cell-block material obtained by thyroid FNA is helpful in
establishing all the following diagnoses, EXCEPT:
A. Medullary thyroid carcinoma
B. Undifferentiated (anaplastic) thyroid carcinoma
C. Large cell type of primary thyroid lymphoma
D. Papillary thyroid carcinoma
E. Metastatic carcinoma
98 Cytopathology Review

22. Molecular alterations that may be found in


association with this thyroid aspirate include all of
the following, EXCEPT:
A. RET/PTC rearrangements
B. BRAF mutations
C. Point mutations of the RAS gene
D. Rearrangements of the TRK gene
E. PAX8-PPARψ fusion

23. This smear was prepared from a thyroid FNA of a


young adult patient who had a prior history of
pheochromocytoma. What is the most likely
diagnosis?
A. Medullary thyroid carcinoma
B. Papillary thyroid carcinoma
C. Follicular neoplasm
D. Follicular neoplasm, Hurthle cell type
E. Metastatic carcinoma

24. A 40-year-old female had a thyroid FNA and the


smear prepared from it is shown here. Which of the
following statements is true in regard to this aspirate?
A. The presence of abundant colloid excludes a
papillary thyroid carcinoma
B. The lack of papillary architecture excludes a
papillary carcinoma
C. The absence of psammoma bodies excludes a
papillary carcinoma
D. Cystic changes if present, indicates a papillary
carcinoma
E. Focal microfollicular architecture if present, does not
alter the diagnosis
Thyroid 99

25. This smear was prepared from an FNA of a cold


thyroid nodule noticed in a 40-year-old woman.
Which of the following statements is accurate with
respect to this aspirate?
A. The finding of microfollicles is diagnostic of
follicular neoplasm
B. The finding of cystic changes is diagnostic of nodular
goiter
C. The finding of intranuclear pseudoinclusions is
diagnostic of papillary carcinoma
D. Microfollicles may be seen in papillary carcinoma
E. Intranuclear pseudoinclusions are commonly seen in
follicular adenoma

26. This smear was prepared from the fine needle


aspiration of an enlarged nodular thyroid of a 36-
year-old woman. What is the best diagnosis?
A. Nondiagnostic/unsatisfactory
B. Multinodular goiter with cystic degeneration
C. Atypia of undetermined significance
D. Follicular neoplasm with cystic degeneration
E. Papillary carcinoma with cystic degeneration

27. This smear was prepared from the FNA of a thyroid


nodule of a 40-year-old woman. What would be the
most appropriate follow-up management?
A. Repeat FNA
B. Clinical follow-up
C. Chemoradiation
D. Lobectomy
E. Total thyroidectomy
100 Cytopathology Review

28. This smear was prepared from the fine needle


aspiration of a thyroid mass of a 60-year-old male,
who presented with a rapidly enlarging thyroid mass.
Which of the following statements regarding this FNA
is true?
A. Clinical and imaging findings are of little help in
establishing the diagnosis
B. The presence of cytokeratin positivity and thyroid
transcription factor 1 (TTF-1)/thyroglobulin
negativity establishes the diagnosis of metastatic
carcinoma
C. The presence of vimentin positivity and thyroid
transcription factor 1 (TTF-1)/thyroglobulin
negativity establishes the diagnosis of sarcoma
D. Positive staining for TTF-1 and/or thyroglobulin is
frequently useful to confirm the diagnosis of
undifferentiated (anaplastic) thyroid carcinoma
E. Evaluation usually includes melanoma and lymphoid
markers

29. This smear was prepared from the FNA of a thyroid


nodule in a 52-year-old woman. What is the most
appropriate initial management for this patient?
A. Hemithyroidectomy
B. Repeat FNA
C. Radiation
D. I131 therapy
E. Near total thyroidectomy

30. This smear was prepared from the FNA of a thyroid


nodule in a 44-year-old woman. What is the best
diagnosis?
A. Benign follicular nodule
B. Atypia of undetermined significance
C. Suspicious for follicular neoplasm
D. Suspicious for papillary carcinoma (follicular
variant)
E. Suspicious for parathyroid neoplasm
Thyroid 101

31. This smear was prepared from the fine needle


aspiration of the thyroid of a 35-year-old woman.
What is the most likely clinical presentation of this
lesion?
A. Painful enlargement of the thyroid gland that
responds well to non-steroidal anti-inflammatory
agents
B. Euthyroid goiter
C. Persistent hyperthyroidism resistant to treatment
D. Compressive symptoms including dysphagia and
hoarseness of voice
E. Asymptomatic with circulating antibodies to thyroid
peroxidase

32. This smear was prepared from the fine needle


aspiration of the thyroid of a 30-year-old woman.
What would be the appropriate management for this
patient?
A. Repeat FNA
B. Repeat FNA with ultrasound guidance
C. Clinical follow-up
D. Lobectomy
E. Thyroidectomy

33. This smear was prepared from the FNA of the thyroid
of a 48-year-old woman. All of the following are
included in the differential diagnoses, EXCEPT:
A. Adenomatoid nodule
B. Follicular neoplasm
C. Hurthle cell neoplasm
D. Papillary carcinoma
E. Parathyroid adenoma
102 Cytopathology Review

34. Which of the following is the approximate 5 year


survival for a patient with this thyroid aspirate?
A. 0-15%
B. 25%
C. 30-40%
D. 50%
E. 80%

35. This smear is prepared from a thyroid FNA from


which of the following patients?
A. An adult with a multinodular neck mass
B. An elderly patient with a rapidly enlarging neck mass
and recent development of a hoarse voice
C. A young adult with a history of radiation exposure
D. A middle-aged female with hyperthyroidism
E. A middle-aged female with a diffusely enlarged
thyroid

36. A 50-year-old adult without a past history of


neoplasia has a fine needle aspirate of a thyroid mass
in the middle third of lobe from which a cell block is
prepared. Which of the following markers is the best
to confirm the diagnosis?
A. Thyroid transcription factor 1 (TTF-1)
B. Thyroglobulin
C. Pancytokeratin
D. Vimentin
E. Calcitonin
Thyroid 103

37. A middle aged female had a diffusely enlarged


thyroid with a focal nodular prominence. Based on
the findings in the aspirate, what is the most
appropriate diagnosis?
A. Atypia of undetermined significance
B. Benign – consistent with lymphocytic (Hashimoto)
thyroiditis
C. Suspicious for a Hurthle cell neoplasm
D. Suspicious for lymphoma
E. Suspicious for metastatic carcinoma

38. What is the most appropriate Bethesda terminology


for this thyroid aspirate?
A. Benign—consistent with lymphocytic (Hashimoto)
thyroiditis
B. Suspicious for follicular neoplasm
C. Suspicious for follicular neoplasm, Hurthle cell type
D. Suspicious for papillary carcinoma
E. Suspicious for malignancy

39. This smear prepared from a thyroid FNA was


obtained from a patient with a clinical history of oral
squamous cell carcinoma. Immunohistochemical
stains on the cellblock sections showed that the cells
are positive for thyroid transcription factor 1 (TTF-
1) and chromogranin. What is the correct diagnosis?
A. Metastatic squamous cell carcinoma
B. Metastatic small cell carcinoma
C. Medullary carcinoma
D. Papillary carcinoma
E. Undifferentiated (anaplastic) carcinoma
104 Cytopathology Review

40. This smear was prepared from an FNA of a left


thyroid nodule in a 54-year-old woman. All smears
showed the same findings. What is the correct
interpretation?
A. Nondiagnostic specimen-blood only
B. Nondiagnostic specimen-cyst fluid only
C. Benign-consistent with a colloid nodule
D. Benign-consistent with amyloidosis
E. Benign-consistent with Graves disease
Thyroid 105

ANSWERS

1. E degenerative changes. The implied risk of malignancy


Note: Strongly PET positive nodules are more likely to for AUS is only 5-15%. A repeat aspirate and assessment
be malignant than other nodules and should be aspirated is usually suggested in view of the clinical findings. Only
by fine needle. The risk of malignancy in such nodules 20-25% of nodules retain the designation of AUS after
has been estimated to be 14-50%. Nodules found by CT a second FNA.
or MRI should be further assessed by ultrasound and if
larger than 1–1.5 cm should also undergo FNA. Nodules 7. C
measuring less than 1 cm that are not worrisome on Note: Subacute thyroiditis is a self-limited granulo-
ultrasound need not be aspirated. However, if they have matous disease of sudden onset, which is 3-5 times more
concerning findings, such as microcalcifications, then common in women than in men. Granulomas with
an FNA should be preformed. prominent multinucleated giant cells are typically found
in fine needle aspirations. This disease is thought to be
2. D caused by viruses and it is not related to infections with
Note: The actual technique for ultrasound-guided FNA Mycobacteria. The clinical findings (fever, neck pain,
is similar to the one used in palpation FNA and per se tenderness of the thyroid) are typical and the thyroid is
it is not any better that the latter technique. Ultrasound aspirated only if the clinical findings are unusual (e.g.
guided FNAs are preferred for non-palpable or cystic a prominent single nodule). The cytologic findings vary
nodules and owing to visualization ensure that the depending on the stage of the disease. In the beginning,
nodule/area of interest is the one sampled. there is non-specific acute inflammation followed by
chronic inflammation with granulomas and multinuc-
3. C leated giant cells containing colloid.
Note: The implied risks of malignancy for the following
Bethesda diagnostic categories are as follows: Atypia 8. B
of undetermined significance–5-15%; Suspicious for Note: Fine needle aspiration of goiters is typically
a follicular neoplasm–15-30%; Suspicious for characterized by abundance of colloid and low cellula-
malignancy–60-75%; Malignant–97-99%. rity. Follicular cells, if present, form sheets and also may
show Hurthle cell metaplasia. Hemosiderin-laden and
4. D foamy macrophages may also be present especially in
Note: Re-aspirating sooner than 3 months may result in cystic lesions, most likely resulting from previous
an inaccurate diagnosis as a consequence of atypia from hemorrhage.
repair or reactive changes. A repeat fine needle
aspiration will produce an adequate diagnosis in as many 9. D
as 60% of all cases. Note: Hurthle cell neoplasm is characterized by almost
exclusive Hurthle cells dispersed predominantly as
5. B isolated cells; macrofollicles are not a feature. Macro-
Note: Multinucleated cells are not usually seen in follicles are the predominant pattern seen in FNAs of
follicular neoplasms, but can be seen in the smears of benign follicular nodules which comprise the
FNA material obtained from all other lesions listed here. adenomatoid nodules or colloid nodules of multinodular
goiter. Macrofollicular patterns consistent of flat/ two-
6. C dimensional sheets of regularly spaced (honeycomb-
Note: According to the published Bethesda diagnostic like), fairly uniform follicular cells that lack nuclear
categories, the diagnosis of AUS should be restricted features of papillary thyroid carcinoma. Colloid is
to appropriate cytologic scenarios. Mild cytologic atypia usually abundant but can be watery and not well
is consistent with a benign diagnosis in several appreciated and in a few cases of benign follicular
situations, such as Hurthle cells in lymphocytic nodules, the smears may be moderately cellular with
thyroiditis, follicular cells post-radio-iodine therapy, and only scant colloid. The diagnosis of atypia of
cells lining cysts in benign follicular nodules with undetermined significance may be made when there are
106 Cytopathology Review

atypical nuclear features in the cells forming macrofolli- combined with oncocytes and follicular cells favors
cles. Papillary thyroid carcinoma may show syncytial- Hashimoto thyroiditis.
like monolayers similar to that of macrofollicles.
13. E
10. B Note: Poorly differentiated carcinomas lie between the
Note: As stated in the published Bethesda diagnostic differentiated thyroid carcinomas (follicular or papillary)
categories, the suggested management for the diagnosis and undifferentiated carcinomas. They are not readily
of “Suspicious for follicular neoplasm, Hurthle cell type” diagnosed on fine needle aspirations, and often resemble
is surgical lobectomy. Approximately 10-25% of thyroid follicular neoplasms or metastatic carcinomas. In com-
nodules with this diagnosis are non-neoplastic parison to follicular neoplasms, poorly differentiated
representing either goiters or Hashimoto thyroiditis. The thyroid carcinomas have a monomorphic appearance at
remaining lesions are neoplasms, most of which (55- low power due to the high nuclear/cytoplasmic ratio of the
85%) are classified as adenomas. Dissociated or single tumor cells. Tumor cells are usually arranged in insular
cells are a frequent finding in Hurthle cell neoplasms or trabecular groups. Mitoses and foci of necrosis are often
though scattered cell clusters can also be seen. Abundant present and are better appreciated in resection specimens.
lymphocytes indicate Hashimoto thyroiditis. There is
usually little or no colloid. 14. D
Note: Riedel thyroiditis, an uncommon disease results
11. A in marked fibrosis of thyroid which extends into
Note: The presence of capsular and/or vascular invasion extrathyroidal tissue and results in a hard gland and not
distinguishes follicular carcinomas from adenomas in infrequently a “dry” FNA. The clinical presentation may
surgical resection material. This distinction is usually in some ways mimic anaplastic thyroid carcinoma but
not feasible on cytologic assessment. The only reliable malignant cells are not appreciated on aspiration.
discriminator is the presence of the translocation Hashimoto thyroiditis may be fibrosing but Hurthle cell
t (2;3)(q13;p25) in follicular carcinomas that produces and lymphoid populations will usually be present.
a PAX8-PPARψ fusion. However when detected by
fluorescence in situ hybridization (FISH), this test has 15. D
a low sensitivity (approximately 26%). Note: Intranuclear inclusions (INCIs) may be seen in
undifferentiated carcinomas, which also show other
12. A morphologic features listed in this question. The diagnosis
Note: The majority of primary thyroid lymphomas are of papillary thyroid carcinoma requires an identification
either diffuse large B-cell lymphoma or extranodal of all the typical features of papillary carcinoma and should
marginal zone B-cell lymphomas of mucosa-associated not be based on finding of INCIs alone.
lymphoid tissue (MALT), also known as MALTomas.
Most arise in the setting of Hashimoto’s thyroiditis. 16. A
Identifying the large monotonous population of diffuse Note: Prolonged therapy with minocycline or other
B-cell lymphoma in smears can be accomplished without members of the tetracycline group may result in pigmen-
much difficulty. Secondary involvement of the thyroid tation of the thyroid (“black thyroid”). The pigmented
by lymphoma is, nevertheless, more common than granules react positively with the Fontana stain.
primary thyroid lymphoma. On the other hand, detecting
an extranodal marginal zone lymphoma can be 17. E
challenging on morphologic grounds alone and here Note: This is a description of amyloid goiter which may
immunophenotyping may help. However, a clonal have an acute presentation in the neck. Typically this
population on flow cytometry may also be detected in occurs in the background of a long-standing illness
Hashimoto thyroiditis and is not by itself diagnostic of which leads to systemic amyloidosis. The amyloid
lymphoma. Therefore, flow cytometry should be done resembles colloid in smears but the former may also
only if the nodule is clinically suspicious or if the show fibroblast nuclei. The amyloid will show apple-
cytology is abnormal (atypical or monotonous lymphoid green birefringence under polarized light when stained
population without admixed oncocytes or follicular with Congo red. The malignant cells of medullary
cells). A mixture of small and large lymphocytes carcinoma will not be present.
Thyroid 107

18. B 23. A
Note: Hyalinizing trabecular tumor (HTT) may appear Note: The dispersed malignant cells with variable
cytologically identical to papillary carcinoma on cytologic morphology including occasional plasmacytoid
assessment. Theoretically amyloid-like hyaline material and forms and coarsely granular chromatin, coupled
cytoplasmic paranuclear yellow bodies indicate HTT but these with the presence of acellular material (amyloid)
are rarely appreciated. Distinction on resection is straight- indicate a medullary thyroid carcinoma. Pheochro-
forward with HTTs displaying encapsulation, trabecular mocytomas are a part of multiple endocrine
architecture and hyalinization. RET/PTC rearrangements may neoplasia (MEN) 2, a syndrome which also
be present in both HTT and papillary carcinoma. HTTs for includes medullary thyroid carcinoma.
the most part behave as benign tumors though rare exceptions
to this general rule have been reported. 24. E
Note: The presence of abundant colloid and
19. E macrofollicular architecture is consistent with a
Note: Intranuclear pseudoinclusions may be seen in several benign follicular nodule. A minor component of
tumors that involve the thyroid including those mentioned as microfollicles may be seen with this diagnosis.
well as papillary thyroid carcinoma, medullary thyroid While papillary carcinomas often have psammoma
carcinoma and undifferentiated thyroid carcinoma. Therefore, bodies and may have papillary architecture or have
it must be emphasized that a single nuclear feature such as plentiful colloid that diagnosis is based on the
intranuclear pseudoinclusions is not specific for a diagnosis presence of several characteristic nuclear changes
of papillary thyroid carcinoma. including enlargement, nuclear grooves, intra-
nuclear inclusions, pale powdery chromatin,
20. A irregular nuclear membranes and small nucleoli.
Note: The aforementioned changes are typically a consequence Both benign follicular nodules and papillary
of radiation therapy, including external radiation and carcinoma may be cystic.
administration of radioactive iodine I131. Determining this
history is very useful in rendering the above diagnosis. The 25. D
preserved N/C ratio despite the overall cellular enlargement Note: While the smear shows follicular architec-
and the dark smudgy nuclear chromatin are highly suggestive ture suggestive of a follicular neoplasm, there is an
of radiation. Additionally the microfollicular pattern of intranuclear pseudoinclusion raising the possibility
follicular neoplasms and the single cell pattern of clearly of the follicular variant of papillary thyroid
malignant cells of undifferentiated (anaplastic) carcinoma are carcinoma. Assessment of both Romanowsky and
both absent. There may be a few grooves/nuclear inclusions Papanicolaou stained smears including a search
raising the possibility of papillary carcinoma but the dark for nuclear features of papillary thyroid carcinoma
smudgy chromatin argues against this diagnosis. may elucidate whether this is a papillary carcinoma
or a follicular neoplasm. Intranuclear pseudoin-
21. D clusions alone are not specific for papillary thyroid
Note: Immunostains are not helpful in establishing the carcinoma because they may be seen in medullary
diagnosis of papillary thyroid carcinoma, which is best carcinoma and poorly differentiated thyroid
diagnosed on the basis of typical cytomorphologic features. carcinoma, and very rarely benign thyroid nodules
In all of the other diagnoses listed in this question, including follicular adenoma. Microfollicles, flat
immunostains are useful ancillary tools and are typically used sheets or papillae may be seen in papillary
in conjunction with clinical and radiological findings. carcinoma. Cystic changes are non-specific findings
in thyroid aspirations and may be associated with
22. E nodular goiter or papillary thyroid carcinoma.
Note: This smear shows the typical features of papillary
carcinoma of the thyroid, which may have the genetic 26. A
alterations mentioned in the answers A through D, except for Note: Only numerous macrophages are seen in this
the translocation t (2;3)(q13;p25) which results in the PAX8- smear. According to the Bethesda system, the
PPARψ fusion. The latter change is characteristic of follicular requirements for an adequate thyroid aspirate
carcinoma. include six or more groups of well-visualized
108 Cytopathology Review

follicular cells, with each group containing 10 or more follicular and overlapping. They may contain a central
cells. Exceptions to this rule include the presence of plentiful droplet of colloid. It has been suggested that
thick colloid (which indicates a benign colloid nodule) and microfollicles consisting of less than 15 cells and
prominent inflammation (which may be seen in benign form two-third or more of a circle. The presence
inflammatory conditions such as lymphocytic thyroiditis, of mostly microfollicular pattern with scant or no
granulomatous thyroiditis or even an abscess). A third exception colloid is suspicious for a follicular neoplasm. The
is the presence of follicular cells with distinct cytologic atypia, follicular cells may also be arranged in trabeculae.
which be reported regardless of how few the follicular cells are. Distinguishing a follicular adenoma from a
Macrophages indicate cystic change which may be seen in benign carcinoma requires identification of capsular or
follicular nodules as well as in papillary carcinoma. If adequate vascular invasion in the latter. This can only be
colloid and/or follicular cells are not present, it is not possible assessed in histologic sections prepared from the
to determine the underlying lesion or tumor. resection specimens. It is often not feasible to
cytologically distinguish parathyroid cells from
27. D thyroid follicular cells in FNA material. A variable
Note: The image shows a microfollicular pattern that would proportion of patients with this diagnosis will
appropriately be diagnosed as suspicious for a follicular ultimately have papillary carcinoma follicular
neoplasm. As resection is required to distinguish a benign variant diagnosed on resection. However, in the
adenoma from a carcinoma, the next recommended step usually absence of cytologically recognizable nuclear
includes lobectomy or hemithyroidectomy. features of papillary thyroid carcinoma, the
diagnosis “suspicious for follicular neoplasm” is
28. E more appropriate.
Note: A thyroid mass in an older patient with a cellular aspirate
composed of singly dispersed malignant cells, including 31. A
apoptotic cells, raises the differential diagnosis of undifferen- Note: The image demonstrates a granuloma com-
tiated (anaplastic) thyroid carcinoma, lymphoma, melanoma, posed of epithelioid histiocytes with elongated
metastatic carcinoma, sarcoma and medullary thyroid nuclei that are clustered together. Granulomas
carcinoma. Establishing the appropriate diagnosis requires along with multinucleated cells that may contain
correlation of clinical and imaging findings, cytomorphology colloid and lymphocytes are characteristic of
and immunostains. If the mass is located in the thyroid, the subacute (de Quervain) thyroiditis. This disease
cells are cytokeratin positive and there is no clinico- typically shows a self-limited course. Granulomas
radiological evidence of another primary, the diagnosis is most may also be seen in other conditions such as
likely undifferentiated (anaplastic) carcinoma. Vimentin is also sarcoidosis or tuberculosis.
often positive but it is uncommon to find positive staining for
thyroid transcription factor 1 (TTF-1) and thyroglobulin in 32. C
undifferentiated (anaplastic) thyroid carcinoma. Note: The aspirate shows profuse colloid consis-
tent with a benign follicular nodule (colloid nodule).
29. E The implied risk of malignancy is low (in the range
Note: The image demonstrates diagnostic features of papillary of 0-3%) and the typical management includes
thyroid carcinoma including nuclei with fine powdery clinical follow-up at 6-18 month intervals for 3-5
chromatin, longitudinal grooves, intranuclear pseudoinclusions years. However, if the nodule enlarges significantly
and irregular nuclear membranes. The cells are arranged in or the subsequent ultrasound shows “concerning
flat two dimensional sheets with focal nuclear crowding and findings”, a repeat FNA should be considered.
overlapping. Papillary carcinoma diagnosed on FNA is
managed by a total or near-total thyroidectomy. A hemithyroi- 33. C
dectomy has more risk for tumor recurrence because these Note: The smear shows scattered microfollicles
carcinomas can be multifocal. and absence of colloid, therefore, suspicious for
a follicular neoplasm. Differential diagnoses
30. C mainly include adenomatoid nodule and follicular
Note: The image shows several microfollicles which are neoplasm. A cellular adenomatoid nodule is
follicular groups that are smaller than normal with crowding distinguished from a follicular adenoma by the
Thyroid 109

presence of a capsule in the latter; this assessment can Hurthle cells is common in the setting of lymphocytic
only be done on resection. Papillary carcinoma thyroiditis. This includes nuclear size variation and
(follicular variant) and parathyroid adenoma also enter degenerative nuclear changes where the nuclei appear
the differential diagnoses from the low-power large and dark. Hurthle cell neoplasms lack the lymphoid
architectural pattern. Hurthle cell neoplasm, on the other component of lymphocytic thyroiditis and often contain
hand, is characterized by a singly dispersed pattern of isolated Hurthle cells. A monomorphic small to medium
oncocytic cells which are not admixed with other cells. sized lymphoid population or the presence of large
malignant appearing lymphoid cells would be
34. A concerning for lymphoma. Metastatic carcinoma would
Note: The aspirate shows clearly malignant cells present with atypical malignant cells clearly representing
including several apoptotic cells, compatible with the a foreign cell population.
diagnosis of undifferentiated (anaplastic) carcinoma.
The patients with undifferentiated thyroid tumors have 38. C
an abysmal prognosis, with a 5-year survival in the range Note: This exclusive population of Hurthle cells,
of only 0-14%. The patients with poorly differentiated including several single cells, without a lymphoid
thyroid carcinomas have an approximate 5-year survival background is suspicious for a follicular neoplasm,
of 50%, and those with medullary thyroid carcinomas Hurthle cell type. Separating an adenoma from a
have an average 5-year survival rate of 83%. carcinoma requires resection and assessment for capsular
and vascular invasion which is present in the latter. The
35. E nuclear features of papillary thyroid carcinoma are not
Note: The smear shows a background lymphoid appreciated, though the oncocytic variant of papillary
population that is polymorphous with both small and carcinoma may have similarly abundant granular
large cells as well as an aggregate of Hurthle cells. cytoplasm.
Hurthle cells are characterized by abundant granular
cytoplasm and variably enlarged nuclei with distinct 39. C
nucleoli. This combination of Hurthle cells and a mixed Note: The smear shows discohesive spindled and
lymphoid population is seen in lymphocytic (Hashimoto) plasmacytoid malignant cells, these are characteristic
thyroiditis. Serologic workup typically reveals antibo- features of medullary thyroid carcinoma (MTC).
dies to thyroglobulin or thyroperoxidase. The condition Positivity for TTF1 and chromogranin confirm the
classically affects middle-aged females who develop diagnosis. The cells have too much cytoplasm to be
diffuse enlargement of the gland though focal nodularity small cell carcinoma. MTC is usually sporadic but in
may be seen. the minority of cases is part of multiple endocrine
neoplasia (MEN) 2A or 2B. Both MEN1 and MEN2
36. E patients have MTC and pheochromocytomas, but the
Note: The cell block shows malignant spindled cells as MEN 2B patients also have mucosal neuromas, and a
well as acellular material consistent with amyloid. These marfanoid habitus.
findings are highly suspicious for medullary thyroid
carcinoma. Medullary carcinoma typically stains for 40. C
TTF-1, calcitonin, carcinoembryonic antigen and Note: On Pap-stain, watery colloid has a light green-
neuroendocrine markers such as chromogranin and pink color and a shiny thin membranous appearance with
synaptophysin. Thyroglobulin is negative. Positive folds and bubbles as shown here. It may be confused
staining for calcitonin or the combination of a thyroid with serum in a bloody smear. Serum is very thin without
specific marker (TTF-1) and a neuroendocrine marker any folding or cracking and does not have a shiny
confirms the diagnosis. appearance and tends to accumulate at the edges of the
slide. Amyloid resembles dense colloid and has a thick
37. B waxy appearance. When abundant colloid predominates
Note: These findings are most consistent with in a smear, a diagnosis of a colloid nodule can be made
lymphocytic (Hashimoto) thyroiditis. Mild atypia of even if the smear contains few or no follicular cells.
110 Cytopathology Review

8
Salivary Gland
Maura O’ Neil

QUESTIONS
1. Which of the following statements is true regarding fine needle aspiration (FNA) of salivary gland
masses?
A. FNA is associated with a lower risk of infection when compared to incisional biopsy
B. FNA is not associated with any risk of bleeding
C. FNA findings always help avoid unnecessary excisions
D. FNA rarely lead to false negative diagnoses in cystic salivary gland lesions
E. FNA can reliably distinguish between primary lymphoma and a normal intraparotid lymph node based on
morphologic characteristics alone

2. The epidemiological features of salivary gland neoplasms can be helpful when evaluating salivary gland
neoplasms. Which statement regarding salivary gland neoplasms is true?
A. Salivary gland neoplasms are more common in men
B. Most parotid gland tumors are benign
C. Polymorphous low-grade adenocarcinoma occurs most commonly in the parotid gland
D. Pleomorphic adenoma occurs almost exclusively in the minor salivary glands
E. Most sublingual gland tumors are benign

3. Which of the following is the most common salivary gland neoplasm?


A. Warthin tumor
B. Pleomorphic adenoma
C. Mucoepidermoid carcinoma
D. Oncocytoma
E. Basal cell adenoma

4. Which of the following is the most common salivary gland neoplasm in infants?
A. Pleomorphic adenoma
B. Mucoepidermoid carcinoma
C. Polymorphous low grade adenocarcinoma
D. Hemangioma
E. Adenoid cystic carcinoma

5. Which of the following is the most common malignant neoplasm of the parotid gland?
A. Mucoepidermoid carcinoma
B. Acinic cell carcinoma
C. Adenoid cystic carcinoma
D. Carcinoma ex pleomorphic adenoma
E. Polymorphous low grade adenocarcinoma
Salivary Gland 111

6. A 45-year-old woman presented with bilateral parotid swelling without a discrete mass. Fine needle
aspiration yielded plenty of normal salivary gland acini and small ducts adherent to thin fibrovascular
stroma. There were no inflammatory cells. What is the correct interpretation?
A. Sialadenosis
B. Sialadenitis
C. Warthin’s tumor
D. Oncocytoma
E. Acinic cell carcinoma

7. When differentiating non-neoplastic salivary gland cysts from cystic neoplasms on fine needle aspirations,
which of the following features favors a non-neoplastic cyst?
A. Numerous mucinophages
B. Scant epithelial cells
C. Abundant crystals
D. Serous fluid contents
E. Complete disappearance of the cyst following aspiration
8. Patients with lymphoepithelial sialadenitis and/or Sjögren syndrome are at an increased risk of
developing which malignancy?
A. Adenoid cystic carcinoma
B. Non-Hodgkin lymphoma
C. Acinic cell carcinoma
D. Mucoepidermoid carcinoma
E. Squamous cell carcinoma

9. Which of the following is a cytologic feature for acinic cell carcinoma, but not for Warthin tumor?
A. Epithelial groups and numerous lymphocytes
B. Cystic background
C. Abundant stripped nuclei in the background
D. Epithelial cells with abundant finely granular cytoplasm
E. Epithelial cells with bland nuclei

10. The cytoplasm of acinic cell carcinoma contains which of the following elements?
A. Zymogen granules
B. Mitochondria
C. Lysosome
D. Intermediate filaments
E. Glycogen
11. An encapsulated firm 3 cm tumor of the hard palate was biopsied in a 50-year-old woman. The aspiration
smear showed tubules, sheets and clusters of polygonal and oval epithelial cells. The tumor cells showed
minimal nuclear atypia. What is the most likely diagnosis?
A. Acinic cell carcinoma
B. Pleomorphic adenoma
C. Polymorphous low-grade adenocarcinoma
D. Mucoepidermoid carcinoma
E. Warthin tumor
112 Cytopathology Review

12. What is the most common metastatic tumor to the parotid gland?
A. Small cell carcinoma
B. Squamous cell carcinoma
C. Adenocarcinoma
D. Melanoma
E. Papillary thyroid carcinoma

13. A 32-year-old HIV-positive male develops bilateral parotid masses. An FNA performed on one of the
masses showed a cellular aspirate with histiocytes, keratin debris and anucleated squamous cells, tingle-
body macrophages and epithelial cell clusters with interspersed lymphocytes. What is the most likely
diagnosis?
A. Intraparotid lymph node
B. Warthin tumor
C. Acinic cell carcinoma
D. Cystic lymphoepithelial lesion
E. Cystic metastatic squamous cell carcinoma

14. The groups of epithelial cells indicated with the


arrows represent which of the following?
A. Adipocytes
B. Ductal cells
C. Lymphocytes
D. Acinar cells
E. Mucus cells

15. The lesion shown and the structures indicated by the


arrows represent which of the following?
A. Amylase crystalloids in a non-neoplastic parotid cyst
B. Sialolithiasis with acute sialadenitis
C. Tyrosine crystalloids in a pleomorphic adenoma
D. Diastase-resistant cytoplasmic zymogen granules in
acinic cell carcinoma
E. Psammoma bodies in metastatic papillary thyroid
carcinoma
Salivary Gland 113

16. Which of the following lesions is the most likely source


of the cells illustrated in this fine needle aspirate from
a parotid gland lesion?
A. Benign squamous epithelial-lined cyst
B. Warthin tumor
C. Acinic cell carcinoma
D. Mucoepidermoid carcinoma
E. Cystic metastatic squamous cell carcinoma

17. This fine needle aspirate was most likely obtained


from which anatomic site?
A. Sublingual gland
B. Submandibular gland
C. Thyroid gland
D. Parotid gland
E. Oral cavity minor salivary gland

18. The cytologic differential diagnosis for the lesion


shown includes all of the following, EXCEPT:
A. Basal cell adenocarcinoma
B. Acinic cell carcinoma
C. Basal cell adenoma
D. Adenoid cystic carcinoma, solid variant
E. Cellular pleomorphic adenoma
114 Cytopathology Review

19. This slide shows a fine needle aspirate from the


parotid of a 68-year-old man. What is the most likely
diagnosis?
A. Squamous cell carcinoma
B. Plasmacytoma
C. Basal cell neoplasm
D. Warthin tumor
E. Myoepithelioma

20. This fine needle aspiration was obtained from a


2.0 cm parotid gland mass in a 69-year-old woman.
None of the slides prepared contained chondromyxoid
matrix material. What is the best interpretation of
this FNA?
A. Squamous cell carcinoma
B. Basal cell adenoma
C. Mucoepidermoid carcinoma
D. Acinic cell carcinoma
E. Epithelial-myoepithelial carcinoma

21. Which of the following statements is true regarding


the lesion shown?
A. Squamous and mucinous metaplasia are not features
of this lesion and should raise concern for muco-
epidermoid carcinoma
B. It is a rare salivary gland neoplasm, comprising
approximately 5-10% of salivary gland neoplasms
C. Focal mild cytologic atypia can be seen in this lesion
D. It occurs almost exclusively in the submandibular
gland
E. It does not occur in children
Salivary Gland 115

22. The tumor shown is characterized by which type of


cells?
A. Large polygonal cells with PAS+ granules
B. Oncocytic epithelial cells and lymphocytes
C. Small monotonous basaloid cells
D. Mucus cells, epidermoid (squamous) cells and
intermediate cells
E. High grade nuclei with prominent nucleoli resemb-
ling comedo ductal carcinoma in situ of the breast

23. This FNA is from an 80-year-old male with a discrete,


well-circumscribed nodule in the parotid gland. The
smears showed a cellular specimen with a clean
background and no lymphocytes. What is the
diagnosis?
A. Warthin tumor
B. Acinic cell carcinoma
C. Mucoepidermoid carcinoma
D. Pleomorphic adenoma
E. Oncocytoma

24. This FNA is from a solid mass in a sublingual gland.


What is the best diagnosis?
A. Warthin tumor
B. Acinic cell carcinoma
C. Mucoepidermoid carcinoma
D. Pleomorphic adenoma
E. Oncocytic carcinoma
116 Cytopathology Review

25. Which of the following statements is true regarding


the lesion shown in this figure?
A. It is associated with rapid aggressive growth
B. It can occur in salivary glands, lacrimal and sweat
glands, respiratory tract and breast
C. It contains basement membrane material which
appears fibrillary on Diff-Quik stained smears
D. It is negative or shows very weak immunoreactivity
for CD117 (c-kit)
E. It is characterized by marked nuclear pleomorphism,
brisk mitotic activity and prominent apoptosis

26. All of the following are true statements about the


lesion shown, EXCEPT:
A. It occurs almost exclusively in the parotid gland
B. It is often associated with a history of smoking
C. It is the most common bilateral salivary gland
neoplasm
D. Most cases present in young adulthood
E. It feels “doughy” on palpation and aspiration

27. The differential diagnosis for the salivary gland FNA


shown includes all of the following, EXCEPT:
A. Acinic cell carcinoma
B. Mucoepidermoid carcinoma, high grade
C. Salivary duct carcinoma
D. Carcinoma ex pleomorphic adenoma
E. Metastatic adenocarcinoma
Salivary Gland 117

28. A 45-year-old woman presented with a 4-year history


of a parotid gland mass that had enlarged in last
several months. A fine needle aspiration was
performed. Based on the cytologic findings shown,
what is the diagnosis?
A. Mucoepidermoid carcinoma
B. Adenoid cystic carcinoma
C. Acinic cell carcinoma
D. Pleomorphic adenoma
E. Basaloid neoplasm

29. A 62-year-old woman presented with a painful


submandibular mass. A Diff-Quik stained smear
obtained by FNA of the parotid gland is shown here.
What is the most likely diagnosis?
A. Mucoepidermoid carcinoma
B. Adenoid cystic carcinoma
C. Acinic cell carcinoma
D. Pleomorphic adenoma
E. Basaloid neoplasm

30. This FNA is from a 5 cm parotid gland mass. What


is the correct diagnosis?
A. Mucoepidermoid carcinoma
B. Adenoid cystic carcinoma
C. Acinic cell carcinoma
D. Pleomorphic adenoma
E. Basaloid neoplasm
118 Cytopathology Review

31. The tumor shown is from a fine needle aspiration of


parotid gland. What is the most likely diagnosis?
A. Warthin tumor
B. Pleomorphic adenoma
C. Adenoid cystic carcinoma
D. Metastatic squamous cell carcinoma
E. Acinic cell carcinoma

32. This image shows an FNA smear from a submandi-


bular gland mass. What is the best diagnosis?
A. Warthin tumor
B. Sialadenitis
C. Pleomorphic adenoma
D. Acinic cell carcinoma
E. Adenoid cystic carcinoma

33. A Diff-Quik stained smear obtained by FNA from a


2 cm parotid mass is shown in this figure. Which
statement is true?
A. The stroma is characteristically globular
B. When malignant transformation occurs, the
malignant component is usually adenocarcinoma
C. This tumor presents as a rapidly growing painful
mass
D. This tumor is associated with tyrosine kinase crystals
in the majority of cases
E. It occurs almost exclusively in the sublingual gland
Salivary Gland 119

34. A 62-year-old man presented with a palate mass. A


fine needle aspiration was performed and the smear
is shown here. Which of the following diagnoses is
the favored diagnosis?
A. Pleomorphic adenoma
B. Warthin’s tumor
C. Polymorphous low-grade adenocarcinoma
D. Adenoid cystic carcinoma
E. Mucoepidermoid carcinoma
120 Cytopathology Review

ANSWERS

1. A 7. E
Note: In comparison with surgical incisional biopsy, Note: Cystic lesions of the salivary gland can be
FNA is associated with a lower risk of infection and diagnostically challenging. Non-neoplastic cysts are
decreased contamination of surgical planes. There is uncommon and include retention cysts, lymphoepithelial
nevertheless a small, but real risk of bleeding, parti- cysts, and mucocele. Among cystic neoplasms, Warthin
cularly in patients with a bleeding disorder. Unnecessary tumor and low-grade mucoepidermoid carcinoma are the
surgical excision is avoided in about one-third of salivary commonest, but pleomorphic adenoma, cystadenoma,
gland masses; there are situations and certain lesions acinic cell carcinoma and squamous cell carcinoma may
which cannot be diagnosed definitively by FNA. False also be cystic. Among the five features listed in the
negative diagnoses are most often encountered in cystic answer above, complete disappearance of the cyst
lesions such as Warthin tumor, low-grade mucoepider- following aspiration is the main finding that favors a
moid carcinoma and metastatic squamous cell non-neoplastic cyst. Clinical follow-up and repeat biopsy
carcinoma. The distinction between primary lymphoma is always advisable if the specimen is hypocellular and
and benign lymphoid lesions often requires the a specific diagnosis cannot be rendered.
incorporation of flow cytometric analysis and sometimes
cannot be made on morphologic characteristics alone. 8. B
Note: Extranodal marginal zone lymphoma of MALT
2. B
(mucosa-associated lymphoid tissue) is a low-grade
Note: Salivary gland neoplasms are somewhat more
B-cell lymphoma that usually develops in the setting of
common in women. The two most common salivary
lymphoepithelial sialadenitis (LESA) or Sjögren
gland tumors (pleomorphic adenoma and Warthin’s
syndrome.
tumor) are benign neoplasms and are almost exclusively
found in the parotid gland. Polymorphous low-grade
9. C
adenocarcinoma occurs almost exclusively in the minor
Note: Both neoplasms are considered oncocytic neo-
salivary glands. The majority of sublingual and minor
plasms and are often in the same differential diagnosis.
salivary gland neoplasms are malignant.
Abundant stripped nuclei in the background is a feature
3. B of acinic cell carcinoma and is due to the high
Note: Pleomorphic adenoma is the most common cytoplasmic fragility of the tumor cells. All of the other
salivary gland neoplasm. In general salivary gland characteristics listed are features of both Warthin tumor
neoplasms are rare; most neoplasms are benign due to and acinic cell carcinoma.
the predominance of pleomorphic adenomas.
10. A
4. D Note: Acinar cells have abundant vacuolated cytoplasm,
filled with PAS-positive diastase-resistant zymogen
5. A granules.
Note: Mucoepidermoid carcinoma is the most common
11. C
salivary gland malignancy of the parotid gland, both in
Note: Polymorphous low-grade adenocarcinoma is
children and adults.
characterized by cytologically bland uniform cells
6. A arranged in different (hence the name polymorphous)
Note: Sialadenosis is a non-neoplastic and non- architectural patterns – such as tubules, cords, single-
inflammatory enlargement of salivary glands, usually file lines, and single cells. Polymorphous low grade
associated with systemic disorders. It mainly involves adenocarcinoma is rarely encountered in fine needle
the parotid gland and is often bilateral. Fine needle aspirations since it occurs almost exclusively in the
aspiration generates abundant normal salivary gland minor salivary glands of the hard palate. These occur
tissues including acini and ducts, unlikely to be mistaken more often in females than males and their peak
for other conditions. incidence is in the 40-50 years age group.
Salivary Gland 121

12. B 17. D
Note: Squamous cell carcinoma is a rare primary Note: The lesion shown in this figure is a pleomorphic
malignancy in the salivary gland. Metastasis to salivary adenoma, characterized by bright magenta fibrillary
glands or intraparenchymal lymph nodes from head and stroma on this Diff-Quik stained slide. The vast majority
neck squamous cell carcinomas is much more common. of these tumors (90%) occur in the parotid gland.

13. D 18. B
Note: HIV-associated cystic lymphoepithelial lesions are Note: The lesion shown in the figure is a basaloid
characterized by epithelial cell clusters with interspersed neoplasm. The differential diagnosis for fine needle
lymphocytes. These cystic lesions are usually multiple aspirations of basaloid neoplasms includes basal cell
and often bilateral. The presence of epithelial cells adenoma, basal cell adenocarcinoma, the solid variant
distinguishes this cyst from a lymph node. Warthin of adenoid cystic carcinoma, and cellular pleomorphic
tumor can occasionally show some squamous meta- adenoma. Acinic cell carcinoma, which is characterized
plasia, but still should have a prominent population of by oncocytic cells, is not included in this differential
oncocytes. Acinic cell carcinoma is composed predo- diagnosis.
minantly of acinar cells and cystic metastatic squamous
cell carcinoma will usually have more severe cytologic 19. D
atypia, necrosis, and increased mitotic activity. Note: Aspirates obtained from Warthin tumor are
typically characterized by cohesive groups of oncocytic
14. D cells in a background of lymphocytes. Oncocytes have
Note: Aspirates of normal salivary gland tissue are abundant granular cytoplasm with round centrally
comprised of acinar cells and ductal cells admixed with located nuclei.
adipose tissue. Acinar cells (illustrated here) are
arranged in cohesive grape-like clusters and have 20. B
granular basophilic cytoplasm. Note: This basal cell adenoma is characterized by small
basaloid cells with peripheral palisading and focally
15. C surrounded by a thin peripheral ribbon of pale-blue
Note: Tyrosine crystalloids are floret-shaped crystals acellular matrix material.
often associated with pleomorphic adenoma, but can be
seen in other benign and malignant lesions. Pleomorphic 21. C
adenoma is characterized by bland cohesive epithelial Note: This smear shows a Pap stain of a pleomorphic
cells within chondromyxoid matrix. adenoma (PA), the most common salivary gland tumor,
both in children and in adults. PAs are most commonly
16. A found in the superficial parotid gland. Squamous and
Note: This aspirate containing degenerated/anucleated mucinous metaplasia can occur in pleomorphic adenoma
squamous cells admixed with bland nucleated squamous and focal mild cytologic atypia is seen in about 20% of
cells with minimal cytologic atypia in a background of PAs. Severe atypia, many mitoses and necrosis are not
mixed inflammatory cells was most likely obtained from features of PA; if found these changes suggest a
a benign squamous epithelial-lined cyst. Warthin tumor, malignancy (carcinoma ex pleomorphic adenoma).
acinic cell carcinoma, mucoepidermoid carcinoma, and
cystic metastatic squamous cell carcinoma can also 22. D
present as cystic tumors. Warthin tumor can occasionally Note: The diagnostic criteria of mucoepidermoid carci-
show some squamous metaplasia, but still should have noma are the combination of mucus cells (arrowhead),
a prominent population of oncocytes. Acinic cell epidermoid (squamoid) cells, and intermediate cells.
carcinoma is composed predominantly of acinar cells Large polygonal cells with PAS+ granules characterize
and mucoepidermoid carcinoma has mucus and acinic cell carcinoma. Oncocytic epithelial cells and
intermediate cells, in addition to the squamous cells. lymphocytes are typical of Warthin tumor. Salivary duct
Cystic metastatic squamous cell carcinoma will usually carcinoma is comprised of cells with high grade nuclei
have more severe cytologic atypia, necrosis, and and prominent nucleoli resembling comedo ductal
increased mitotic activity. carcinoma in situ of the breast.
122 Cytopathology Review

23. E cytoplasm. There is a predominance of bare nuclei


Note: This oncocytoma is characterized by a cellular embedded in a foamy background substance, represen-
aspirate comprised of sheets of cohesive oncocytes with ting the high cytoplasmic fragility of these tumor cells.
a clean background and no lymphocytes. Warthin tumor
has cohesive clusters of oncocytes as well but also shows 29. B
granular debris and lymphocytes in the background. Note: Classic adenoid cystic carcinoma is characterized
Acinic cell carcinoma is usually a cellular aspirate, by small epithelial cells with a cribriform architecture
however in addition to the sheets of large polygonal and globular pink hyaline membrane material. These
cells, single cells and naked nuclei are usually tumors are often painful as they have a propensity for
prominent. The cytoplasm of acinic cells is more peri-neural invasion.
basophilic and granular than that of oncocytes.
30. D
24. C Note: The abundant magenta fibrillary stroma and bland
Note: Features of mucoepidermoid carcinoma include epithelial cells is diagnostic of pleomorphic adenoma.
the combination of mucus cells (arrowhead), epidermoid
(squamoid) cells (arrow) and intermediate cells 31. D
(asterisk). Occasionally, individual cells show both Note: Squamous cell carcinoma is a rare primary
mucus and squamous cells differentiation, a feature malignancy in the salivary gland; metastasis to an
diagnostic of mucoepidermoid carcinoma. intraparotid lymph node from a head and neck site is
more common.
25. B
Note: Adenoid cystic carcinoma is the most common 32. E
primary malignancy of all salivary glands except the Note: This Pap-stained smear shows basaloid cells with
parotid. It can also occur in many other places including variably sized hyaline matrix globules. The globular
lacrimal and sweat glands, respiratory tract and breast. matrix is much less prominent in Pap-stained slides
The tumor is characterized by slow but relentless growth compared to Diff-Quik stained slides, however the
and the overall prognosis is poor, often with eventual cribriform architecture can still be appreciated.
metastasis to lung and bone. The metachromatic
basement membrane material is globular as opposed of 33. B
fibrillary (which is the characteristic pattern of matrix Note: Microscopically carcinoma ex pleomorphic
associated with pleomorphic adenoma). Most adenoid adenoma shows features of pleomorphic adenoma and
cystic carcinomas show strong immunoreactivity with an epithelial malignancy – most often adenocarcinoma.
the antibody to CD117. Pleomorphic adenoma occurs most commonly in the
parotid gland, usually, presenting as a firm painless
26. D nodule. In Diff-Quik stained smears it contains magenta
Note: The majority of Warthin tumors are found in older colored fibrillary stroma which is associated with
adults in the 50-70 years age group. tyrosine kinase crystals in a minority of cases. Malignant
transformation occurs in these tumors relatively rarely
27. A but it occurs in a time dependent manner: 2% of
Note: The tumor shown is a high-grade malignant tumor; incompletely removed and recurrent tumors or tumors
this category encompasses salivary duct carcinoma, that were not resected undergo malignant transformation
high-grade mucoepidermoid carcinoma, adenocarcinoma over the period of 5 years, and 10% undergo malignant
NOS, metastatic tumors and carcinoma ex pleomorphic transformation over the period of 10 years.
adenoma. Acinic cell carcinoma is an oncocytic tumor
with bland cytologic features. 34. C
Note: The smear shows monomorphous population of
28. C cuboidal cells adhering to strands of fibrovascular
Note: This Diff-Quik stained smear is composed of stroma in a trabecular (pseudopapillary) pattern with
acinar cells with eccentric round monomorphic nuclei scant myxoid matrix. The cells have finely granular
with scant nucleoli and abundant granular basophilic chromatin and indistinct nucleoli. These cytologic
Salivary Gland 123

features plus the tumor location of palate favor a Compared to pleomorphic adenoma, PLGA has scant
diagnosis of polymorphous low-grade adenocarcinoma extracellular matrix. Compare to adenoid cystic
(PLGA). The distinction between PLGA and carcinoma, tumor cells in PLGA are not basaloid, they
pleomorphic adenoma or adenoid cystic carcinoma can have moderate amount of cytoplasm; the nuclei have
be very challenging, even in histology sections. open chromatin and sometimes are vacuolated.
124 Cytopathology Review

9
Lymph Nodes
Fang Fan

QUESTIONS
1. Indications for lymph node fine needle aspiration (FNA) include all of the following, EXCEPT:
A. Diagnosing a suspected malignancy
B. Staging a known malignancy
C. Monitoring recurrent malignancy
D. Confirming progression of a known lymphoma
E. Subtyping of classic Hodgkin lymphoma

2. All of the following are included in the differential diagnosis of granulomatous lymph node inflammations,
EXCEPT:
A. Sarcoidosis
B. Cat scratch disease
C. Castleman disease
D. Mycobacterial lymphadenitis
E. Tularemia (Francisella tularensis infection)

3. Which of the following is the key cytomorphologic feature of Rosai-Dorfman disease?


A. Emperipolesis
B. Dendritic-lymphocytic aggregates
C. Granulomas
D. Asteroid bodies
E. Karyorrhexis

4. All of the following findings are characteristic features of Kikuchi lymphadenitis, EXCEPT:
A. Necrotic debris
B. Cytoplasmic tangible bodies
C. Small phagocytic histiocytes
D. Abundant neutrophils
E. Increased immunoblasts

5. A 45-year-old woman with a clinical history of Sjögren syndrome presented with a right parotid mass.
FNA demonstrated a polymorphous population of lymphocytes, plasma cells, follicular dendritic cells
and occasional monocytoid cells. The aspirate was also sent for a cytogenetic study. A FISH probe for
which of the following translocations would be most useful for establishing the final diagnosis?
A. t(11;14)
B. t(14;18)
C. t(11;18)
D. t(8;14)
E. t(2;8)
Lymph Nodes 125

6. Which of the following lymphomas contain numerous macrophages with ingested nuclear fragments
(so called “tingible-body macrophages”) in the FNA smears?
A. Small lymphocytic lymphoma
B. Follicular lymphoma
C. Mantle cell lymphoma
D. Marginal zone lymphoma
E. Burkitt lymphoma

7. A 52-year-old man was found to have an enlarged right axillary lymph node. A previous history of
right forearm skin “tumor” was noted. FNA of the axillary lymph node yielded cellular smears showing
aggregates and dispersed tumor cells with round to oval nuclei, scant cytoplasm, fine chromatin, and
small inconspicuous nucleoli. Occasional perinuclear cytoplasmic condensations and nuclear molding
are seen. A positive immunohistochemical stain to which of the following antigens would confirm the
diagnosis?
A. S-100
B. Mart-1
C. Tumor protein p63
D. Cytokeratin CK20
E. Cytokeratin CK5/6

8. The differential diagnosis of plasmacytoid neoplastic cells in a lymph node FNA includes all of the
following, EXCEPT:
A. Plasmablastic lymphoma
B. Anaplastic large cell lymphoma
C. Melanoma
D. Metastatic breast carcinoma
E. Metastatic neuroendocrine carcinoma

9. A 54-year-old Asian male presented with a nasal mass and an enlarged right neck lymph node. FNA of
the lymph node revealed abundant isolated intermediate-sized neoplastic cells with coarse granular
chromatin and pale vacuolated cytoplasm. Immunohistochemical stains on the cell block sections were
positive for CD2 and CD56, and negative for CD3 and CD4. In-situ hybridization for EBV was positive
in the tumor cells. What is the correct diagnosis?
A. Nasopharyngeal carcinoma
B. Small cell carcinoma
C. Hodgkin lymphoma
D. NK/T-cell lymphoma
E. Anaplastic large cell lymphoma

10. Which of the following immunohistochemical stains is positive in cells of Langerhans cell histiocytosis,
but negative in Rosai-Dorfman disease?
A. S-100
B. CD68
C. CD1a
D. CD163
E. CD45
126 Cytopathology Review

11. Solitary left supraclavicular lymph node enlargement (so called “Virchow node”) is most often caused
by which process?
A. Reactive lymphoid hyperplasia
B. Granulomatous inflammation
C. Castleman disease
D. Hodgkin lymphoma
E. Metastasis of gastrointestinal tract carcinoma

12. A 12-year-old boy with a clinical history of Ewing sarcoma is found to have an enlarged inguinal lymph
node. FNA shows clusters and single uniform cells with round to oval nuclei, scant cytoplasm and
inconspicuous nucleoli. Which of the following stains will support the diagnosis of metastatic Ewing
sarcoma?
A. CD34
B. WT-1
C. FLI1
D. MyoD1
E. CD45

13. What do these structures (arrows) represent in a


lymph node FNA smear?
A. Platelets
B. Fungus
C. Viral particles
D. Cytoplasmic fragments
E. Staining contaminant

14. A 35-year-old man has a right neck enlarged rubbery


lymph node. An FNA is performed and the most
representative area is shown here. What is the
diagnosis?
A. Reactive lymphoid hyperplasia
B. Cat scratch disease
C. Sarcoidosis
D. Non-Hodgkin lymphoma
E. Metastatic carcinoma
Lymph Nodes 127

15. A 15-year-old boy was found to have an enlarged


right neck lymph node. An FNA was performed (see
image). Special stain demonstrated gram-negative
bacilli. What is the most likely diagnosis?
A. Sarcoidosis
B. Reactive lymphoid hyperplasia
C. Castleman disease
D. Cat scratch disease
E. Mycobacterial lymphadenitis

16. This lymph node FNA smear from a 29-year-old man


is highly suggestive of which of the following diseases?
A. Reactive lymphoid hyperplasia
B. Infectious mononucleosis
C. Hodgkin lymphoma
D. Anaplastic large cell lymphoma
E. Nasopharyngeal carcinoma

17. This FNA was performed on a neck lymph node of a


42-year-old man. The cell group in the center of the
smear indicates which of the following?
A. Fungal organisms
B. Parasites
C. Germinal center fragment
D. Metastatic carcinoma
E. Metastatic melanoma
128 Cytopathology Review

18. A 37-year-old woman presented with left axillary


lymph node enlargement. An FNA was performed
and the smear is shown here. This fine needle
aspiration smear is diagnostic of which of the
following conditions?
A. Reactive lymphoid hyperplasia
B. Dermatopathic lymphadenopathy
C. Rosai-Dorfman disease
D. Metastatic melanoma
E. Hodgkin lymphoma

19. A 45-year-old woman with known BRCA1 gene


mutation was found to have enlarged inguinal lymph
nodes. An FNA was performed and the smear
prepared from the aspirated material is shown here.
What is the most likely diagnosis?
A. Metastatic melanoma
B. Non-Hodgkin lymphoma
C. Metastatic colonic adenocarcinoma
D. Metastatic lung adenocarcinoma
E. Metastatic ovarian adenocarcinoma

20. A 56-year-old woman complaining of shortness of


breath and chest pain was found to have an enlarged
supraclavicular lymph node. She had a history of
breast tubular carcinoma, which was removed
surgically 5 years ago. This image represents a smear
from this lymph node’s FNA. What is the most likely
diagnosis?
A. Metastatic breast tubular carcinoma
B. Metastatic small cell carcinoma
C. Metastatic melanoma
D. Diffuse large B-cell lymphoma
E. Burkitt lymphoma
Lymph Nodes 129

21. A 25-year-old woman was found to have an enlarged


right neck lymph node on palpation. She had a history
of papillary thyroid carcinoma when she was 20.
What is the correct interpretation of this FNA smear?
A. Metastatic papillary thyroid carcinoma
B. Hodgkin lymphoma
C. Granulomatous process
D. Cat scratch disease
E. Reactive hyperplasia

22. The presence of numerous structures like these


(arrows) in a lymph node aspiration smear is
commonly seen in which of the following conditions?
A. Small lymphocytic lymphoma
B. Burkitt lymphoma
C. Diffuse large B-cell lymphoma
D. Metastatic small cell carcinoma
E. Metastatic melanoma

23. This FNA smear was collected from a retroperitoneal


lymph node of a 32-year-old man. What is the correct
diagnosis?
A. Reactive lymphoid hyperplasia
B. Diffuse large B-cell lymphoma
C. Anaplastic large T-cell lymphoma
D. Metastatic melanoma
E. Metastatic seminoma
130 Cytopathology Review

24. A 62-year-old man with a history of lip cancer


presented with two enlarged left neck lymph nodes.
FNA was performed and shown here. What is the
correct diagnosis?
A. Metastatic basal cell carcinoma
B. Metastatic squamous cell carcinoma
C. Metastatic papillary thyroid carcinoma
D. Metastatic small cell carcinoma
E. Lymphoma

25. This FNA was obtained from a neck lymph node of a


35-year-old man. Gene translocation involving which
of the following genes would be diagnostic of Burkitt
lymphoma?
A. C-MYC
B. BCL-2
C. BCL-6
D. CYCLIN D1
E. ALK-1

26. A 27-year-old female was found to have a large


mediastinal mass. A CT-guided FNA was performed
and shown in this image. What is the most likely
diagnosis?
A. Hodgkin lymphoma
B. Diffuse large B-cell lymphoma
C. Thymoma
D. Teratoma
E. Papillary thyroid carcinoma
Lymph Nodes 131

27. An endoscopic-ultrasound guided fine-needle


aspiration (EUS-FNA) was performed to sample a
para-tracheal lymph node in a patient with a clinical
history of esophageal adenocarcinoma. The smear is
shown here. What is the diagnosis?
A. Metastatic adenocarcinoma
B. Metastatic small cell carcinoma
C. Hodgkin lymphoma
D. Reactive lymphoid follicle
E. Granulomas

28. A 56-year-old man was found to have enlarged


mediastinal lymph nodes. This image was taken from
the endobronchial ultrasound-guided FNA smear of
the lymph node. What is the correct diagnosis?
A. Metastatic carcinoma
B. Metastatic sarcoma
C. Metastatic melanoma
D. Anthracosis
E. Rosai-Dorfman disease

29. A 5-year-old boy presented with respiratory distress


and was found to have a large anterior mediastinal
mass. An emergent CT-guided FNA was performed
and shown here. Which of the following positive
stains would confirm the diagnosis?
A. CD20
B. CD15
C. CD30
D. Anaplastic lymphoma kinase -1 (ALK-1)
E. Terminal deoxynucleotidyl transferase (TdT)
132 Cytopathology Review

30. A 45-year-old man had an enlarged para-aortic


lymph node. An FNA was performed and shown here.
What is the most likely diagnosis?
A. Metastatic small cell carcinoma of the lung
B. Metastatic urothelial carcinoma
C. Metastatic germ cell tumor
D. Diffuse large B-cell lymphoma
E. Necrotizing granuloma
Lymph Nodes 133

ANSWERS

1. E 5. C
Note: FNA is usually the first approach for establishing Note: The clinical history and cytologic findings are
the cause of lymphadenopathy, including the above listed suspicious for marginal zone lymphoma of the mucosal
reasons A to D. Accurate subtyping of classic Hodgkin associated lymphoid tissue (MALT). The most common
lymphoma is difficult on FNA smears and may lack genetic alteration in MALT lymphoma is t(11;18)
clinical significance because the stage not subtypes of (q21;21). The other translocations listed in the suggested
Hodgkin lymphoma determines the prognosis and therapy. are found in the following lymphomas: t(11;14)- mantle
cell lymphoma; t(14;18)- follicular lymphoma; t(8;14)
2. C and t(2;8)- Burkitt lymphoma.
Note: All the conditions listed here besides Castleman
disease are granulomatous processes and are usually 6. E
included in the differential diagnosis of granulomatous Note: Burkitt lymphoma is characterized by tumor cells
lymphadenitis. Castleman disease is a form of lymphoid with high proliferative activity, showing abundant
hyperplasia with prominent large dendritic cells mitoses and apoptosis with numerous tingible-body
(hyaline-vascular type) or sheets of plasma cells (plasma macrophages which corresponds to the “starry sky”
cell type). Granulomas are not found in lymph nodes pattern in tissue sections.
involved by Castleman disease.
7. D
3. A Note: The cytologic descriptions, especially the presence
Note: Emperipolesis is a key feature of Rosai-Dorfman of “occasional perinuclear cytoplasmic condensations
disease, also called sinus histiocytosis with massive and nuclear molding”, suggest a diagnosis of metastatic
lymphadenopathy. It refers to the engulfment of lympho- Merkel cell carcinoma. CK 20 immunostain with a
cytes into the cytoplasm of histiocytes (macrophages) characteristic “dot-like” cytoplasmic positivity will
which are S100 and CD68 positive. Granulomas are not confirm the diagnosis. Melanoma cells (positive for
usually found in Rosai-Dorfman disease. Dendritic- S100 and Mart-1) typically have prominent nucleoli and
lymphocytic aggregates occur in reactive lymphoid moderate amount of cytoplasm. Squamous cell carci-
hyperplasia. Asteroid bodies are small cytoplasmic noma cells (positive for p63 and CK5/6) have abundant
bodies often seen in sarcoidosis but may be present in dense cytoplasm and grow in large cohesive groups.
other granulomas as well; hence they do not have any
diagnostic significance. Karyorrhexis refers to fragmen- 8. B
tation of the nucleus during cell death. Although it is Note: The cells of all the listed tumors, except anaplastic
not diagnostic of any particular disease it is often large cell lymphoma, may have plasmacytoid morpho-
prominent in Kikuchi lymphadenitis. logy. Anaplastic large cell lymphoma is characterized
by large pleomorphic tumor cells with irregular nuclei
4. D or horseshoe-shaped nuclei and nuclei reminiscent of
Note: Kikuchi lymphadenitis (also known as histiocytic Reed-Sternberg cells. These anaplastic cells do not
necrotizing lymphadenitis or Kikuchi-Fujimoto disease) resemble plasma cells and need not be considered in the
is a self-limited lymphadenopathy of unknown etiology, differential diagnosis of plasmacytoid neoplasms
that usually affects young Asian individuals with a slight involving the lymph nodes.
female predominance. FNA smears show necrotic
debris, nuclear dusts, admixed with small phagocytic 9. D
histiocytes. These histiocytes are smaller than so called Note: The morphologic and ancillary test results in this
tangible-body macrophages and have crescent-shaped case are diagnostic of natural killer cell-T-cell (NK/T-
peripheral nuclei. Immunoblasts and plasmacytoid cell) lymphoma. It affects mainly the Asian population,
monocytes are increased in number. Neutrophils are and typically presents as a sinonasal mass with
absent. The process resolves spontaneously in weeks to occasional lymph node involvement. The tumor cells
months and antibiotic therapy is not necessary. express CD2 and CD56, and show a loss of CD3 and
134 Cytopathology Review

CD4 expression. Positivity for EBV is required for the have bland morphology and thus the diagnosis of
diagnosis. Anaplastic large cell lymphoma is positive carcinoma can be excluded.
for CD3 and negative for EBV.
15. D
10. C Note: The smear displays abundant neutrophils and a
Note: The neoplastic cells in Langerhans cell necrotizing granuloma, which are characteristic findings
histiocytosis are positive for CD1a, and antigen that is in Cat scratch disease. Steiner stain is usually used to
not expressed on cells of the Rosai-Dorfman disease. demonstrate the gram-negative bacillus Bartonella
S100 is expressed in cells of Langerhans histiocytosis henselae. Sarcoidosis is characterized by tightly struc-
and Rosai-Dorfman disease. tured granulomas composed of epithelioid histiocytes
and giant cells with a “clean background”. It most often
11. E affects middle-aged African American women. Reactive
Note: In adults, supraclavicular lymphadenopathy most lymphoid hyperplasia is composed of dendritic
often results from metastatic carcinomas involving the lymphocytic aggregates and tingible-body macrophages.
lymph nodes. Abdominal malignancies are more likely Castleman disease is a form of lymphoid hyperplasia
to metastasize to the left supraclavicular lymph node containing large dendritic cells in the hyaline-vascular
which they reach through the lymph of the thoracic duct. type or sheets of plasma cells in the plasma cell type of
Thoracic duct lymph is namely filtered through the the disease. Mycobacterial lymphadenitis has similar
Virchow node prior to entering the superior vena cava. cytology features, and the definitive diagnosis is
Carcinomas of the lung, breast and head and neck established by demonstrating the causative acid-fast
usually metastasize to the supraclavicular lymph node bacilli.
ipsilateral to the primary tumor.
16. C
12. C Note: The image shows a classic binucleated Reed-
Note: Ewing sarcoma is positive for FLI1 and CD99. Sternberg cell with prominent macronucleoli. The
background contains small lymphocytes and eosinophils
13. D (not shown). The diagnosis is confirmed immunohisto-
Note: These small bluish globules are sloughed off chemically, because these large cells are CD15 and
cytoplasm fragments of lymphocytes. Because CD30 positive.
lymphocytes are fragile and their cytoplasms are easily
stripped off during smearing. These cytoplasmic 17. C
fragments are also called “lymphoglandular bodies”. Note: This group contains dendritic cells and a tingible
Their presence indicates a proliferative lymphoid body macrophage indicating a germinal center fragment.
process, which may be either benign or malignant.
18. D
14. A Note: The abundant cytoplasmic melanin pigments in
Note: The smear shows mixed population of large and the tumor cells are diagnostic of metastatic melanoma.
small lymphocytes and loose dendritic-lymphocytic Dermatopathic lymphadenopathy is associated with
aggregates composed of small round lymphocytes and chronic dermatoses and is characterized by melanin-
large dendritic cells which have abundant pale and laden macrophages. In this smear, melanin pigment
delicate cytoplasm. They are also called germinal center granules are seen in the cytoplasm of tumor cells.
fragments when tingible-body macrophages are present.
The identification of dendritic-lymphocytic aggregates 19. E
in a lymph node FNA is indicative of reactive lymphoid Note: The cells in the smear form cohesive groups, and
hyperplasia. Cat scratch disease is a necrotizing granulo- have enlarged vesicular nuclei and prominent macro-
matous process. Sarcoidosis is also a granulomatous nucleoli; these features suggest a high-grade adeno-
disease containing granulomas characterized by carcinoma. In light of the clinical history of BRCA1
epithelioid histiocytes. Non-Hodgkin lymphoma is mutation, this carcinoma most likely arose from the
characterized by a monotonous population of lympho- ovarian surface epithelium and represents an ovarian
cytes without germinal center cell fragments. These cells serous adenocarcinoma.
Lymph Nodes 135

20. B 25. A
Note: The cells have high nuclear-to-cytoplasmic ratio Note: The smear shows intermediate-sized lymphocytes,
and show characteristic nuclear molding and finely apoptotic cells and a tingible-body macrophage,
granular nuclear chromatin. These are features of a small suggestive of a high-grade lymphoma. A positive FISH
cell carcinoma and are not compatible with the diagnosis analysis for C-MYC translocation would confirm the
of metastatic breast tubular carcinoma. diagnosis of Burkitt lymphoma.

21. E 26. B
Note: The smear shows mixed populations of lympho- Note: The smear shows isolated large tumor cells with
cytes and a dendritic-lymphocytic aggregate, features moderate amount of cytoplasm, vesicular nuclei and
characteristic of reactive hyperplasia. A dendritic- prominent nucleoli. These cytomorphologic features and
lymphocytic aggregate is composed of loose collections the typical clinical presentation are characteristic of a
of small lymphocytes and dendritic cells and should not primary mediastinal diffuse large B-cell lymphoma.
be mistaken for epithelial groups or granulomas.
27. E
22. A Note: The smear shows a cell group of histiocytes typical
Note: These structures are called “smudge cells” or of a granuloma. It is important not to misdiagnose
“basket cells” or “shadow cells of Gumprecht”. They granulomas as metastatic carcinoma in FNA procedure
represent naked nuclei of lymphoid cells composed used for staging of carcinomas.
of basophilic nuclear material in thin interwoven strands.
They have irregular contours and are much larger 28. D
than the surrounding lymphocytes. Smudge cells are Note: The image shows spindle-shaped aggregates of
more common in aspirates of small lymphocytic pigmented histiocytes consistent with an anthracotic
lymphoma and sometimes represent a large portion of mediastinal lymph node.
all cells in the smear from lymph nodes involved by this
tumor. 29. E
Note: The clinical presentation and the cytology images
23. E are characteristic of lymphoblastic lymphoma of T-cell
Note: The smear shows characteristic “tigroid” (tiger derivation (T-LBL). The image shows a pure population
stripe-like) background of seminoma. They represent of blasts with round nuclei, finely dispersed chromatin,
sloughed off strands of cytoplasm of the seminoma cells. inconspicuous nucleoli, and scant cytoplasm. The cells
The tumor cells are large with abundant cytoplasm and are positive for terminal deoxynucleotidyl transferase
prominent nucleoli. (TdT) and T-cell markers.

24. C 30. B
Note: The cells form a cohesive flat sheet. They have Note: The smear shows a loosely cohesive group of
nuclear grooves and intranuclear pseudoinclusions, malignant cells. The cells have irregular nuclear
which are highly indicative of papillary thyroid carci- membrane, vesicular chromatin, prominent nucleoli and
noma. Further clinical examination revealed a papillary moderate amount of cytoplasm. These cytologic features
thyroid carcinoma with ipsilateral neck lymph node indicate a non-small cell carcinoma. Metastatic urothelial
metastases, which occur often in these patients. carcinoma is the most suitable diagnosis in this case.
136 Cytopathology Review

10
Liver
Maura O’ Neil

QUESTIONS
1. Which of the following diagnoses can reliably be made by fine needle aspiration (FNA) of a liver
lesion?
A. Hepatic adenoma
B. Hepatocellular carcinoma
C. Focal nodular hyperplasia
D. Regenerative nodule in cirrhosis
E. Focal fatty change

2. Bile duct epithelium is usually present, but sparse in FNA specimens obtained from normal liver.
An FNA specimen from which of the following lesions typically does not contain bile duct
epithelium?
A. Hepatic adenoma
B. Bile duct hamartoma
C. Bile duct adenoma
D. Cirrhosis
E. Focal nodular hyperplasia

3. A 24-year-old female from a Mediterranean country was found to have a 5 cm liver cyst. The cyst was
aspirated by interventional radiology and shorty after the procedure an anaphylactic shock developed.
The cyst fluid revealed fragments of laminated membrane material and rare unstained hooklets. What
is the most likely diagnosis?
A. Entamoeba histolytica abscess
B. Escherichia coli bacterial abscess
C. Polycystic liver disease
D. Echinococcal cyst (hydatid cyst)
E. Miliary tuberculosis

4. An FNA of an 8 cm liver lesion from a 22-year-old female was performed with image guidance. The
remainder of her liver appeared normal and she does not have a history of liver disease. FNA showed
large oncocytic neoplastic cells, some with intracytoplasmic hyaline globules, admixed with fragments
of hyalinized fibrosis. What is the most likely diagnosis?
A. Cholangiocarcinoma
B. Metastatic adenocarcinoma
C. Hepatocellular carcinoma, classic type
D. Fibrolamellar hepatocellular carcinoma
E. Hepatoblastoma
Liver 137

5. A 56-year-old male with a history of cirrhosis and colonic adenocarcinoma had a CT scan of the abdomen
which showed a large liver mass. FNA was performed which yielded cohesive groups of malignant
poorly differentiated cells. A cell block was made and immunohistochemical stains gave the following
results: polyclonal carcinoembryonic antigen (CEA) (diffuse cytoplasmic +), cytokeratin 7 (-), cytokeratin
20 (+), HepPar1 (-), MOC-31 (+), CDX-2 (+), synaptophysin (-). What is the correct diagnosis?
A. Metastatic lung adenocarcinoma
B. Metastatic colonic adenocarcinoma
C. Hepatocellular carcinoma
D. Metastatic neuroendocrine carcinoma
E. Metastatic squamous cell carcinoma

6. A 34-year-old male with a history of tuberous sclerosis was found to have an incidental liver lesion on
imaging studies. An FNA of the lesion showed adipocytes, endothelial cells and abundant blood, clusters
of spindle and epithelioid cells, and hematopoietic cells. A cell block was prepared and the plump spindle
cells were immunohistochemically positive with the antibody to HMB-45. What is the best diagnosis?
A. Hepatocellular carcinoma
B. Angiosarcoma
C. Angiomyolipoma
D. Myelolipoma
E. Metastatic carcinoma

7. A 25-year old female presents with abdominal pain


and a subsequent abdominal CT showed a liver lesion.
An FNA was performed on the liver lesion and a
representative group of cells is shown here. Based on
the cells shown, what is the best diagnosis?
A. Hepatocellular carcinoma
B. Hepatic adenoma
C. Hemangioma
D. Metastatic gastrointestinal stroma tumor
E. Metastatic adenocarcinoma
138 Cytopathology Review

8. A 50-year-old male with a history of alcoholic


cirrhosis had a CT-guided FNA of a “dominant
nodule/mass” in the right lobe of the liver. What is
the best interpretation of the cells shown in this
smear?
A. Cholangiocarcinoma
B. Hepatocellular carcinoma
C. Hemangioma
D. Regenerative nodule in cirrhosis
E. Metastatic adenocarcinoma

9. What is the diagnosis for this smear prepared from


a liver FNA?
A. Normal liver
B. Hepatocellular carcinoma
C. Hepatic adenoma
D. Cholangiocarcinoma
E. Metastatic adenocarcinoma

10. A 48-year-old male with a history of ulcerative colitis


and primary sclerosing cholangitis had a large liver
mass and an elevated serum CA19-9. An FNA was
performed and is shown here. Immunohistochemical
stains performed on the cell block showed that the
tumor cells are positive for keratin CK7 and MOC-
31. What is the best interpretation?
A. Cholangiocarcinoma
B. Normal liver
C. Hepatic adenoma
D. Hepatocellular carcinoma
E. Metastatic colonic adenocarcinoma
Liver 139

11. A 50-year-old male with a history of chronic hepatitis


C infection and multiple liver masses had an FNA of
the largest mass. This smear was prepared from that
FNA. What is the best diagnosis?
A. Cholangiocarcinoma
B. Normal liver
C. Hepatic adenoma
D. Hepatocellular carcinoma
E. Metastatic colonic adenocarcinoma

12. A 72-year-old female with a history of cancer (site


not specified) was found to have multiple liver masses.
A CT-guided FNA was performed and is shown here.
What is the most likely diagnosis based on the group
of cells shown?
A. Cholangiocarcinoma
B. Normal liver
C. Hepatic adenoma
D. Hepatocellular carcinoma
E. Metastatic colonic adenocarcinoma

13. A 62-year-old male with a history of hepatitis C-related


cirrhosis and colon cancer had a large liver mass. A
Pap stained smear from the CT-guided FNA is shown
below. A cell block was also prepared and immuno-
histochemical stains performed on the tumor cells gave
the following results: CK7(-), CK20(-), (TTF-1) (-),
HepPar1(+), polyclonal (CEA) (+ in a pericanalicular
pattern). What is the correct diagnosis?
A. Cholangiocarcinoma
B. Metastatic bronchogenic adenocarcinoma
C. Metastatic melanoma
D. Hepatocellular carcinoma
E. Metastatic colonic adenocarcinoma
140 Cytopathology Review

14. This image shows cells from a CT-guided right lower


lobe lung mass. They most likely represent:
A. Mesothelial cells
B. Bronchogenic adenocarcinoma
C. Normal hepatocytes
D. Reactive pneumocytes
E. Renal cell carcinoma

15. An FNA of a liver mass was performed and the cells


shown were obtained. Immunohistochemical stains
performed on a cell block from the same aspirate
showed that the tumor cells are positive for CDX2
and negative for cytokeratin CK7. What is the most
likely diagnosis?
A. Metastatic colonic adenocarcinoma
B. Metastatic lung adenocarcinoma
C. Metastatic melanoma
D. Hepatocellular carcinoma
E. Cholangiocarcinoma

16. An FNA of a liver lesion in a patient with abnormal


liver enzymes and a history of chronic viral hepatitis
C infection. The cells shown are representative of the
hepatic lesion. What is the best interpretation of these
cells?
A. Renal cell carcinoma
B. Metastatic colon adenocarcinoma
C. Hepatic steatosis
D. Hepatocellular carcinoma
E. Benign macrophages (Kupffer cells)
Liver 141

17. A 76-year-old male was found to have multiple liver


lesions and an FNA from one of the lesions is shown
below. What is the most likely diagnosis?
A. Metastatic squamous cell carcinoma
B. Metastatic adenocarcinoma of the pancreas
C. Metastatic small cell carcinoma
D. Hepatocellular carcinoma
E. Cholangiocarcinoma

18. A 65-year-old male with a history of lung cancer


presented with multiple liver lesions. An FNA is
performed on one of the lesions and shown here.
What is the best diagnosis?
A. Metastatic squamous cell carcinoma
B. Metastatic adenocarcinoma of the pancreas
C. Metastatic small cell carcinoma
D. Hepatocellular carcinoma
E. Cholangiocarcinoma

19. An FNA is performed on a lesion in a patient with a


history of alcohol abuse and abnormal liver function
tests. What is the best interpretation of this smear?
A. Cirrhosis
B. Hepatocellular carcinoma
C. Hepatic adenoma
D. Angiosarcoma
E. Metastatic carcinoma
142 Cytopathology Review

20. A 54-year-old man presented to his primary care


physician complaining of abdominal pain. Imaging
studies revealed a 7 cm mass in his liver. Subsequent
laboratory studies included positive serologic tests for
viral hepatitis C infection. An FNA was performed
on the large liver mass and aspirates are shown here.
What is the diagnosis?
A. Metastatic squamous cell carcinoma
B. Metastatic adenocarcinoma of the pancreas
C. Cirrhotic nodule of the liver
D. Hepatocellular carcinoma
E. Cholangiocarcinoma

21. This image shows a FNA smear from a right lobe liver
mass of a 72-year-old man. Immunohistochemical
stains performed on the cell block demonstrated
tumor cells being positive for glypican-3 and CD10
(canalicular positivity), and negative for CK7 and
CK20. What is the correct diagnosis?
A. Cholangiocarcinoma
B. Hepatocellular carcinoma
C. Metastatic colonic adenocarcinoma
D. Metastatic lung adenocarcinoma
E. Melanoma

22. A 57-year-old male presented to the emergency room


with severe secretory diarrhea. During clinical work
up, a right lobe liver mass was discovered. A CT-
guided fine needle aspiration was performed and
shown here. What is the most likely diagnosis?
A. Lymphoma
B. Hepatocellular carcinoma
C. Cholangiocarcinoma
D. Metastatic neuroendocrine tumor
E. Cirrhosis
Liver 143

ANSWERS

1. B 5. B
Note: Hepatocellular carcinoma (HCC) can be reliably Note: Metastatic colon adenocarcinoma is typically
diagnosed in the majority of cases; the reported sensiti- positive for the following immunohistochemical stains:
vity of FNA diagnosis of HCC ranges from 87 to 100% CK20, CDX-2, villin, and MOC-31. Polyclonal CEA
with a specificity from 90 to 94%. Hepatic adenoma will show a diffuse cytoplasmic staining in contrast to
cannot be distinguished by FNA from focal nodular the sharp canalicular staining of hepatocellular carci-
hyperplasia or regenerative nodules in cirrhosis or focal noma. Hepatocellular carcinoma is typically positive
fatty change. However, a benign result is useful in that with HepPar1 and CK8/18 and negative for CK7, CK20,
it can exclude a malignant neoplasm. An assessment of and MOC-31. Metastatic lung adenocarcinoma will also
tissue architecture is required for the definitive diagnosis show diffuse cytoplasmic positivity with polyclonal
of these benign lesions and accordingly best diagnostic CEA but is positive for CK7, MOC-31, and TTF-1.
results are obtained by core needle biopsy.
6. C
2. A Note: The triad of adipose tissue, smooth muscle cells,
Note: Biliary duct epithelial cells are increased in bile and blood vessels characterizes angiomyolipoma
duct adenoma/hamartoma, cirrhosis, and focal nodular (AML). Extramedullary hematopoiesis is a frequent
hyperplasia and absent in hepatic adenoma and finding in angiomyolipomas, which brings myelolipoma
hepatocellular carcinoma. into the differential diagnosis; however, myelolipoma
does not have HMB-45 positive myoid cells. HMB-45
positive myoid cells are a diagnostic feature of AML.
3. D Hepatic AMLs, like the more common renal AML, are
Note: Anaphylactic shock may develop as an occasional often found in patients with tuberous sclerosis.
complication of hydatid cyst aspiration. Echinococcus
granulosus is a tapeworm which causes infection in 7. B
humans; it has a predilection for forming cysts in the Note: This Pap stained aspirate shows benign hepato-
liver. The cysts are typically large (>4 cm) and the cytes characterized by abundant eosinophilic cytoplasm,
aspirated fluid may contain fragments of laminated a normal nuclear to cytoplasmic ratio, and small
membranes, scolices, and hooklets. The hooklets are monomorphic nuclei with small pinpoint nucleoli. Bile
typically pale to unstained. Hepatic abscesses can be duct epithelial cells are not seen. This group of hepato-
caused by bacteria, fungi or amoebas. Smears prepared cytes could represent a hepatic adenoma, focal nodular
from FNA of liver abscesses caused by fungi or bacteria hyperplasia, normal liver or a regenerative nodule in
contain abundant necrotic material and numerous cirrhosis. Hepatic adenomas are more common in young
neutrophils. Amebic abscesses contain necrotic debris women, particularly those with a long history of oral
resembling “anchovy paste” with few if any contraceptives. Hemangiomas and gastrointestinal
inflammatory cells. Miliary tuberculosis is characterized stromal tumors are composed of spindle cells. The cells
by numerous granulomas. of hepatocellular carcinoma have a higher nuclear to
cytoplasmic ratio with pleomorphic nuclei and usually
4. D show endothelial wrapping which gives the edge of
Note: Fibrolamellar hepatocellular carcinoma is a rare cellular groups a smooth outer contour, in contrast to
variant of hepatocellular carcinoma that presents in the bumpy peripheral outline shown here.
younger (<35 years old) female patients who do not have
cirrhosis or a history of liver disease. FNA shows large 8. D
oncocytic hepatocytes with occasional intracytoplasmic Note: This FNA is from a regenerative nodule in a
hyaline globules and abundant fibrous tissue. In general cirrhotic liver. The cells shown are mostly normal
these tumors portend a better prognosis than the classic appearing hepatocytes with mild variation in size and
type of hepatocellular carcinoma. an occasional prominent nucleolus. Although the cell
144 Cytopathology Review

at the center is enlarged but the nuclear-to-cytoplasmic 13. D


ratio is preserved. Endothelial wrapping of hepatocytes Note: This image shows a cohesive tissue fragment
is not seen. composed of malignant hepatocytes with prominent
nucleoli, and an occasional intranuclear inclusion
9. A (bottom left corner). The groups of cells have smooth
Note: The large group of cells is a group of normal bile outlines, suggesting endothelial wrapping around the
duct epithelial cells characterized by a cohesive sheet malignant hepatocytes, which is a diagnostic finding in
of small epithelial cells with evenly spaced nuclei hepatocellular carcinoma. The immunohistochemical
(“honeycomb” appearance). Three normal hepatocytes stains confirmed the diagnosis of hepatocellular
(one of which is binucleated) are in the upper right carcinoma.
corner.
14. C
10. A Note: Normal hepatocytes may be present in aspirates
Note: The aspirate shown is composed of pleomorphic from other organs (especially from the right side) if the
cuboidal/columnar cells with high nuclear/cytoplasmic needle inadvertently enters the liver. Hepatocytes have
ratios, uneven spacing of nuclei and prominent nucleoli. abundant eosinophilic cytoplasm, well-defined cell
A vague acinar/glandular structure can be appreciated. borders, and centrally located nuclei with prominent
There is no cytoplasmic bile or endothelial rimming to pinpoint nucleoli. Binucleation and multinucleation (as
suggest hepatocellular carcinoma. The patient also had shown here) are common.
primary sclerosing cholangitis, the most common risk
factor of cholangiocarcinoma in the U.S. The elevated 15. A
serum CA19-9 and positive immunohistochemical Note: Hepatic metastases from colon are composed of
staining for CK7 and MOC-31 also support the diagnosis columnar cells with cigar-shaped nuclei in a “picket
of cholangiocarcinoma. Metastatic colon adenocarci- fence” alignment at the periphery of cell groups. The
noma could also be positive for MOC-31 but it is background contains necrotic material. Immunohistoche-
negative for CK7. mical CDX2 positivity favors a colonic primary.
Cholangiocarcinoma and lung adenocarcinoma are
11. D usually positive for CK7. Melanoma and hepatocellular
Note: This Pap stained aspirate shows a cellular carcinoma are negative for both CK7 and CDX2.
specimen composed of broad trabeculae wrapped by
endothelial cells. These thickened trabeculae are greater 16. C
than 2 cells thick and composed of hepatocytes with an Note: The majority of hepatocytes shown have large
increased nuclear to cytoplasmic ratio. The slender lipid vacuoles replacing the normal cytoplasm. The
endothelial cells can be seen at the periphery of each vacuoles vary in size from many very small vacuoles to
group of cells, so called “endothelial wrapping”. Normal single large vacuoles. The large vacuoles displace the
liver trabeculae are not more than 2 cells thick and are nucleus to an eccentric position in the cell. Steatosis
composed of cells with more abundant cytoplasm. occurs in many disease states, most commonly in
association with alcohol use, diabetes, obesity, drugs,
12. E and hepatitis C chronic infection. Steatosis is usually a
Note: Metastatic colonic adenocarcinoma has elongated diffuse process but can cause localized radiographic
cigar-shaped nuclei with a peripheral picket fence abnormalities. In addition, large regenerative nodules in
pattern, often in a background of necrosis, as shown in cirrhosis may have significant steatosis. The hepatocyte
this figure. While cholangiocarcinoma should be a nuclei are bland and the nuclear to cytoplasmic ratio is
consideration in the differential diagnosis of any low, excluding hepatocellular carcinoma and metastatic
adenocarcinoma in the liver, metastatic adenocarcinomas adenocarcinoma. Metastatic renal cell carcinoma can
to the liver are far more common than cholangio- also show vacuolated cytoplasm; however, the vacuoles
carcinoma. Furthermore the morphology of this group are uniformly small, giving the cytoplasm a bubbly
of cells favors the diagnosis of a colonic primary. appearance and the nuclei will show pleomorphism with
Liver 145

finely granular chromatin. Benign macrophages can be biopsied when the radiologic features of the lesion are
“bubbly” as well, but will have a small centrally located suspicious for malignancy.
kidney-shaped nucleus.
20. D
17. A Note: This medium power picture shows a cellular
Note: The cells shown are malignant cells with high aspirate composed of widened trabeculae of hepatocytes
nuclear-to-cytoplasmic ratios. The malignant squamous enveloped by endothelial cells, giving the cellular
cells have dark condensed chromatin. An orangeophilic borders smooth, rounded contours. These features are
squamous pearl is also shown. characteristic of well-differentiated hepatocellular
carcinoma.
18. C
Note: The aspirate shown here consists of cohesive 21. B
clusters and single cells with very little to no cytoplasm Note: The cells in the image appear obviously malignant
and nuclei with fine chromatin and indistinct nucleoli. with large tumor cells containing prominent nucleoli.
Scattered apoptotic cells are also seen. These cytomor- Immunohistochemical stains are helpful in differentia-
phologic features support the diagnosis of metastatic ting poorly differentiated hepatocellular carcinoma
small cell carcinoma. (HCC) from other tumors. Glypican-3 is positive in
approximately 80% of HCCs. Like antibodies to
19. A polyclonal CEA, antibodies to CD10 show a similar
Note: Aspirates from cirrhotic livers show variable canalicular staining pattern in HCC. HCCs are typically
cellularity and are mostly composed of reactive/ negative for both cytokeratin CK7 and CK20.
regenerative hepatocytes. This image shows reactive
appearing hepatocytes, fragments of fibrous tissue and 22. D
benign bile duct epithelium consistent with cirrhosis. Note: The tumor cells have a characteristic plasmacytoid
The hepatocyte groups in cirrhosis have irregular borders appearance with focal rosette formation. These
as compare to the smooth borders of the cell groups in morphologic features in combination with the clinical
well-differentiated hepatocellular carcinoma. The presentation of severe diarrhea strongly suggest a
presence of bile duct epithelium is helpful in excluding metastatic neuroendocrine tumor in the liver. Tumors
a neoplasm such as hepatic adenoma and hepatocellular with a plasmacytoid appearance also include
carcinoma. Fragments of fibrotic stroma, as in this case, plasmacytoma, melanoma and breast carcinoma. The
are often present in aspirates from cirrhotic livers. The focal rosette formation in the current smear is a more
diagnosis of cirrhosis can be suggested on the basis of characteristic feature for a neuroendocrine tumor. Final
these FNA findings, but a histologic examination of a diagnosis of a neuroendocrine tumor relies on positive
core needle biopsy is required for the final diagnosis. immunohistochemical stains for neuroendocrine
Regenerative nodules of cirrhosis are occasionally markers.
146 Cytopathology Review

11 Gastrointestinal Tract,
Pancreas and Biliary Tree
Fang Fan

QUESTIONS
1. Which of the following statements is true regarding esophageal squamous cell carcinoma in the United
States?
A. The majority are associated with Barrett’s esophagus
B. The majority are associated with human papilloma virus (HPV) infection
C. The majority are associated with Helicobacter pylori infection
D. The majority are associated with alcohol and tobacco use
E. The majority are associated with nutritional deficiencies

2. Helicobacter pylori infection is associated with all of the following diseases, EXCEPT:
A. Chronic gastritis
B. Duodenal ulcer
C. Gastric adenocarcinoma
D. Gastric mucosa-associated lymphoid tissue (MALT) lymphoma
E. Gastrointestinal stromal tumor (GIST)

3. The majority of gastrointestinal stromal tumors (GIST) are associated with gain-of-function mutations
of the gene encoding which of the following proteins?
A. Tyrosine kinase c-KIT
B. Platelet-derived growth factor receptor α (PDGFRA)
C. Epidermal growth factor receptor (EGFR)
D. Beta-catenin
E. Cyclin D1

4. In differentiating pancreatic endocrine neoplasm (PEN) from solid-pseudopapillary neoplasm (SPN)


of the pancreas, which of the following immunohistochemical stains, if positive, supports the diagnosis
of SPN?
A. Keratin
B. CD56
C. Beta-catenin
D. Synaptophysin
E. Trypsin

5. Which of the following cancer related genes is the most frequently activated in pancreatic cancer?
A. KRAS
B. CDKN2A (p16)
C. SMAD4
D. P53
E. BRCA-2
Gastrointestinal Tract, Pancreas and Biliary Tract 147

6. When comparing pancreatic mucinous cystic neoplasm (MCN) with intraductal papillary mucinous
neoplasm (IPMN), all of the followings are correct, EXCEPT:
A. MCNs usually involve the body or tail while IPMNs usually occur in the head of the pancreas
B. MCNs occur more frequently in women and IPMNs in men
C. MCNs do not connect to the pancreatic duct and IPMNs do
D. Microscopically, both tumors have an ovarian-like stroma
E. Distinction between benign and malignant lesions depends on evidence of stromal invasion

7. A 6-year-old boy complained of abdominal pain. Image studies revealed a well-demarcated heterogeneous
mass in the pancreas. Fine-needle aspiration (FNA) yielded cohesive epithelial groups and spindle cell
stromal fragments. Squamoid corpuscles were seen in the cellblock sections. These findings are
characteristic of which tumor?
A. Pancreatoblastoma
B. Teratoma
C. Pancreatic endocrine neoplasm
D. Solid pseudopapillary neoplasm
E. Acinar cell carcinoma

8. This esophageal bushing specimen was collected from


a 45-year-old man. What is the correct diagnosis?
A. Epithelial repair
B. Herpes simplex infection
C. Cytomegalovirus infection
D. Barrett esophagus
E. Squamous cell carcinoma

9. This smear was prepared from an endoscopic


ultrasound-guided fine needle aspiration (EUS-FNA)
of a gastric mass. What is the most likely diagnosis?
A. Chronic gastritis
B. Gastric adenocarcinoma
C. Gastric carcinoid
D. Diffuse large B-cell lymphoma
E. Gastrointestinal stromal tumor
148 Cytopathology Review

10. These cell groups in a smear from a EUS-FNA of


pancreas represent which of the following?
A. Normal pancreatic acinar cells
B. Normal pancreatic ductal cells
C. Normal pancreatic islet cells
D. Gastrointestinal epithelium (contaminant)
E. Mesothelial cells

11. This biliary brushing specimen was collected from a


patient with a clinical history of primary sclerosing
cholangitis. What is the correct diagnosis?
A. Reactive atypia
B. Adenocarcinoma
C. Hepatocellular carcinoma
D. Squamous cell carcinoma
E. Small cell carcinoma

12. An EUS-FNA was performed to sample a duodenal


wall mass. The smear is shown here. Immunohisto-
chemical stain was negative for smooth muscle actin
and CD117, but positive for DOG-1. What is the
correct diagnosis?
A. Gastrointestinal stromal tumor (GIST)
B. Leiomyoma
C. Schwannoma
D. Angiosarcoma
E. Spindle cell carcinoid
Gastrointestinal Tract, Pancreas and Biliary Tract 149

13. A 56-year-old man had a prior history of right


nephrectomy 8 years ago. A follow-up CT scan
revealed a pancreatic body mass. An FNA was
performed and shown here. What is the most likely
diagnosis?
A. Pancreatic adenocarcinoma
B. Pancreatic endocrine neoplasm
C. Pancreatic acinar cell carcinoma
D. Pancreatic solid pseudopapillary neoplasm
E. Metastatic renal cell carcinoma

14. A 42-year-old woman who had a history of von Hippel


Lindau (VHL) syndrome presented with a pancreatic
head mass. An endoscopic ultrasound-guided fine
needle aspiration was performed and is shown here.
What is the most likely diagnosis?
A. Hemangioblastoma
B. Metastatic renal cell carcinoma
C. Metastatic small cell carcinoma
D. Pancreatic adenocarcinoma
E. Pancreatic endocrine neoplasm

15. This smear was collected from a 42-year-old man’s


pancreatic head cyst. The cyst fluid contained a high
level of amylase. What is the correct diagnosis?
A. Pseudocyst
B. Serous cystadenoma
C. Mucinous cystic neoplasm (MCN)
D. Intraductal papillary mucinous neoplasm (IPMN)
E. Ductal adenocarcinoma
150 Cytopathology Review

16. This epithelial group seen in a pancreatic EUS-FNA


smear represents which of the following?
A. Duodenal epithelium
B. Pancreatic ductal epithelium
C. Pancreatic acinar cells
D. Pancreatic ductal adenocarcinoma
E. Pancreatic acinar cell carcinoma

17. A 65-year-old man had symptoms of jaundice and


weight loss and was discovered to have a pancreatic
head mass. An endoscopic ultrasound-guided fine
needle aspiration (EUS-FNA) was performed and the
obtained cells are shown in this smear. What is the
correct diagnosis based on the image shown here?
A. Chronic pancreatitis with reactive atypia of ductal
epithelium
B. Pancreatic intraepithelial neoplasia (PanIN)
C. Pancreatic ductal adenocarcinoma
D. Pancreatic endocrine neoplasm
E. Pancreatic acinar cell carcinoma

18. This EUS-FNA smear was collected from a pancreatic


body cyst from a 56-year-old woman. What is the
correct diagnosis?
A. Pseudocyst
B. Retention cyst
C. Gastrointestinal (GI) contaminant mucin and
epithelium
D. Mucinous cystic neoplasm
E. Mucinous adenocarcinoma
Gastrointestinal Tract, Pancreas and Biliary Tract 151

19. A 30-year-old woman presented with abdominal pain


and discomfort. An abdominal CT-scan revealed a
pancreatic tail mass. An EUS-FNA was performed
and shown here. What is the best diagnosis?
A. Intraductal papillary mucinous neoplasm
B. Solid pseudopapillary neoplasm
C. Acinar cell carcinoma
D. Pancreatic ductal adenocarcinoma
E. Metastatic carcinoma

20. When obtaining such a smear during the on-site


adequacy assessment from an EUS-FNA specimen of
a pancreatic cyst, which of the following step would
be appropriate?
A. Send the specimen for microbiology culture
B. Send the specimen for flow cytometry study
C. Send the specimen for cytogenetic analysis
D. Send the specimen for chemical analysis (CEA and
amylase)
E. Send the specimen for mucicarmine stain

21. This image was taken from a biliary brushing


specimen. What is the correct interpretation?
A. Benign ductal epithelium
B. Papilloma
C. Adenoma
D. Carcinoma in situ
E. Invasive adenocarcinoma
152 Cytopathology Review

22. This FNA was performed on a pancreatic tail mass


in a 45-year-old man. What is the correct diagnosis?
A. Squamous metaplasia of pancreatic ducts in chronic
pancreatitis
B. Squamous cell carcinoma
C. Pancreatoblastoma
D. Acinar cell carcinoma
E. Melanoma

23. This EUS-FNA smear was collected from a pancreatic


neck mass in a 45-year-old man. What is the correct
diagnosis?
A. Pancreatic acinar cell carcinoma
B. Pancreatic ductal adenocarcinoma
C. Pancreatic endocrine neoplasm
D. Diffuse large B-cell lymphoma
E. Metastatic small cell carcinoma

24. This epithelial group is seen in a pancreatic EUS-FNA


smear. What does it represent?
A. Benign acinar group
B. Benign ductal epithelial group
C. Normal islet cell group
D. Well-differentiated ductal adenocarcinoma
E. Metastatic adenocarcinoma
Gastrointestinal Tract, Pancreas and Biliary Tract 153

25. This smear represents an EUS-FNA smear from an


ampullary mass. What is the correct diagnosis?
A. Benign ampullary mucosa with reactive changes
B. Ampullary adenoma
C. Ampullary adenocarcinoma
D. Gastrointestinal stromal tumor
E. Lymphoma

26. This cellular smear was obtained from a pancreatic


mass. Which of the following stains is most
appropriate?
A. CK7
B. Chromogranin
C. CD45
D. CD117
E. Villin
154 Cytopathology Review

ANSWERS

1. D 8. B
Note: In the United States, most esophageal squamous Note: The smear shows cells with multinucleation,
cell carcinomas are associated with alcohol and tobacco nuclear molding and a ground-glass appearance of the
use. Adenocarcinoma of esophagus is associated with nuclei, these are characteristic features of Herpes virus
Barrett esophagus. Human papilloma virus infection and inclusions.
nutritional deficiencies have been implicated in
esophageal squamous cell carcinoma in some regions. 9. D
H. pylori infection is not associated with esophageal Note: The image shows isolated highly atypical cells
squamous cell carcinoma. with large nuclei, scant cytoplasm, vesicular chromatin
and prominent nucleoli. The background appears
2. E necrotic. These cytologic features are consistent with
Note: H. pylori infection is the major cause of chronic diffuse large B-cell lymphoma. Adenocarcinoma cells
gastritis and duodenal ulcer and it is associated with an form cohesive groups and have more abundant
increased risk for gastric adenocarcinoma and gastric cytoplasm. Carcinoid tumor is composed of smaller
MALT lymphoma. GIST is not associated with H. pylori monomorphic cells with a finely dispersed “salt-and-
infection. pepper” chromatin in their nuclei. Gastrointestinal
stromal tumor cells are typically spindle-shaped.
3. A
Note: Approximately 80% of GIST has an oncogenic 10. A
mutation of the gene encoding the tyrosine kinase Note: This figure shows normal pancreatic acinar cells
which have eccentrically located round nuclei with
C-KIT, which is the receptor for stem cell factor. A few
inconspicuous nucleoli, abundant pyramid-shaped
GISTs have gene mutations of PDGFRA. In sporadic
granular cytoplasm and indistinct cell borders.
GISTs, c-KIT and PDGFRA mutations are mutually
exclusive. GISTs are considered to arise from the
11. B
interstitial cells of Cajal, which express C-KIT (also Note: The brushing specimen shows features typical of
known as CD117). adenocarcinomas. The upper left corner has a group of
highly pleomorphic tumor cells (compare to the benign
4. C
group in the center of the image). Tumor cells have
Note: PEN and SPN can be both positive for keratin, irregular nuclear contours, with hyperchromasia and
CD56, synaptophysin and negative for trypsin. However, prominent macronucleoli.
SPN shows positive nuclear staining with antibodies to
beta-catenin. PEN is negative for beta-catenin. 12. A
Note: Up to 15% of GISTs are CD117 negative. DOG-1
5. A (an acronym for “Discovered On GIST-1”) is considered
Note: KRAS is activated by point mutation in 80-90% a more sensitive and specific marker for GIST than CD117
of all pancreatic adenocarcinomas. p16, SMAD4 and p53 and is useful in detecting CD117 negative GISTs.
are commonly inactivated in pancreatic cancer.
13. E
6. D Note: The smear shows a group of tumor cells with abun-
Note: MCNs have ovarian-like stroma in the cyst wall dant clear cytoplasm and surrounded by strand-like
magenta-colored basement membrane material. These
while IPMNs do not. The other answers are correct.
cytologic features are characteristic of renal cell
7. A carcinoma.
Note: The clinical presentation and the findings of acinar 14. E
cells and squamous cells in the fine needle aspiration Note: The smear shows abundant loose clusters and
smears are characteristic of pancreatoblastoma. isolated cells with uniform round nuclei, finely stippled
Gastrointestinal Tract, Pancreas and Biliary Tract 155

chromatin and moderate amount of cytoplasm. These are mucinous epithelium and GI contaminants is always
cytologic features of neuroendocrine tumors. VHL problematic in these EUS-FNA specimens. In general,
syndrome was originally described as a constellation the GI contaminant mucin is thin and appears “dirty”,
of cerebellar hemangioblastoma and vascular lesions of and the neoplastic mucin is usually thick and colloid-
the retina. However, it was shown later that these like.
patients may also have a variety of tumors such as renal
cell carcinoma, pheochromocytoma and pancreatic 19. B
endocrine neoplasms. Note: The thin and branching papillary structures shown
here are characteristic of solid pseudopapillary
15. A neoplasms of pancreas. The papillary fronds are lined
Note: The smear shows granular debris, macrophages by small monotonous and bland cells. The cells have
and bile without epithelial cells, consistent with contents round or oval nuclei with finely granular chromatin,
of a pseudocyst. The aspirates from other four neoplasms inconspicuous nucleoli, and indistinct cell borders.
suggested as possible answers would have all contained Nuclear grooves and intranuclear inclusions may be seen
neoplastic epithelial cells. Amylase is almost always under high magnification (not shown here).
elevated in pseudocyst, although it can also be elevated
in IPMNs and some MCNs. A low amylase level is usually 20. D
taken as evidence against the diagnosis of pseudocyst. Note: The aspirate contains thick colloid-like mucin.
Cyst fluid CEA level is a very useful adjunct study in
16. A evaluating pancreatic cystic lesions and should be
Note: This large flat sheet epithelium containing routinely performed in all cystic lesions of the pancreas.
scattered goblet cells (so called “starry sky” appearance) When the fluid CEA level is greater than 800 units, the
is typical of duodenal epithelium contaminating the lesion is most likely a neoplastic mucinous cyst.
pancreatic sample obtained by the EUS-FNA procedure;
it should not be mistaken for neoplastic tissue. 21. A
Note: This group of cells has an organized architecture.
17. C The cells at the periphery of the group are columnar with
Note: This cell group is a flat sheet with disordered bland basally located nuclei and abundant delicate
arrangement. The cells have irregular nuclear cytoplasm. These features are typical of benign ductal
membranes, open and fine chromatin with small epithelium as seen in a biliary brushing specimen.
nucleoli, and abundant mucinous cytoplasm, as compare
to normal orderly arranged honeycomb-like appearance 22. B
of benign ductal epithelial cells. The degree of Note: These are obviously malignant keratinizing
anisonucleosis and nuclear irregularity exceed those squamous cells showing pleomorphic tumor cells with
seen in reactive atypia associated with chronic anaplasia and keratinized cytoplasm. The patient was
pancreatitis. PanINs are considered precursor lesions for later discovered to have a history of head and neck
pancreatic cancer but do not present clinically as a mass squamous cell carcinoma with extensive metastases,
lesion. Pancreatic endocrine neoplasm and acinar including the nodule sampled for the present smear.
cell carcinoma have smaller and monomorphic tumor
cells. 23. B
Note: The smear shows a loosely cohesive disorganized
18. D group composed of highly pleomorphic tumor cells,
Note: The aspirate is composed of thick mucin and bland these are features of pancreatic ductal adenocarcinoma.
columnar mucinous cells. These findings from a
pancreatic body cyst of a woman are characteristic of a 24. B
mucinous cystic neoplasm. No overt cytologic atypia is Note: This epithelial group is composed of small
seen to suggest malignancy. A definitive diagnosis of uniform cells arranged in a honeycomb sheet. The cells
mucinous adenocarcinoma is made histologically by have round nuclei with smooth nuclear membrane, fine
identifying stromal invasion in the surgically resected chromatin and small nucleoli. These are features of
specimens. The distinction between bland neoplastic benign pancreatic ductal epithelial group.
156 Cytopathology Review

25. C 26. B
Note: This cell group is composed of epithelial cells with Note: The cells are uniform with fine and open
marked pleomorphism. Some cells have large chromatin and scant cytoplasm. These cytologic features
cytoplasmic mucin vacuoles. These are features of are highly suggestive of a pancreatic endocrine
adenocarcinoma. neoplasm. Therefore, an immunohistochemical stain for
chromogranin is most appropriate.
12 Laboratory Management,
Quality Control and
Quality Assurance
Ivan Damjanov

QUESTIONS
1. All of the following are part of the requirements by the Clinical Laboratory Improvement Amendments
of 1988 (CLIA 88), EXCEPT:
A. Prospective rescreening of 10% of negative Pap smears
B. Five-year retrospective review of all negative Pap smears from women with a newly diagnosed high-
grade squamous intraepithelial lesion
C. Correlation of cytologic-histologic findings
D. Implementation of Pap Proficiency Test
E. Pathologists review of all abnormal Pap smears

2. All of the following statements are true regarding the qualifications for a Technical Supervisor (TS) of
a cytology laboratory, EXCEPT:
A. Must be licensed to practice medicine or osteopathy
B. Must have certification in anatomic pathology
C. Must have subspecialty certification in cytopathology
D. A cytotechnologist is not qualified regardless of experience
E. The Laboratory Director may assume the role of Technical Supervisor

3. All of the following are activities of a cytotechnologist as defined by CLIA88, EXCEPT:


A. Screening Pap smears and reporting normal Pap smears without pathologist’s review
B. Screening effusion specimens and reporting negative results without pathologist’s review
C. Performing adequacy assessments during fine needle aspirations
D. Performing cytopreparation procedures in the lab
E. Performing quality control procedures
4. A Pap test performed under which of the following situations is considered a Diagnostic Pap test as
defined by Medicare?
A. History of early onset of sexual activity (<16 years of age)
B. History of positive human immunodeficiency virus infection
C. History of multiple sexual partners (>five)
D. Daughter of a woman with diethylstilbestrol use during pregnancy
E. Significant complaint referable to the cervix
158 Cytopathology Review

5. Which of the following health care providers can classify a Pap test as screening, high risk or diagnostic
Pap test?
A. The referring physician
B. The screening cytotechnologist
C. The cytopathologist
D. The cytopathology laboratory director
E. A Medicare agency

6. Which of the following dyes stains the nuclei in the Papanicolaou stain?
A. Hematoxylin
B. Orange-G
C. Eosin Y
D. Light green SF
E. Bismarck brown Y

7. How often should the staining quality of cytologic preparations be checked?


A. Daily
B. Weekly
C. Bi-weekly
D. Monthly
E. Quarterly

8. All of the following are possible causes of false-negative Pap results, EXCEPT:
A. The specimen collection procedure does not sample lesional cells
B. The lesional cells are not transferred from the collection device to the glass slide
C. The abnormal cells are missed in the screening
D. The abnormal cells are misinterpreted in the interpretation
E. The specimen is not sent for high-risk HPV molecular testing

9. Which of the following statements is true regarding the workload limit established by CLIA88 for
cytotechnologist?
A. Maximum 100 slides per 24-hr period for primary screening
B. Maximum 100 slides per 12-hr period for primary screening
C. Maximum 120 slides per 24-hr period for primary screening
D. Maximum 120 slides per 12-hr period for primary screening
E. It varies depending on qualification and years of experience

10. When counting slides for the workload purpose, which of the following slides counts a full slide?
A. A ThinPrep CSF slide
B. A SurePath pleural fluid slide
C. A ThinPrep urinary bladder washing slide
D. A cellular fine-needle aspiration smear
E. A cell block section
Laboratory Management, Quality Control and Quality Assurance 159

11. Which of the following statements regarding the Pap smear Proficiency Test (PT) is correct?
A. The test is administered every other year
B. The test is administered only in designated sites
C. Each participant evaluates 20 slides
D. Participants are allowed to select the type of Pap slides (direct smear, ThinPrep, SurePath) for the test
E. Pathologists are not allowed to use a prescreened set of slides

12. Which of the following statements is true regarding the minimum retention requirements for glass
slides/cell blocks/reports by College of American Pathologists (CAP)?
A. Gynecologic glass slides – 10 years
B. Non-gynecologic glass slides – 10 years
C. Fine needle aspiration glass slides – 10 years
D. Cell blocks – permanent
E. Cytopathology reports - permanent

13. All of the following categories in gynecologic cytology should be interpreted by a pathologist, EXCEPT:
A. Malignant cells
B. Low-grade squamous intraepithelial lesions (LSIL)
C. Atypical squamous cells of uncertain significance (ASCUS)
D. Reactive or repair
E. Unsatisfactory

14. According to the requirement of CLIA for the 10% rescreen of gynecologic cases, which of the following
statements is true?
A. At least 10% of each cytotechnologist’s Pap smears are rescreened
B. At least 10% of each cytotechnologist’s negative Pap smears are rescreened
C. The slides can be rescreened for selected microscopic fields
D. Slides screened by a senior cytotechnologist (>10 years experience) are not subjected to rescreen
E. Slides screened by a board-certified pathologist are still subjected to rescreen

15. A 5-year retrospective review of previously negative gynecologic slides is performed in which of the
following cases under CLIA requirement?
A. A new diagnosis of Trichomonas vaginalis infection
B. A new diagnosis of Herpes simplex virus infection
C. A new diagnosis of atypical squamous cells of uncertain significance
D. A new diagnosis of low grade squamous intraepithelial lesion
E. A new diagnosis of high-grade squamous intraepithelial lesion

16. All of the following data are good measures of a cytology lab’s performance in gynecologic cytology,
EXCEPT:
A. The lab’s ASCUS percentage
B. The lab’s ASC/SIL ratio
C. The lab’s high-risk HPV positivity rate for ASCUS cases
D. The lab’s cytology/biopsy correlation concordance
160 Cytopathology Review

17. A pathologist performed an adequacy assessment in an ultrasound-guided fine needle aspiration of a


thyroid nodule. The first aspirate pass was determined to be inadequate, and the subsequent 2nd and
3rd passes were determined to be adequate. Direct smear and cell block slides were made for the final
interpretation and report of the case. All of the following are correct for the cytology billing, EXCEPT:
A. Fine needle aspiration diagnosis and report (88173x1)
B. First adequacy assessment performed by the pathologist (88172x1)
C. Second adequacy assessment performed by the pathologist (88177x1)
D. Direct smear preparation (88104x1)
E. Cell block preparation (88305x1)

18. A pathologist performed a fine needle aspiration of a right parotid mass. The first pass was inadequate,
the second pass was adequate. Direct smears and a cell block were prepared. All of the following are
correct for the cytology billing, EXCEPT:
A. FNA performance by a pathologist without image guidance (10021x2)
B. Fine needle aspiration diagnosis and report (88173x1)
C. First adequacy assessment performed by the pathologist (88172x1)
D. Second adequacy assessment performed by the pathologist (88177x1)
E. Cell block preparation (88305x1)

19. What is a positive predictive value of a test?


A. The proportion of individuals with a positive test result who actually have the disease
B. The proportion of actual positive patients that are correctly identified as positive by the test
C. The proportion of actual negative patients that are correctly identified as negative by the test
D. The proportion of individuals with a negative test result who do not have the disease

20. Which of the following findings in a cytology specimen should prompt a communication to the clinician
and a written documentation that the clinician was informed about the cytopathology findings?
A. Low-grade squamous intraepithelial lesion in a Pap smear
B. Identification of Actinomyces spp. in a Pap smear
C. Metastatic squamous cell carcinoma in a neck mass of a patient with a history of oral squamous cell
carcinoma
D. Metastatic adenocarcinoma in a pleural effusion of a patient with pneumonia
E. Adenocarcinoma in a pancreatic mass of a patient with pancreatic head mass, jaundice and weight loss
Laboratory Management, Quality Control and Quality Assurance 161

ANSWERS

1. A as a screening (high-risk) Pap test in the situations A to


Note: Mandated rescreening of 10% of all negative Pap D in the answers. When there is a significant complaint,
smears was published in the 1960s as part of the CLIA or any sign or symptom related to the gynecologic
67. All the others (B to E) were part of the CLIA 88 system, the Pap test is defined as a Diagnostic Pap test.
requirements. CLIA is a program administered by Other conditions included in a Diagnostic Pap test are
Centers for Medicare and Medicaid Services (CMS). previous history of cancer of the vagina, cervix, or
CMS is one of the divisions of the U.S. Department of uterus; previous abnormal Pap; and current abnormal
Health and Human Services, parallel to other divisions findings of vagina, cervix, uterus and ovaries.
such as National Institute of Health (NIH), Food and
Drug Administration (FDA) and Centers for Disease 5. A
Control (CDC). A clinical laboratory must have a CLIA Note: Only the referring physician can classify a Pap
certificate to bill and receive Medicare or Medicaid test as screening, high-risk or diagnostic. When a
payments. A clinical laboratory must be accredited by cytology laboratory discovers that a referring physician
the Joint Commission or the College of American may have misclassified the test, the lab may contact the
Pathologists (CAP) in order to obtain a CLIA certificate. referring physician and amend the order after obtaining
The CAP surveys are performed once every 2 years and the authorization.
have been unannounced since 2006.
6. A
2. C Note: The hematoxylin stains the nucleus, the remaining
Note: A Technical Supervisor of a cytology laboratory dyes listed stain the cytoplasm with different hues of
should be a certified anatomic pathologist who is licensed pink, orange, yellow, green and blue depending on the
to practice medicine or osteopathy. A subspecialty cytoplasmic keratin content.
certification in cytopathology is not required. A
cytotechnologist who has at least 3 years of full-time 7. A
experience within the past 10 years is qualified to serve Note: The technical quality of all preparations performed
as a General Supervisor of a cytology laboratory, but not in a cytology laboratory should be checked daily by the
as a Technical Supervisor. In many cytology laboratories, pathologist or supervisor-level cytotechnologist.
the cytology Laboratory Director has the role of the
Technical Supervisor of the lab. Responsibilities of the 8. E
TS include establishing Quality Control procedures in Note: The above A and B are also referred to as
the lab, resolving technical issues, establishing a workload “sampling error”, and C and D are referred to as
limit for each cytotechnologist and performing “laboratory error”. The sampling error or laboratory
semiannual evaluations of cytotechnologists. error may result in false-negative Pap results. An
educational note is usually attached to all negative Pap
3. B test reports stating the fact that the Pap test is a screening
Note: CLIA88 requires pathologists to review all non- test for cervical cancer with inherent false-negative
gynecologic specimens. The other activities can be results. Answer E is not relevant here.
performed by cytotechnologists.
9. A
4. E Note: The workload limit is 100 slides per 24-hour
Note: The Pap test is classified as a screening (routine) period with the minimum amount of time examining
Pap test, a screening (high-risk) Pap test, and a slides being 8 hours. Pathologists who screen and
diagnostic Pap test by Medicare. A Pap test is defined interpret previously unscreened gynecologic or non-
as a screening (routine) Pap test when a woman has no gynecologic slides must adhere to the same workload
current signs or symptoms referable to the female limit. However, pathologists who review the previously
reproductive system, no previous abnormal Pap and no screened gynecologic or non-gynecologic and FNA
high-risk factors for cervical cancer. A Pap test is defined slides are not subject to the workload limit.
162 Cytopathology Review

10. D 15. E
Note: When counting slides for the workload purpose, Note: Under CLIA, a new diagnosis of high-grade
the above mentioned A, B, C and E count as half slides, intraepithelial lesion or malignancy in Pap smears will
whereas a cellular fine needle aspiration smear with prompt a retrospective review of all available previously
cellular material covering more than half of a slide negative slides within the past 5 years.
counts as a full slide. A Pap smear slide (ThinPrep or
SurePath) counts as a full slide. 16. A
Note: A cytology laboratory’s ASCUS percentage is also
11. D determined by the screening population. If a lab has a
Note: The Pap PT test is administered every year on site. high ASCUS rate and appears to be over diagnosing
Each participant evaluates 10 gynecologic slides. ASCUS, but the lab’s ASC/SIL ratio is at the national
Participants are allowed to choose the type of Pap slides median level, this indicates that the lab serves a high-
they are accustomed to. Pathologists who ordinarily risk population. CAP compiles and publishes nationwide
review prescreened Pap slides are allowed to examine a percentile rankings for ASC/SIL ratios based on large
prescreened set that have been dotted by a surveys of laboratories. For example, the current 50th
cytotechnologist. percentile for ASC/SIL ratio for ThinPrep is 1.6 and the
upper limit (95th percentile) is 3.2.
12. C
Note: According to CAP, the minimum retention 17. D
requirements for gynecologic and non-gynecologic glass Note: Direct smear (88104) code is for non-gynecologic,
slides are 5 years, and for fine-needle aspiration glass non-FNA cases. The code 88173 covers all slide
slides are 10 years. Cell blocks should be retained for preparations for an FNA (including direct smears and
the same period as glass slides. Cytopathology reports ThinPrep), except only cell block preparation.
must be retained for a minimum of 10 years.
18. A
13. E Note: Only one procedure code (10021) may be billed
Note: The above categories A to D should be reviewed for one anatomic site regardless of the number of passes
and signed out by a pathologist. An unsatisfactory Pap made.
can be interpreted by a cytotechnologist without a
pathologist’s review. The reason for the unsatisfactory 19. A
diagnosis should be given in the report (e.g. insufficient Note: A is the definition for positive predictive value.
squamous cell component). Other definitions are as follows: B- sensitivity; C-
specificity; D- negative predictive value.
14. B
Note: At least 10% of each cytotechnologist’s negative 20. D
Pap smears are rescreened. The negative cases must Note: College of American Pathologists requires
include some cases from high-risk patients (as defined cytopathologists to formulate a policy regarding the
by the lab) and some random negative cases. The slides communication and documentation of clinically
must be rescreened in their entirety and the results are significant and unexpected findings to the clinicians. A
not reported until the rescreen is complete. Slides screened positive effusion diagnosis in a patient with no history
by all cytotechnologists including cytotechnologist of cancer is an unexpected finding and should be
supervisor and senior cytotechnologist are subjected to communicated to the clinician. Diagnoses to be defined
rescreen. However, slides screened by a board-certified as “significant/unexpected” or “critical” are determined
pathologist are not subjected to a rescreening procedure. by the cytopathology department.

You might also like